You are on page 1of 171

Avi Sayag Table of contents

Clinical Biochemistry

Topic 1: Reference range, requesting a test, sources of error, interpretation of results, specificity and sensitivity, predictive value.4 Topic 2: Laboratory signs of cellular damage. Basis of clinical enzymology7 Topic 3: Inborn errors of amino acid metabolism and lab diagnosis of CF9 Topic 4: inborn errors of carbohydrate and lipid metabolism Topic 5: Pathobiochemistry of inflammation. Topic 6: Pathobiochemistry of plasma proteins. Topic 7: Biochemical effects of tumors. Topic 8: Tumor markers in the diagnosis of malignant diseases Topic 9: Iron metabolism, hemochromatosis, iron deficiency anemia.. Topic 10: Lab diagnosis of hemoglobinopathies Topic 11: Lab diagnosis of hemolytic anemia.. Topic 12: Lab diagnosis of megaloblastic anemia.. Topic 13: Major lab characteristics of ALL and CLL Topic 14: Major lab characteristics of AML and CML Topic 15: Lab diagnostics of quantitative platelet disorders.. Topic 16: Inheritance of ABO blood group system and its clinical significance Topic 17: Inheritance and clinical significance of Rh blood group system Topic 18: Coagulopathies, lab control of anticoagulant treatment. Topic 19: Platelet function disorders.. Topic 20: Inherited thrombophilias Topic 21: Acquired thrombophilias Topic 22: Consumption coagulopathies. Topic 23: Lab tests of glomerular and tubular functions Topic 24: Clinical biochemistry of acute and chronic renal failures; tubulopathies. Topic 25: Disturbances of acid-base balance. Topic 26: Predominant water depletion; isoosmolar volume depletion Topic 27: Water and sodium excess; predominant sodium depletion Topic 28: Hypokalemia. Topic 29: Hyperkalemia. Topic 30: Pathogenesis of diabetes mellitus. Topic 31: Lab diagnosis and management of diabetes mellitus. Topic 32: Acute metabolic complications of diabetes mellitus. Topic 33: Hypoglycemia. Topic 34: Disorders and lab diagnosis of lipid metabolism.. Topic 35: Risk factors of atherosclerosis. Topic 36: Disturbances of uric acid metabolism Topic 37: Lab diagnostics of acute MI Topic 38: Hypovitaminosis and hypervitaminosis. Topic 39: Lab diagnosis of hepatocellular damage; evaluation of liver function. Topic 40: Lab diagnosis of cholestasis and liver cirrhosis. Topic 41: Pathobiochemistry and laboratory diagnostics of the gastrointestinal tract. Topic 42: Laboratory diagnosis of acute pancreatitis.. Topic 43: Clinical biochemistry of the hypothalamus and the pituitary. Topic 44: Pathobiochemistry and laboratory diagnosis of hypothyroidism and hyperthyroidism.. Topic 45: Hypocalcemia Topic 46: Hypercalcemia.. Topic 47: Clinical biochemistry of disturbances of the adrenal cortex. Topic 48: Clinical biochemistry of disturbances of the adrenal medulla. Topic 49: Clinical biochemistry of the reproductive system. Topic 50: Lab procedure and diagnosis of bone and skeletal disorders. Topic 51: Lab procedures in the diagnosis of muscle disorders. Topic 52: Clinical biochemistry at the extremes of age. Topic 53: Clinical biochemistry and lab diagnosis of porphyrias. Topic 54: Lab diagnostics of CNS diseases; lab investigation of the CSF. Topic 55:

Avi Sayag Practical topics

Clinical Biochemistry

Topic 1: Molecular genetic methods for the investigation of inherited diseases.14 Topic 2: Determination of Hb and Htc. Topic 3: Procedure of blood drawing, vacutainer tubes Topic 4: Topic 5: Topic 6: Principles of cell counting and differentials by hematology analyzers. Topic 7: Characterization of leukemic cells by morphology Topic 8: Characterization of leukemic cells by cytochemistry and immunophenotyping. Topic 9: Determination of ABO and Rh blood group.. Topic 10: Determination of PT. Topic 11: APTT assay. Topic 12: Thrombin time, D-dimer determination. Topic 13: Bleeding time, Detection of fibrin monomers... Topic 14: Lab methods for the determination of urea and creatinine.. Topic 15: Examination of urine (general and sediment analysis). Topic 16: Measurement of serum sodium and potassium.. Topic 17: Determination of glucose in serum; point of care tests. Topic 18: Topic 19: Cholesterol, HDL, LDL assays.. Topic 20: Triglyceride assay, visual test, lipoprotein electrophoresis.. Topic 21: Tests in the lab diagnosis of MI. Topic 22: Assay of serum bilirubin. Detection of bilirubin and UBG in the urine Topic 23: Topic 24: Tests in the investigation of bone metabolism.. Topic 25: Principles of chromatography and its application in diagnostics.15

Avi Sayag

Clinical Biochemistry

Topic 1
Reference range, requesting a test, sources of error, interpretation of results, specificity and sensitivity, predictive value
In clinical lab investigation there are 3 phases: 1. The pre-analytical phase: requesting the test, preparing the patient, collecting the sample, transporting it and storing it. 2. Analytical phase (manual or automatic) 3. Post-analytical phase: calculation of the results, validation of the results, consultation, reporting, making the archive and interpreting the results. Requesting a test: The test request should include the following: 1. Patient's name, sex, date of birth and insurance number; 2. Ward/clinic/address; 3. The name of the requesting doctor and ways to contact him/her. 4. The diagnosis; 5. The name of the test requested; 6. The type of specimen; 7. Date and time of sampling; 8. Relevant treatment; 9. Indication of potentially hazardous samples; Sources of error In the pre-analytical phase, any one of the steps can lead to an error: 1. Requesting the test in an inappropriate manner (switching names of patients, e.g.). Errors made during collection of the specimens: when blood specimen is collected, some variables can influence the results, such as posture, venous stasis, hemolysis, the labeling of the container. Also, the chemical composition of urine can change during collection: destruction of glucose by bacteria; urea is converted to ammonia, the pH decreases and phosphate precipitates; urobilinogen and porphobilinogen are oxidized. Hemolysis interferes with the determination of CK, because RBCs release adenylate cyclase (not CK!). AC converts 2 ADPs to ATP + AMP, and the ATP carries the second reaction, leading to overestimation of CK levels. 2. During storage, blood specimens can change: glycolysis occurs in RBCs (glycolysis in whole blood causes a 5% decrease in blood glucose level per hour), K+ and LDH are released from RBCs (hemolysis during venipuncture can lead to pseudohyperkalemia see topic 29), CO2 is lost from the specimen, organic phosphate esters are hydrolyzed, and labile enzymes lose their activity. In the analytical phase, human errors or instrumental errors can occur. In this category, the error can be systemic: accuracy vs. inaccuracy; precision vs. imprecision; or random errors. The ideal analytical method is accurate, precise, sensitive and specific. Precise: the result is the same if the procedure is repeated. It is assessed by repeated analysis (n=15-20): Within batch variation: the variation between the results of repeated tests in the same "batch" (on the same day, for the same person, with the same specimen collected). Day-to-day variation Precision is expressed as coefficient of variance: CV(%) = SDx100/mean. Therefore, if the SD is very small and approaches the mean, the CV(%) is close to 100% (= the test is very precise). Accurate: it gives the same result, that is, the deviation from the assigned value ("true" value). Accuracy is given by: 100(mean-Xt)/ Xt

Avi Sayag

Clinical Biochemistry

The mean value is the same in method A and B, but the scatter about the mean is less in method A than in method B. Therefore, method A is said to be more precise. Both method C and D are equally precise, but in method D the mean value differs from the true value. Method C is more accurate.

The last source of variation is the biological one. The variation can be within-individual or between-individual. The diet of a person, one's posture, the drug the person takes, etc. all influence the results and variation can occur from test to test for the same person. Age, race and sex can make the results different for different individuals. After we have requested the test and considered all possible errors on the way, we have finally received the results and have to interpret them. Reference interval: the interval between two reference limits, including the limits. It is designed as the central interval of values bounded by the lower and upper reference limit at certain designated percentiles, usually at percentile 2.5 and 97.5 (that is, it is supposed to include 95% of the reference population). When choosing the reference population, we have to select for the most appropriate ones, and exclude those who consume alcohol for example (unless we wish to establish reference range for alcoholics), obese people, drug abusers, fasting people or non-fasting, etc. Also, we have to select the group we wish to establish reference range for. Therefore, we partition the population according to sex, race, age, blood group, stage of menstrual cycle, pregnancy, exercise, fasting/non-fasting and so forth. As the reference limit refers to the mean value of the sampled population, there are other limits as well: 1. Medical decision limit (optimal cut-off values): in glucose levels, for example, DM is defined as the level of glycemia at which diabetes-specific complications occur rather than on deviations from a population-based mean. 2. Risk limit: for example, the risk limit for cholesterol is 5.8 mmol/L (a reference range does not exist, as cholesterol level exceeds the desired value in the majority of the population). 3. Panic value - results from a specimen that must be reported immediately to a clinician, i.e., of such severity as to mandate urgent therapy. For example, glucose levels < 2.6 mmol/L or > 26.9 mmol/L, Na+ < 120 mmol/L or > 158 mmol/L. When considering a lab test, it is important to know its sensitivity and specificity. Sensitivity is the incidence of true positive results or the ability of the method to measure low concentrations of the analyte. It is given by the formula: Sensitivity =

TP 100 TP + FN TN 100 TN + FP

Specificity is the incidence of true negative results, and the test is not subject to interference by other substances. It is given by the formula: Specificity =

When judging the positive or negative results, the predictive value should be considered.

Avi Sayag

Clinical Biochemistry TP 100 . This is the proportion of patients TP + FP TN 100 . This is the proportion of patients TN + FN

The predictive value of positive results =

with positive results who are correctly diagnosed. It reflects the probability that a positive result reflects the underlying condition being tested for. The predictive value of negative results =

with negative test results who are correctly diagnosed. For example, the predictive negative value of a low result of CRP in CSF ruling out bacterial meningitis is 97%. That is, if a person gets a low CRP result in CSF examination, he has a 97% chance that he really does not have a bacterial meningitis. Myoglobin lacks cardiac specificity as a marker. That is, if the myoglobin is elevated 4-8 hours following the onset of pain, but the ECG shows no sign of cardiac condition, then more cardiac-specific markers should be sought for. However, if myoglobin is not elevated 4-8 hours following the onset of pain, myocardial necrosis can be excluded. That is, myoglobin is sensitive. It is the only early marker measured (4-6 hours), despite its specificity.

Avi Sayag

Clinical Biochemistry

Topic 2
Laboratory signs of cellular damage. Basis of clinical enzymology
Cellular damage can be caused by physical agents, genetic defects, hypoxia, chemical agents, nutritional imbalance, immunological reactions, infections and aging. As the cell injury progresses, such as in MI, the ion pumps are the first to fail, leading to ion leakage to the ECF. Then, metabolites leak out (e.g. lactate, adenosine) and finally, as the membrane is damaged, macromolecules leak out (enzymes and proteins). Ions and metabolites drain to the intravascular space and less to the interstitial space, as opposed to macromolecules that drain to the interstitial space and slowly drained through the lymphatic system. There are 2 main groups of enzymes in the plasma: 1. Enzymes released from cells as a result of leakage or cell death. These enzymes have no known function in the blood. Serum enzymes in health are derived from the metabolic breakdown and turnover of cells and tissues. 2. Enzymes with clearly defined actions in the blood (e.g. coagulation factors, ACE). When measuring an enzyme, what influences the value obtained? 1. The rate of release from cells: 4 factors affect the rate a. Hypoxia/anoxia/drugs membrane permeability leakage b. Cell necrosis release of mitochondrial enzymes c. Increased synthesis (e.g. in bile duct obstruction, synthesis of enzymes (ALP, GGT) is induced) d. Duct obstruction (e.g. liver and pancreas) 2. The volume of distribution of the enzyme in the ECF 3. The rate of removal from the plasma (catabolism in RES or excretion in bile and urine). The clearance from the plasma also depends on the half life of the enzyme. For example, CK has a 1/2 life of 1.4 days and GPT - 6.3 days. 4. The presence of factors in the plasma that may affect the method of assay (i.e. inhibitors or activators of enzyme activity). Enzymes can have isoenzymes and isoforms: Isoenzymes: enzymes that differ in amino acid sequence but catalyze the same chemical reaction. These enzymes usually display different kinetic parameters (e.g. different KM values), or different regulatory properties. The existence of isoenzymes permits the finetuning of metabolism to meet the particular needs of a given tissue or developmental stage (e.g. LDH). In many cases, they are coded for by homologous genes that have diverged over time. Alleloenzymes represent enzymes from different alleles of the same gene, and isoenzymes represent enzymes from different genes that process or catalyse the same reaction. An example of an isozyme is glucokinase, a variant of hexokinase which is not inhibited by G6P. Its different regulatory features and lower affinity for glucose (compared to other hexokinases), allows it to serve different functions in cells of specific organs, such as control of insulin release by the beta cells of the pancreas, or initiation of glycogen synthesis by liver cells. Both of these processes must only occur when glucose is abundant, or problems occur. Isoenzymes can be organelle specific (enzymes of the mitochondria e.g.) or tissue specific. Isoforms: different forms of the same enzyme, which are not the result of genetic causes. Isoforms can be formed by post-translational modification on the protein component or on the non-protein component as well as by aggregation of enzyme molecules (CK-MM, CK-MB, CK-BB). We can use several techniques to differentiate isoenzymes and isoforms: 1. Zone electrophoresis 2. Isoelectric focusing 3. Differences in catalytic properties 4. Immunochemical methods 5. Selective inactivation 6. Ion exchange chromatography These factors are important in making the choice of enzyme activity measurement: Sensitivity, selectivity, time-course of elevation and technical errors.

Avi Sayag

Clinical Biochemistry

The international unit of enzyme activity: the amount of enzyme which, under given assay conditions, will catalyze the conversion of 1 mol of substrate per minute. Enzyme assays have not been standardized. Therefore, the reference range may change from lab to lab. The given assay conditions include: 1. The nature and concentration of the substrate 2. The direction of the reaction 3. The temperature 4. The pH, concentration and nature of the buffer 5. The presence of inhibitors and activators Katal = mol/sec 1 nkatal = 16.7 IU (international unit)

Avi Sayag

Clinical Biochemistry

Topic 3
Inborn errors of amino acid metabolism and lab diagnosis of CF
Amino acids (AA) are supplied to the body by diet (essential AA) and by de novo synthesis. As proteins are synthesized in the liver and in other tissues, the AA pool decreases. Deamination (degradation) of AA in the liver and kidney, as well as transamination, also decrease the AA pool of the body. Under normal conditions, the AA level in the plasma changes during the day (diurnal variation). In the urine, AA are present: glycine>alanine>serine>glutamine>histidine. During pregnancy it changes to histidine>phenylalanine>lysine>tyrosine. Cells have more AA than the plasma, and the plasma level of AA is greater than that of the CSF. In the liver, AA and -ketoglutarate contribute to the synthesis of liver and plasma proteins, purines, pyrimidines, porphyrines, hormones, etc. Also, they contribute to gluconeogenesis and the formation of urea. In the kidney, AA are filtered and completely reabsorbed by active transport systems (one for neutral AA, one for basic AA (lysine, arginine and histidine), one for proline, hydroxyproline and glycine, and one for dicarboxylic AA (aspartate and glutamate) ). Homocysteine and cystathionine are not efficiently reabsorbed. Aminoaciduria: high blood levels of AA that result in significant renal excretion. This can be primary (genetic) or secondary (liver/kidney disorders). There are 3 types: 1. Overflow aminoaciduria due to plasma AA level that exceeds tubular capacity. In this case the plasma AA level is high. 2. Renal aminoaciduria due to a defect in the renal transport. In this case the plasma level of AA is normal. 3. No-threshold aminoaciduria due to excessive level of AA in the plasma, but with renal excretion, plasma AA level remains normal. Effects of enzyme defects: (a) Product D is synthesized from A by a series of reactions catalyzed by enzymes a, b and c. Enzyme c' catalyzes the formation of a small amount of product E in a minor pathway. (b) In the absence of the enzyme c, no D is synthesized. (c) If the conversion of C to D is blocked, the concentration of the intermediate C, and the possibly other precursors, may increase. (d) Increased formation of E may occur if the concentration of C increases and conversion of C to D is blocked.

There are 6 disorders that should be mentioned (there are more than 6 inherited AA disorders, but these are the ones addressed in the course): 1. Hyperphenylalaninemia (and phenylketonuria) 2. Tyrosinemia 3. Alkaptonuria 4. Cystinuria 5. Homocystinuria 6. Urea cycle disorders

Avi Sayag

Clinical Biochemistry

1. Hyperphenylalaninemias (HPA) HPA results from impaired conversion of Phe to tyrosine.

The most common and clinically important is phenylketonuria (PKU). PKU is an autosomal recessive disorder. There are several HPAs that are not PKU and are called non-PKU HPAs. HPA is defined as a plasma phenylalanine concentration >120M. PKU is characterized by plasma phenylalanine >1000M and non-PKU hyperphenylalaninemias have plasma phenylalanine amounts that are <1000M. PKU is caused by mutation in the phenylalanine hydroxylase gene (PAH). The HPAs are disorders of phenylalanine hydroxylation. Because the reaction catalyzed by PAH involves tetrahydrobiopterin (BH4) as a co-factor, the HPAs can result from defects in any of the several genes required for synthesis and recycling of BH4. Removal of excess phenylalanine normally proceeds via the tyrosine biosynthesis reaction and then via tyrosine catabolism. The first reaction in this process is the PAH catalyzed hydroxylation of phenylalanine. There are 5 types of HPA, of which 3 are PKU (types I and II are worth mentioning: type I: a defect in Phe hydroxylase, and type II: a defect in dihydropteridine reductase). The accumulation of Phe inhibits the transport of other AA required for protein or neurotransmitter synthesis, reduces synthesis and increases degradation of myelin, and leads to inadequate formation of norepinephrine and serotonin. Phe is a competitive inhibitor of tyrosinase, a key enzyme in the pathway of melanin synthesis, and accounts for the hyperpigmentation of hair and skin. Untreated children with classic PKU are normal at birth, but fail to attain early development milestones, develop microcephaly, and demonstrate progressive impairment of cerebral function. Hyperactivity, seizures and mental retardation are major clinical problems later in life. To prevent mental retardation, diagnosis and initiation of dietary treatment of classic PKU must occur before the child is 3 weeks of age. Lab tests: Screening: Guthrie test (on the 6th-10th day of life) Specific diagnosis: determination of plasma Phe and Tyr by HPLC (see following practical topic 25) Monitoring: determination of plasma Phe by HPLC Prenatal diagnosis: DNA analysis (see following practical topic 1) Treatment consists of diet low in Phe and supplemented with Tyr. (Gene therapy is being developed). Guthrie test: A drop of blood is usually obtained by pricking the heel of a newborn infant on the 6th or 7th day of life. A small disk of the filter paper is punched out and placed on an agar gel plate containing Bacillus subtilis and B-2-thienylalanine. Each gel holds 60-80 disks. The agar gel is able to support bacterial growth but the B-2-thienylalanine inhibits bacterial growth. However, in the presence of extra phenylalanine leached from the impregnated filter paper disk, the inhibition is overcome and the bacteria grow. Within a day the bacterial growth surrounding the paper disk is visible to the eye. The amount of growth, measured as the diameter of the colony, is roughly proportional to the amount of phenylalanine in the serum.

Avi Sayag

Clinical Biochemistry

2. Tyrosinemia There are three types of tyrosinemia, each with distinctive symptoms and caused by the deficiency of a different enzyme. Type I tyrosinemia is the most severe form of this disorder and is caused by a shortage of the enzyme fumarylacetoacetate hydrolase. Fumarylacetoacetate hydrolase is the last in a series of five enzymes needed to break down tyrosine. Symptoms of type I tyrosinemia usually appear in the first few months of life and include failure to gain weight and grow at the expected rate (failure to thrive), diarrhea, vomiting, jaundice, cabbagelike odor, and increased tendency to bleed (particularly nosebleeds). Type I tyrosinemia can lead to liver and kidney failure, problems affecting the nervous system, and an increased risk of liver cancer. Worldwide, type I tyrosinemia affects about 1 person in 100,000. Type II tyrosinemia is caused by a deficiency of the enzyme tyrosine aminotransferase Tyrosine aminotransferase is the first in a series of five enzymes that converts tyrosine to smaller molecules, which are excreted by the kidneys or used in reactions that produce energy. This form of the disorder can affect the eyes, skin, and mental development. Symptoms often begin in early childhood and include excessive tearing, photophobia, eye pain and redness, and painful skin lesions on the palms and soles. About half of individuals with type II tyrosinemia are also mentally retarded. Type II tyrosinemia occurs in fewer than 1 in 250,000 individuals. Type III tyrosinemia is a rare disorder caused by a deficiency of the enzyme 4hydroxyphenylpyruvate dioxygenase. This enzyme is abundant in the liver, and smaller amounts are found in the kidneys. It is one of a series of enzymes needed to break down tyrosine. Specifically, 4-hydroxyphenylpyruvate dioxygenase converts a tyrosine byproduct called 4-hydroxyphenylpyruvate to homogentisic acid. Characteristic features of type III tyrosinemia include mild mental retardation, seizures, and periodic loss of balance and coordination (intermittent ataxia). Type III tyrosinemia is very rare; only a few cases have been reported.

Alkaptonuria is an autosomal recessive condition that is due to a defect in the enzyme homogenistic acid oxidase, which participates in the degradation of tyrosine. As a result, a toxic tyrosine byproduct called homogentisic acid (or alkapton) accumulates in the blood and is excreted in urine in large amounts. Excessive homogentisic acid causes damage to cartilage (HGA binds to collagen). The presentation also includes pigmentation in the ears and degenerative arthritis in middle age. 3. Cystinuria Cystinuria is an AR disorder caused by defective transporters in the apical brush border of proximal renal tubule and small intestinal cells. It is characterized by impaired reabsorption and excessive urinary excretion of the dibasic AA lysine, ornithine and cystine. Because cystine is poorly soluble, its excess excretion predisposes to the formation of kidney stones, which are responsible for the signs and symptoms of the disorder. Lab tests: Screening: cystine stones in the urine Specific diagnosis: AA detection in the urine by HPLC Treatment includes intake of large amount of water, alkalinizing the urine and administration of penicillamine (captopril) which chelates cystine.

Avi Sayag

Clinical Biochemistry

4. Homocystinurias Homocystinurias are 7 biochemically and clinically distinct disorders characterized by increased concentration of homocystine in blood and urine. The most common one is the classic homocystinuria, which results from reduced activity of cystathionine -synthase (the enzyme that condenses homocysteine with serine to form cystathionine). Symptoms include vascular complications during the first decade of life and are the major cause of morbidity and mortality (due to homocysteine). Other symptoms include defects in collagen metabolism due to cystine, and some patients develop marfanoid habitus and radiologic evidence of osteoporosis. Other rare causes include a deficiency in MTHFR and MTHF transferase. Lab tests: Screening: Guthrie test Specific diagnosis: determination of homocystine and methionine by HPLC. False negative results can occur if the diagnosis is made before 3 days of life, and false positive result can occur in case of premature enzymes, excessive protein intake, tyrosinemia and hepatitis. 5. Urea cycle defects Excessive ammonia generated from protein nitrogen is removed by the urea cycle, a process mediated by several enzymes and transporters. Complete absence of any of these enzymes usually causes severe hyperammonemia in newborns, while milder variants can be seen in adults. The accumulation of ammonia and glutamine leads to brain edema and direct neuronal toxicity. These disorders are AR, except for ornithine transcarbamylase deficiency, which is X-linked. Any of the 5 enzymes can be deficient: 1. CPS: Carbamyl-Phosphate synthetase 2. OTC: Ornithine transcarbamylase 3. ASS: Argininosuccinate synthetase 4. ASL: Argininosuccinase 5. ARG: Arginase

Symptoms include lethargy, vomiting, liver failure and coma. Lab tests: Screening: detection of plasma ammonia Specific diagnosis: detection of plasma and urine AA/enzyme detection.

Avi Sayag

Clinical Biochemistry

Lab diagnosis of CF 1. Sweat test: useful in the severe form of CF. Sweat is collected by pilocarpine iontophoresis, and the concentration of chloride is determined: a. Borderline values: 40-59 mM b. Elevated values: >60 mM 2. The presence of IgG against Pseudomonas aeruginosa 3. Immunoreactive trypsinogen (used in newborn screening programs) 4. Tests of exocrine pancreatic functions: a. Indirect: presence of enzymes in the stool and the fat levels; the level of carotene in the serum; b. Direct: analysis of the pancreatic secretion before and after stimulation with secretin and CCK (cholecystokinin) 5. DNA based CFTR mutation analysis: the tested mutations cover almost 60-95% of diagnosed CF cases. There are 4 classes of mutations: a. Class I: caused mainly by premature termination due to splicing defects, frameshifts due to small deletions or insertions; non-sense mutations. The CFTR can be reduced or completely absent. b. Class II: defective protein synthesis; misfolded protein; no transport to the membrane. c. Class III: defective regulation; mutant proteins reach the membrane, but the function of the channel is altered. d. Class IV: defective conduction. The channel is correctly processed and delivered to the membrane, but it generates reduced Cl- current. The rate of ion flow is reduced or the amount of time the channel is open is reduced. 6. ARMS, PCR-OLA, oligonucleotide hybridization assay a. PCR-ARMS Amplification Refractory Mutation System (also called: allele specific oligonucleotide hybridization): a method of PCR which distinguishes between alleles that differ in even a single nucleotide substitution. In this procedure the allele specific oligonucleotides (ASO) are immobilized on a nitrocellulose membrane and hybridized with a labeled PCR product spanning the variant nucleotide site. Discrimination between the 2 alleles is based on the fact that in a specific hybridization temperature the perfectly matched hybrid is more stable than the mismatched. After hybridization and washing, the detection is mainly colorimetric: if the PCR product is labeled with biotin, e.g. the detection is based on the strepatavidin-alkaline phosphatase conjugate enzyme reaction with a substrate. By using 2 ASOs it is possible to determine the wild type, heterozygous or homozygous. b. PCR-OLA: PCR followed by oligonucleotide ligation assay (OLA) is a molecular method that can be used for the detection of nucleotide sequence variants. PCR-OLA is based on the enzymatic ligation of two oligonucleotides that anneal next to each other onto the PCR-amplified target DNA. Even a single-nucleotide mismatch between the oligonucleotides and the template precludes the ligation. c. 2 CF mutations + an abnormal sweat test are diagnostic of CF! 7. Nasal potential difference: this test measures the potential difference between the apical and the basal surface of the nasal mucosa in the presence or absence of an epithelial sodium channel antagonist (amiloride, e.g.). A decrease in the potential difference is much higher in CF patients than in healthy ones. 8. Bronchoalveolar lavage (lavage=washing a hollow organ): in this procedure there is a high percentage of neutrophils in the lavage fluid among CF patients (normally, neutrophils comprise 3% of the lavage fluid, whereas in CF patients it reaches 50%). P. aeruginosa may also be present. 9. Prenatal diagnosis is performed when family anamnesis suggests CF: a. Immunoreactive trypsin in amniotic fluid b. DNA tests (chromosome 7q31.2)

Avi Sayag

Clinical Biochemistry

Molecular genetic methods for the investigation of inherited diseases (Practical topic 1)
There 4 steps in genetic diagnosis of inherited diseases: 1. Isolation of genomic DNA: by traditional method or spin column purification 2. Determination of the concentration and purity of the genomic DNA 3. Storage of DNA 4. Mutation detection: by Southern blot or PCR. Following these, electrophoresis, restriction digestion, hybridization probes, hybridization with allele specific oligonucleotides and multiplex ligation-dependent probe amplification (MPLA) can be used. 1. Isolation of genomic DNA The sample is taken from the buffy coat from peripheral blood anticoagulated by citrate or EDTA. In the traditional method: a. Cell harvesting by SDS1 b. Proteolysis with a special enzyme c. Multiple extractions by phenol, chloroform, isoamyl alcohol to dissolve hydrophobic contaminants and remove proteins. d. DNA is precipitated by 96-100% ethanol e. DNA is washed by 70% ethanol f. DNA is dissolved in TRIS buffer Spin column method: this method is based on the phenomenon that DNA/RNA binds selectively to silica in high salt concentration. Cell lysates are loaded onto a spin column containing silica membrane, and then several washing steps follow. This method is fast but expensive. 2. Determination of DNA purity DNA yield is determined by measuring the concentration of DNA by absorbance at 260nm. As the peak absorbance of DNA is at 260nm, that of proteins is at 280nm. Thus, purity is determined by calculating the ratio of absorbance at 260nm to that at 280nm. Pure DNA has a ratio of 1.7-1.9. Ratio less than 1.7 indicates protein contamination. 3. Storage DNA can be stored at 4C for months, at -20C for years, and at -70C for at least 10 years. 4. Mutation detection Southern blot: a. Restriction digestion: using enzymes that recognize specific base compositions that cleave the DNA at these sites. If the recognition sequence changes (for example due to a point mutation), the enzyme cannot cleave the DNA. b. Electrophoresis: separation according to size through an agarose gel or acrylamide gel. The detection is based on the intercalation of the fluorescent dye (ethidium bromide). c. Denaturation d. Transference to nylon membrane by capillary, vacuum or electric transfer. e. Immobilization by UV light f. Hybridization with a labeled probe g. Washing off the unbound probes h. Detection (depending on the labeling)

SDS Sodiun Dodecyl Sulfate. This compound disrupts non-covalent bonds in proteins, denatures them, and causes the molecule to lose its native shape. Also, anions of SDS bind to the main peptide chain at a ratio of 1 SDS for every 2 AA residues. This imparts a negative charge on the protein that is proportional to the mass of that protein. This new negative charge is significantly greater than the original charge of that protein. The electrostatic repulsion created by binding of SDS causes proteins to unfold into a rod-like shape, thereby eliminating differences in shape as factor for separation in the gel.

Avi Sayag

Clinical Biochemistry

PCR: amplification of a well-defined DNA fragment. The reaction consists of cycles: a. Denaturation at 95C to separate the strands of the DNA template b. Fast cooling to 50-65C to hybridize the primers and the polymerases (Taq) c. Incubation at 72C to synthesize the new strands 4 main types of PCR: 1. Standard PCR: to detect small variations in genetic material (e.g. Factor V Leiden mutation). 2. Multiplex PCR: involves parallel amplification of different pieces of DNA (e.g. Duchene muscular dystrophy). 3. Long PCR: combines 2 enzymes: Taq polymerase and a proof-reader, which make it possible to amplify larger DNA fragments (e.g. Friedrich ataxia). 4. Quantitative Real-time PCR: used to quantify the copy number of a specific DNA in the sample (e.g. translocation 9:22).

Principles of chromatography and its application in diagnostics (Practical topic 25)


Chromatography is used when the substance to be measured needs to be separated from other disturbing components. A small sample volume is injected in a continuous flow of gas or liquid (the mobile phase). The mixture of the sample and the mobile phase flows through the stationary phase, which usually consists of solid particles. On the stationary phase, the components of the mixture can be separated using different principles, because the different components spend different period of time in the stationary phase due to their different chemical properties. There are 3 main methods: 1. Adsorption chromatography: the particles of the stationary phase bind the components of the sample mixture. Different components remain bound for various amounts of time according to their chemical properties (those with low affinity to the stationary phase spend shorter time). 2. Ion exchange chromatography: the stationary phase consists of ion exchange particles, which bear surface-bound ionic groups with their counter-ions. Components of the sample replace the counter-ions from the surface of the particles. Different components bind to the fixed ions with different affinity. 3. Size exclusion chromatography: molecules are separated based on their physical size. Column chromatography may utilize 2 methods: gas chromatography and high performance liquid chromatography (HPLC). Gas chromatography: this method utilizes a high purity gas as eluent with constant pressure or flow rate. This technique is suitable for the detection of analytes that can be brought to gas phase and stay stable at the high analysis temperature of 100-350C. Gas chromatography can be used both for quantitative and qualitative analysis. HPLC: in the standard HPLC, the mobile phase is a stable liquid, and the stationary phase can be either a liquid or a solid substrate. The eluent circulates in a closed system and the constant flow is provided by a pump. The effluent coming from the column is transferred into a small cuvette of a detector. The detectors can be photometric, fluorimetric, electrochemical and refraction detectors. HPLC is applicable to measure materials which are soluble and stable in the mobile phase. The upgraded methods of HPLC are GC/MS and LC/MS (GS: gas chromatography; LC: liquid chromatography; MS: mass spectrometry). The different analytes reach the detector (the MS), separated, and analyzed. Clinical uses: PKU, cystinuria, homocystinuria, detection of HbA1c (on cation exchange column), hemoglobinopathies, vitamin D disorders, and detection of catecholamines and their metabolites (for example, catecholamines are elevated in neuroblastoma and melanoblastoma).

Avi Sayag

Clinical Biochemistry

Topic 5
Pathobiochemistry of inflammation
Inflammation can be acute or chronic: the acute one lasts from few minutes to few days, while the chronic one lasts from days to weeks. Acute inflammation is characterized by the presence of exudate and neutrophils, and the chronic one is characterized by the presence of lymphocytes, macrophages and tissue destruction. The chemical mediators released during the inflammatory process are responsible for the vascular response and for the cellular response. The vascular changes: in acute inflammation they manifest in a local increase in blood flow that results in redness and heat, and in increased vascular permeability that results in transudation. As the intravascular osmotic pressure decreases, the interstitial osmotic pressure increases and exudation replaces transudation. The accumulation of fluids in the extravascular tissue contributes to the edema. As the blood becomes more viscous, stasis ensues. The cellular changes: manifest in emigration of leukocytes from the microcirculation that accumulate in the inflamed site and are activated there. Leukocytes undergo 5 stages: 1. Margination and rolling: this is facilitated by the selectins expressed on the leukocytes as they are activated. L-selectins bind to E-selectins and P-selectins. 2. Adhesion and transmigration: adhesion is carried out by expression of integrins that bind to ICAM-1 expressed on the endothelium. 3. Chemotaxis and activation 4. Phagocytosis and degranulation 5. Leukocyte-induced tissue injury: injury is caused by lysosomal enzymes, by ROS, and arachidonic acid metabolites. Pain and loss of function result. Chemical mediators: 1. Chemical mediators of cellular origin: a. Vasoactive amines stored in secretion granules: i. Histamine: present in mast cells and basophils. It is released in response to physical injury, immune reactions, C3a and C5a, proteins released from leukocytes and IL-1 and IL-8 (cytokines). It causes arteriolar dilation, contraction of endothelium in venules and it increases vascular permeability. ii. Serotonin: present in platelets and is released by platelet-activating agents. Its effects are similar to those of histamine. b. Enzymes stored in secretion granules: i. Acid proteases: present in phagolysosomes only. ii. Neutral proteases: these include elastase, collagenase and cathepsins. They cause the destruction of the basement membrane and ECM. The control of these proteases is carried out by anti-proteases (e.g. 1antitrypsin). c. Newly synthesized mediators: i. Prostaglandins: synthesized in leukocytes, platelets and endothelium. PgI2 causes vasodilation and inhibits platelet activation; PgD2, PgE2 and PgF2 cause mainly vasodilation and potentiate edema; TXA2 causes vasoconstriction and activates platelets. ii. Leukotrients: synthesized in leukocytes. LTB4 is chemotactic; LTC4, LTD4 and LTE4 cause vasoconstriction, bronchospasm and increased permeability. iii. Platelet-activating factor: synthesized in leukocytes and endothelium. iv. Oxygen radicals: synthesized in neutrophils and macrophages by NADPH oxidase. These include superoxide, hydrogen peroxide, hydroxyl free radical and toxic NO derivatives. They cause endothelial injury and thrombosis, activation of proteases and inactivation of anti-proteases, and injury of other cells. Normally, they are counteracted by catalase, superoxide dismutase and glutathione.

Avi Sayag

Clinical Biochemistry

v. NO: synthesized in endothelium and macrophages. It is synthesized from arginine, in the presence of O2, NADPH and NOS. It causes vasodilation, inhibits platelet activation and killing of bacteria in activated macrophages. vi. Cytokines: synthesized in activated lymphocytes, macrophages and endothelium. Cytokines are polypeptides that have local effects on endothelium, leukocytes and fibroblasts, as well as systemic effects. 1. IL-1, IL-6 and TNF are synthesized in activated macrophages. They are produced in the acute phase reaction and are responsible for the fever, lethargy, neutrophilia, synthesis of acute-phase proteins, corticosteroids and cachexia. They also cause endothelial activation that results in leukocyte adhesion, synthesis of PgI, NO and thrombogenesis. 2. IFN- is synthesized in T-cells and activates neutrophils and macrophages, and induces NO synthesis. 3. Chemokines: these are synthesized in activated macrophages, endothelium and fibroblasts. They cause chemotaxis and neutrophil activation (e.g. IL-8). 2. Mediators derived from the plasma: a. Activation of the kinin system: FXII is activated and cleaves prekallikrein. The formed kallikrein cleaves off bradykinin from kininogen. Bradykinin increases vascular permeability, causes vasodilation, bronchospasm and mediates pain. As FXII is activated, tissue plasminogen activator (t-PA) is also released. b. Activation of coagulation: induction of tissue factor in macrophages and endothelium. c. Activation of complement system

Avi Sayag

Clinical Biochemistry

Topic 6
Pathobiochemistry of plasma proteins
When measuring the proteins in the plasma it is possible to measure the total protein concentration, the albumin and globulin fractions (semi-quantitative assessment) or specific proteins. What is the clinical significance in measuring the total serum protein? Total serum protein may decrease in the following settings: 1. Decreased protein synthesis (malnutrition, malabsorption, starvation, hepatic failure) 2. Hemodilution 3. Humoral immunodeficiency 4. Increased loss of proteins 5. Catabolic states Total serum protein may increase in the following settings: 1. Hemoconcentration (dehydration, stasis, posture) 2. Increased synthesis of plasma proteins (hypergammaglubolinemia, paraproteinemia) What are the most important serum proteins? Prealbumin Albumin 1 globulins 1-antitrypsin 1-acid glycoprotein 2 globulins 2-macroglobins Haptoglobins Ceruloplasmin globulins Transferrin Complement components IgG, IgA, IgM, IgD, IgE globulins 1. Prealbumin Prealbumin is a protein status indicator; it has a much shorter half-life and smaller serum pool than albumin. The half-life of prealbumin is approximately 2 days, making prealbumin a more timely and sensitive indicator of protein status. Prealbumin is a tryptophan-rich protein, and like albumin, it is synthesized in the hepatocytes of the liver. Prealbumin's main function is to serve as a binding and transport protein. The term prealbumin is actually a misnomer-the prefix pre implies that it is a precursor for albumin, which it is not. The more accurate name for prealbumin is transthyretin to indicate that it is a serum transport protein for thyroxin and retinol-binding protein. Evaluating prealbumin Like albumin, prealbumin is a negative acute-phase reactant. This limits its use as a screening tool for malnutrition because low levels could result from either inadequate nutrition or inflammatory stress. Rather than a diagnostic tool, prealbumin should be used as an indicator of nutritional improvement and as a measure of how well nutritional interventions are working. The very short half-life and small serum pool allows small changes in nutritional status to be identified in a short time frame. Elevated prealbumin levels may be seen in patients taking corticosteroids and in patients with Hodgkin disease. 2. Albumin Low levels of albumin can result from: a. Overhydration b. Decreased synthesis (malabsorption, malnutrition, starvation) c. Increased loss (nephrotic syndrome, Crohn's disease) The consequences of low levels of albumin may be: a. Edema b. Secondary aldosteronism c. Hypocalcemia (but ionized Ca+2 is not decreased) d. Increased risk of kernicterus in infants e. Altered pharmacokinetics of albumin-bound drugs

Avi Sayag

Clinical Biochemistry

High levels of albumin may result from: a. Dehydration b. Venous stasis during blood collection c. Overinfusion of albumin Remember: increased albumin synthesis does not cause hyperalbuminemia! So when do we decide to measure albumin levels in the plasma? 1. When we have to elucidate the cause of edema; 2. To follow up nutritional status (also measuring prealbumin); 3. To evaluate liver function (except for acute hepatitis where albumin is not informative); 4. To elucidate the cause of low calcium levels in the plasma; 5. When we wish to administer a drug that binds to albumin; 6. When we wish to diagnose analbuminemia. 3. 1-antitrypsin AAT is an 1-globulin, a protease inhibitor and an acute phase protein. When it is deficient, homozygotes are prone to develop emphysema and liver disease. The deficient level is detected by isoelectric focusing, and the genotype (homo- or heterozygote) is determined by PCR techniques. 4. Haptoglobin This 2-globulin binds free Hb and is an acute phase protein. Its levels decrease in: a. Intravascular hemolysis (absent) and extravascular hemolysis (low) b. Chronic liver disease c. Metastases of carcinoma d. Severe sepsis Its levels increase in: a. Acute phase reactions b. Nephrotic syndrome 5. 2-macroglobulin This is a protease inhibitor, whose synthesis is increased in hypoalbuminemia (the decreased albumin is replaced by the high-molecular-weight macroglobulin) 6. Ceruloplasmin Ceruloplasmin transports copper. Low levels are associated with Wilson's disease, and high levels are evident in acute phase reactions, pregnancy and use of oral contraceptives. Acute phase reactions This is a systemic, non-specific reaction to acute inflammation, infection, burns, tissue necrosis and tumor proliferation. It includes fever, leukocytosis, complex hormonal and metabolic changes (altered levels of acute phase proteins). Acute phase proteins can be positive or negative. 1. Positive acute phase reactants: Early, sensitive reactants: a. 1-antitrypsin b. 1-acid glycoprotein c. Haptoglobin d. Fibrinogen e. C-reactive protein (CRP) f. Procalcitonin Late, weak reactants: a. Ceruloplasmin b. C3, C4 2. Negative acute phase reactants: a. Prealbumin b. Albumin c. Transferrin

Avi Sayag

Clinical Biochemistry

Cytokines, such as IL-1, IL-6 and TNF, increase the mRNA of positive acute phase reactants, and decrease the mRNA of the negative ones. Synergethic hormones, such as glucocorticoids, enhance the reaction. What is the role of the acute phase proteins? 1. They inhibit the destructive proteases released from leukocytes during inflammation. 2. Scavanger proteins (CRP, haptoglobin) help to present cellular debris and breakdown products to the RES in order to process them and retain precious components. 3. Fibrinogen is required for wound healing. What are the classic parameters used to detect acute phase reactions? Fever, leukocytosis, ESR and serum protein electrophoresis. Of the acute phase reactants, CRP is the most sensitive and specific. C-reactive protein (CRP) Synthesized in the liver, CRP reacts with the C-polysaccharide of Streptococcus pneumonia. In the presence of calcium it bnds to phospholipids, polyanions and galactans. Through these molecules it binds to many bacteria, fungi, protozoa and cellular debris. In protein electrophoresis, it migrates somewhere between the band to mid- region. Its function is to initiate opsonization, phagocytosis and cell lysis. Once it is complexed, it activates the classical complement pathway. CRP is elevated in 4 main scenarios: 1. Inflammation: CRP shows an early rise (4-6 hours), which is more intense than ESR. When the patient recovers, its levels decrease before those of ESR. Mostly, its levels decrease when the inflammatory process is suppressed by steroids or salicylates. Thus, it is an excellent tool to monitor the activity level of rheumatoid arthritis, rheumatic fever, vasculitides, etc. Of particular interest, CRP is significantly higher in Crohn's disease than in ulcerative colitis, and its levels correspond to relapse, remission and response to therapy. 2. Tissue injury and necrosis: in AMI, CRP starts rising within 24-48 hours and reaches its peak on the 3rd day. It returns to normal level within 1-2 weeks. If there is tissue damage, it manifests in permanent increase of CRP. Its levels are elevated in infarctions of other tissues other than the heart. Other examples of CRP rise are rejection of kidney and bone marrow transplants, burn injuries and following surgery. If its levels fail to decrease after surgery, it is highly indicative of complications. 3. Infections: its levels are the highest in bacterial infections, but it is useful in the diagnosis of both bacterial and viral meningitis. It is useful in monitoring the activity of the disease and the efficacy of therapy. Last but not least, it is useful in diagnosing post-operative and intercurrent infections, mainly in active severe SLE and leukemia. 4. Malignancy: its levels are increased particularly in breast cancer, lung cancer and GI cancer. It can be thus considered a "general" tumor marker (see Topic 8). Procalcitonin Procalcitonin is a stable precursor of calcitonin coded by CALC-I gene. It is composed of 116 amino acids, and has a half-life of 25-30 hours. It consists of an N-terminal region, calcitonin and katacalcin domains. Procalcitonin is not synthesized in the thyroid, but in the lung, liver and intestines. What is the diagnostic value of procalcitonin? Procalcitonin levels are increased in sepsis and severe bacterial infections. It is practically absent (<0.1g/L) in the serum of healthy individuals. Its levels correlate well with the activity and extent of systemic bacterial infection. In viral infections, chronic and acute nonbacterial inflammations, however, its level remains unchanged or only slightly increases. Its increase precedes the clinical manifestations of severe generalized sepsis and has a prognostic value to it. Thus, it is useful in monitoring the treatment of such diseases.

Avi Sayag

Clinical Biochemistry

Topic 7
Biochemical effects of tumors
The biochemical effects of tumors (malignancies) are indirect: they may secrete biologically active substances even below the threshold of 109 cells (a tumor formed after 30 divisions, weighs 1 gram and spans 1 cm in diameter. This is when the tumor is clinically detectable). A paraneoplastic syndrome (PNS) is a disease or symptom that is the consequence of the presence of cancer in the body, but is not due to the local presence of cancer cells. These phenomena are mediated by humoral factors (by hormones or cytokines) excreted by tumor cells or by an immune response against the tumor. Recognition of a PNS alerts a new diagnosis of cancer. The frequency of PNS is 10-15%. PNS can be endocrine, neurological, hematological, dermatological, etc. 1. Endocrine syndromes: Cushing syndrome: caused by ectopic secretion of ACTH by lung cancers. SIADH: caused by ectopic secretion of ADH by lung cancers. Gynecomastia: caused by ectopic secretion of hCG by lung and liver cancers. Hypercalcemia: caused by ectopic secretion of PTH-rP by lung cancers. Hypocalcemia: caused by ectopic secretion of calcitonin by breast cancers. Hypoglycemia: caused by ectopic secretion of IGF-II by sarcomas. Carcinoid syndrome: caused by ectopic secretion of serotonin by carcinoid tumors. Carcinoid tumors are discrete, yellow, well-circumscribed tumors that can occur anywhere along the gastrointestinal tract and in the lung. They most commonly affect the appendix, ileum, and rectum (90%). Carcinoids are tumors of neuroendocrine nature, that originate in the cells of the neuroendocrine system (APUD) and are characterized by production of serotonin (5-hydroxytryptamine; 5-HT). Multiplex Endocrine Neoplasia (MEN): these are familial cancers (AD) characterized by the simultaneous appearance of tumors (benign or malignant) of 2 or more endocrine organs. MEN-I: parathyroid, pancreatic islets, anterior pituitary. MEN-IIa: thyroid (medullary cell carcinoma), adrenal medulla (pheochromocytoma), parathyroid. MENIIb: same as MEN-IIa but with various somatic abnormalities (marfanoid habitus, e.g.). 2. Hematological syndromes: Anemia: the anemia can be of two types hypoproliferative or autoimmune hemolytic anemia. The hypoproliferative anemia is a mild anemia (Hb: 80-120 g/L) due to impaired erythropoiesis (e.g. insufficient EPO production or impaired response of the BM to EPO). The autoimmune hemolytic anemia is due to Ab against RBCs. The Abs can be warm reacting (as in HL, NHL, CLL, MM, breast and lung cancer) or cold reacting (as in chronic granulocytic leukemia, carcinoid tumors and adrenocorticocarcinoma). Erythrocytosis: caused by ectopic production of EPO or by hypoxemia (that can be the result of a direct effect of the tumor, as it narrows the O2 supply highway to the kidney). Erythrocytosis is mainly seen in hypernephromas (30%), cerebellar hemangioblastoma (20%), hepatoma (1%) and Wilm's tumor (1%). Leukocytosis: caused by ectopic production of G-CSF. It is associated with gastric adenocarcinoma, lung cancer, pancreatic cancer, HL, and NHL. Symptoms appear late in the course of the malignant process. The WBC count exceeds 20x109/L without a left shift. Thrombocytosis: caused by thrombopoetin overproduction in 1/3 of cancer patients (HL and NHL). PLT count is mildly high (1000 G/L). Thrombocytopenia: caused by ITP-like or hypoplasia, and is associated with HL, NHL, CLL, lung cancer, breast cancer and testicular cancer. PLT count is less than 30 G/L. Hemostasis disorders: thromboembolism is the second cause of death among cancer patients with promyelocytic leukemia, myeloproliferative disease, primary brain tumor, lung cancer and ovarian cancer. Patients with acute leukemias suffer from hemorrhages due to consumption coagulopathies. DIC is seen among patients with prostate cancer, adenocarcinoma, and AML-M3.

Avi Sayag

Clinical Biochemistry

3. Neurological syndromes The paraneoplastic neurological syndromes result from autoimmune processes. Certain tumors produce large amounts of proteins expressed in the PNS or CNS. These molecules stimulate the humoral or the cellular immune system. Lambert-Eaton syndrome: Abs against Ca+2 channels in the presynaptic neuron. Thus, Ach is not released. It is most frequently associated with lung cancer. Cerebellar degeneration: Anti-Yo Abs. Encephalomyelitis: Anti-Hu Abs. Degeneration of peripheral nerves: Anti-Hu Abs. Retinopathy: CAR Abs. 4. Dermatological syndromes These PNS result from autoimmune processes or from mediators produced locally or systemically such as in carcinoid tumors.

Avi Sayag

Clinical Biochemistry

Topic 8
Tumor markers (TM) in the diagnosis of malignant disease
What is TM? Tumor markers are molecules produced by the tumor or by the host (reacting to the presence of the tumor). These markers can be enzymes (and isoenzymes), hormones, oncofetal antigens, carbohydrate epitopes, cytokeratines, receptors and products of oncogenes. TM can be measured qualitatively and quantitatively by chemical methods, immunochemical methods or molecular biological methods in the serum or in other body fluids (urine, saliva, CSF, nipple discharge). What is the ideal TM? The ideal TM: Is produced specifically by the malignant tissue or by the premalignant tissue; Is produced in all patients with a specific tumor type; Is produced in an organ-specific manner; Is measurable in an easily accessible body fluid; Its concentration in body fluid correlates with tumor volume or with the biological behavior of the tumor; Has a short half-life for the post-therapy follow-up; Informs of the progression and regression by its rise-and-falls; Is cheap, simple, standardized with an available reproducible assays. What are the "buts"..? But the ideal TM test does not exist yet (if you read this after 2009, double-check if it's still so.) But TM lack specificity: elevated concentrations can be seen in benign diseases too. But TM lack sensitivity for early malignancy. But TM are rarely elevated in all patients of a particular type of cancer. But no marker so far has an absolute organ specificity. What are the types of TM?

What are TM used for? Screening: serum TM are not ideal for population screening because they have low sensitivity and specificity plus a low positive predictive value of these tests. Aiding in diagnosis: is the tumor benign or malignant? What is the histological type of the tumor (e.g. is it a germ-cell tumor, a seminoma or a non-seminoma tumor?) Is it a metastatic cancer with unknown primary site? Assessing the prognosis: beside the classical prognostic factors (tumor size, grade, stage), TM can serve as independent prognostic factors. Predicting the response to therapy: in breast cancer, for example, the presence or absence of hormone receptors may predict the response to therapy. Monitoring patients with diagnosed disease: TM aid in early detection of recurrent malignancy as well as in monitoring the treatment given to the patient. Those markers that were positive at the time of diagnosis are applied in the monitoring process.

Avi Sayag

Clinical Biochemistry

What are the tumor markers used for screening? There are 6 TM that are worth mentioning: 1. VMA (vanillylmandelic acid) and HVA (homovanillic acid) for neuroblastoma (NB): NB is the most common solid tumor among children younger than 5 years of age. The tumor originates from the primitive nervous tissue, and it synthesizes adrenaline, noradrenaline and dopamine. VMA and HVA are breakdown products of these catecholamines and can be measured in the urine. High levels of these markers can be found in 85-90% of NB patients at 6 months of age. Screening, however, does not reduce mortality. 2. PSA for prostate cancer: prostate cancer is the 4th most common malignancy in males. PSA and a digital rectal examination are used for screening males older than 50 years of age, and for screening high-risk groups at 40-45 years of age. 3. CA125 for ovarian cancer (OC): OC is the most frequent cancer in females in the western world. CA125 combined with recto-vaginal pelvic examination and transvaginal ultrasound (TVUS)/transabdominal US (TAUS) and Doppler test are used for screening. Screening, however, is recommended for high-risk population. 4. AFP for hepatocellular cancer: HC cancer is the 5th most common cancer in males and the 8th most common in females. It is associated with HBV and HCV infections, hemachromatosis and liver cirrhosis. AFP combined with TAUS are used for screening. Screening is recommended for high-risk groups (those with chronic active hepatitis, cirrhosis, etc.). When performed, AFP is screened for every 3-4 months, and TAUS is performed every 4-6 months. 5. hCG for choriocarcinoma: this tumor is composed of villous trophoblasts, and may follow normal pregnancy, non-molar abortion, ectopic pregnancy or hydatidiform mole. hCG is elevated in almost all patients with choriocarcinoma. It is very sensitive and is considered a nearly-ideal TM. Even though a good therapy is available, screening is recommended for high-risk groups only (e.g. hydatidiform mole). 6. Calcitonin for medullary thyroid cancer: this is a rare malignancy that can be hereditary or acquired, solitary or in combination with other tumors (MEN2A, MEN2B). The parafollicular cells are malignant. As they produce calcitonin in basal values or in stimulated values (following the administration of pentagastrin or calcium gluconate), screening is recommended for high-risk groups once a year. What are the TM for germ-cell tumors? Seminoma: AFP Non-seminoma: hCG Mixed germ-cell tumor: LDH, P-ALP What are the gynecological tumor markers? Ovarian cancer: CA125 (to a lesser degree: CA15-3, CA72-4, TPA) Cervical cancer: SCC Endometrial cancer: CA125 What are the lung cancer tumor markers? Adenocarcinoma: cyfra21-1, CEA (to a lesser extent: CA125) Squamous cell carcinoma: cyfra21-1, SCC Small cell carcinoma: NSE, cyfra 21-1 Large cell carcinoma: cyfra21-1, CEA (to a lesser extent: CA125). What are the breast cancer TM? CA15-3, CEA (to a lesser extent: TPA, Cyfra21-1) What are the prostate cancer TM? PSA, fPSA (to a lesser extent: P-AcP (prostatic acid phosphatase)) What are the GI cancer TM? Colorectal cancer: CEA Gastric cancer: CEA<CA19-9<CA72-4 Esophageal cancer: SCC, TPA, cyfra21-1 (to a lesser extent: CA19-9) Pancreatic cancer: CA19-9 Hepatocellular cancer: AFP

Avi Sayag

Clinical Biochemistry

What do CA, TPA, SCC, Cyfra, hCG, AFP, CEA, PSA abbreviate? CA carbohydrate antigen; TPA tissue polypeptide antigen SCC squamous cell carcinoma antigen Cyfra cytokeratine 19 fragment hCG human chorionic gonadotropin AFP alpha-feto-protein CEA carcino-embryonic antigen PSA prostatic specific antigen What is the frequency of TM determination? Before first therapy 2-10 days after therapy (according to the half-life of the TM) In intervals of 3 months during the 1st and 2nd year In intervals of 6 months in the 3rd, 4th and 5th year after the first therapy Before any change of therapy If a relapse or metastasis are suspected If restaging is to be carried out 2-4 weeks after the first occurrence of a significant increase of the TM How do we evaluate the results? During monitoring of therapy, a decrease of at least 50% in the concentration of the TM indicate partial remission. TM cannot be used to determine complete remission. If an increase of the TM concentration is detected, repeat the measurement in 2-4 weeks (twice; all-in-all: 3 measurements). An increase of at least 25% in the TM concentration is significant enough to determine progression of the tumor. What are the factors that alter the serum concentration of TM? The production of the TM: how much TM is produced. The release of the TM: the rate of release, and whether it is released or not. The mass of the tumor: the larger the tumor is, the more TM might be produced. Blood supply of the tumor: the presence of the TM in the serum depends on that. Diurnal variation Position of the body during blood drawing Iatrogenic influences Catabolism of the tumor marker Life-style (habits): smoking, alcohol consumption.

Avi Sayag

Clinical Biochemistry

Topic 9
Iron metabolism, hemochromatosis, iron deficiency anemia (note: this topic includes practical topics 2, 3 and 6)
The daily iron intake is about 15 mg, but less than 10% is absorbed (0.6-1.5 mg). The total body content of iron is ~4 g: 3g in hemoglobin, ~0.5g in myoglobin and enzymes and ~4mg in the plasma. The body stores up to 1.5g of iron. The average male loses ~1mg per day and a female loses ~2.5mg per day. The regulation of iron absorption is complex. There are three major influences: The state of the body's iron stores (absorption is increased when they are depleted and decreased when they are replete) Erythropoiesis (absorption is increased when erythropoiesis is increased, irrespective of the state of the iron stores) Recent iron intake (a dietary iron bolus decreases iron absorption for several days). The main site of iron absorption is the proximal small bowel. Iron is more readily absorbed in the Fe2+ form but dietary iron is mainly in the Fe3+ form. Gastric secretions are important in iron absorption in that they liberate iron from food (although heme can be absorbed intact) and promote the conversion of Fe3+ to Fe2+. Ascorbic acid and other reducing substances facilitate iron absorption while phytic acid (in cereals), phosphates and oxalates form insoluble complexes with iron and decrease its absorption. Iron is continuously recycled in an almost closed system there is no significant excretion. Body stores are determined by the control of intestinal absorption. Free iron is toxic, and thus, iron is protein-bound (i.e. it is transported bound to transferrin and stored in ferritin and hemosiderin).

Once absorbed into the intestinal mucosal cells, iron is either transported directly into the bloodstream, or else combines with apoferritin, a complex iron-binding protein, to form ferritin. This iron is lost into the lumen of the gut when mucosal cells are shed. In iron deficiency, the apoferritin content of mucosal cells decreases and a greater proportion of absorbed iron reaches the bloodstream. Transport: In the blood, iron is transported mainly bound to transferrin, each molecule of which binds two Fe3+ ions. Transferrin concentration is increased in pregnancy, among women who take oral contraceptive, estrogen, and in iron deficiency. Its concentration is decreased in chronic inflammation, malignancies, nephrotic syndrome and iron overload. The transferrin saturation is normally 15-45%. In iron deficiency, saturation is < 15% and in iron overload it is > 60%. Iron can be taken from the plasma by cells other than erythrocytes,

Avi Sayag

Clinical Biochemistry

as these cells express transferrin receptors. Transferrin receptor (TfR) is present on the surface of all cell types (80% on erythroid precursors). It is a transmembrane protein that consists of 2 identical subunits. Its extracellular part is cleaved off by a serine protease and the receptor is liberated (mainly from late erythroid precursors, i.e. normoblasts and reticulocytes). Once in the circulation, it is complexed with transferrin. Iron uptake is regulated by the number of transferrin receptors. Storage: ferritin and hemosiderin store iron. Ferritin is found in all cell types. It has a Fe+3 hydroxide-phosphate core covered by 24 subunits of protein shell (apoferritin). On the surface of this "ball" there are channels through which Fe+3 enters and leaves. There are 2 types of subunits: a heavy chain, which has a ferroxidase activity, and a light chain that is responsible for the building up of the iron core. Each subunit (of the 24 subunits) is called an apoferritin. Each apoferritin binds 3000-4500 iron ions. Small amount circulates in the serum and is in equilibrium with that in stores. Serum ferritin reflects the size of iron stores. Ferritin is an acute phase reactant. It is increased in iron overload, liver disease, cancer, and in acute and chronic inflammation. It is decreased in iron deficiency.

Iron regulation of gene expression: Cells in rest have very low transferrin receptors. When the iron level is low, cells need more transferrin receptors in order to get enough iron from the circulation. However. They do not require much storing proteins (ferritin). The opposite is true when the iron concentration in the blood is high. Both transferrin receptors and ferritin are regulated at the mRNA level, according to the cytosolic iron level. Both mRNAs have a iron-response element sequence, to which binding proteins (IRE-BP) can bind. When the binding proteins bind to ferritin mRNA, translation is blocked, but when they bind to transferrin receptor IRE, translation takes place.

Avi Sayag

Clinical Biochemistry

Hepcidin: this is a small peptide hormone synthesized in the liver. It has a regulatory role in iron homeostasis and an antimicrobial effect. In case of inflammation (IL-6), its levels increase. It then binds to ferroportin and leads to its degradation. Thus, iron is trapped in the enterocyte. Some 48 hours following ferroportin degradation, the enterocyte is desquamated and the iron content spills. Iron is trapped in macrophages as well leading to anemia of chronic disease. Iron deficiency anemia: This anemia is microcytic (MCV<80fL) and hypochromic (MCH < 27 pg). Iron deficiency can be caused due to increased demand for iron or due to a decreased supply. Increased demand for iron can be due to: o Chronic blood loss (GI, urogenital, respiratory system) o Growth o Pregnancy and lactation Decreased supply can be due to: o Diet with low amount of heme iron o Impaired iron absorption (intestinal disease, inhibitors of absorption such as phosphates, phytates that inhibit reduction of Fe+3, HCl deficiency and accelerated passage). Development of iron deficiency anemia:

Lab diagnosis of iron deficiency anemia: 1. hematological tests: Hb, hematocrit, MCV, MCH, MCHC, RDW, blood smear and bone marrow smear. 2. Biochemical tests: serum iron, serum transferrin, transferrin saturation, serum ferritin, soluble transferrin receptor.

Procedure of blood drawing, vacutainer tubes (practical topic 3)


1. Vacutainer tubes: Anticoagulant None EDTA (ethylene diamine tetra-acetic acid) Sodium citrate Common tests Color of stopper Chemistry Red Hematology, molecular Lavender genetic tests Hemostasis, molecular Blue genetic tests Sodium/lithium-heparin Flow cytometry, Green chromosome analysis Sodium fluoride Glucose determination Gray from stored samples ACD (acid citrate dextrose) Blood culture Yellow 2. Needle size: the gauge is a measurement of the needle diameter the larger the number is, the smaller the diameter is (gauge 21 or 23 are usually used). 3. Needle type: multiple draw needle, butterfly needle, blood lancet. 4. Tube holder: barrel (a device for a safe and secure blood drawing) 5. Alcohol swabs: isopropyl alcohol (does not induce hemolysis) 6. Tourniquets 7. Gloves 8. Gauze

Avi Sayag

Clinical Biochemistry

9. Adhesive bandages/tapes 10. Needle disposal container Procedure: 1. Wear gloves 2. Positioning of the patient 3. Apply tourniquet in the upper arm 4. Locate the vein (middle cubital for example) 5. Assemble the equipment 6. Clean the site 7. Perform venipuncture 8. Release tourniquet 9. Remove the needle 10. Apply the gauze 11. Dispose the needle 12. Label the specimen Microcapillary blood collection: used in case of newborns and infants. The site chosen is usually the finger or the heel in case of an infant (for PKU testing for example). The skin can be preheated if necessary. After cleaning the site, puncture the skin off-center and perpendicular to the lines of the fingerprint. Then, collect the blood by first wiping off the first drop and proceed as above-mentioned.

Determination of hemoglobin and hematocrit (practical topic 2)


The Hb-cyanide method: Advantages: 1. Beside Hb, methemoglobin and carboxyhemoglobin (HbCO) can be measured. 2. Stable cyanmethemoglobin standards are available for calibration. Disadvantages: 1. It does not measure sulphhemoglobin, which is rarely present. 2. The total conversion of HbCO is slow it takes 30 minutes. If during determination the extinction of the solution is measured exactly at the 3rd minute in the presence of 20% HbCO (which may be found in heavy smokers) the Hb concentration is overestimated by 6% (HbCO absorbs more light at 540nm than does hemoglobin cyanide). 3. Considering that the reagents contain cyanide, special fluid waste disposal is required. Procedure: 1. Collection of whole blood anticoagulated with EDTA 2. Lysis of RBCs by SLS (Sodium-Lauryl-Sulphate) 3. Hb ----------> hemiglobin -------------> HiCN. The reagent contains K3Fe(CN)6 that carries out the first reaction, and KCN (CN-) that carries out the 2nd reaction. 4. A non-ionic detergent (Nonidet P40) is added to assure total and rapid hemolysis and for the prevention of precipitation of proteins and lipoproteins. 5. The extinction of the color product is measured at 540 nm. The transformation of Hb, HbO2 and Hi occurs in 5 minutes. If there is a high WBC count, PLT count or high concentration of lipoproteins, turbidity is visualized and the specimen should be centrifuged before measurement. Reference interval: Male: 140-180 g/L Female: 120-160 g/L Hematocrit: Htc can be measured using centrifugation, measuring impedance and by calculation. Centrifugation: 1. Collection of blood anticoagulated with EDTA or heparin. The blood is mixed to homogenize RBCs

Avi Sayag

Clinical Biochemistry

2. Fill the blood into the capillary and close with wax. Make sure the closing is perfect and the surface of the wax is horizontal. 3. Place the capillary in the centrifuge, with the closed end facing outwards. 4. Centrifuge for 5 minutes with RPM corresponding to 12000g. 5. Transfer the capillary to the reading device. The bottom of the sample should fall exactly on the zero level. The capillary holder should be moved horizontally till the top of the plasma reaches the line corresponding to the 1.00 value. 6. The line marking the top of the RBCs cylinder gives the Htc value. 7. Multiple the value by 0.97: sedimented RBCs enclose about 3% plasma (trapped plasma). This value can be higher when the form of the RBC is altered (like in sickle cell anemia, when the trapped plasma can reach 20%). Reference interval: Male: 0.38-0.52 Female: 0.37-0.46 Errors may occur: 1. If the capillary is filled with incorrectly suspended blood. 2. If the capillary is not well closed some RBC may leak. 3. If the closing wax at the bottom of the capillary is not horizontal (the setting of the zero line is not correct). 4. If the WBC count is high, and not taken into consideration, it may lead to falsely high Htc value. In this case, the Htc should be read at the top of RBC layer. 5. Unequal diameter of the capillary, inadequate RPM, short centrifugation time Calculation: Automated hematology analyzers obtain Htc value as a calculated parameter derived from RBC count and MCV: Htc =

RBC MCV . (RBC in T/L (1012) and MCV in fL (10-15) ). 1000

Avi Sayag

Clinical Biochemistry

Topic 10
Laboratory diagnostics of hemoglobinopathies
Globin chains are synthesized in the liver of the fetus and in the bone marrow of adults. Adults Hb (HbA) can be of 2 type: HbA1 composed of 22 chains (96-98%) HbA2 composed of 22 (2%) Fetal Hb (HbF) is composed of 22 (0.5-0.8%) Gene defects in the Hb molecule are the most common genetic disorders in the world. There are about 400 Hb variants registered. Abnormal Hb can be detected in electrophoresis. The most common variants are: HbS, HbC, HbS-C, HbE. Sickle cell anemia (HbS) The cause of the disease is a point mutation in the chain of the globin: the -globin gene is found on the short arm of chromosome 11. The association of two -globin subunits with two mutant -globin subunits forms hemoglobin S (HbS). Valine replaces glutamate residue at position 6 of the chain located at the surface of the Hb, which is exposed to water. Glu is a polar amino acid while Val is non-polar. Thus, the replacement causes the formation of a sticky patch on the surface of the chains. On the surface of the deoxyHb there is another sticky patch that causes polymerization of Hb into a fibrous structure.

The loss of red blood cell elasticity is central to the pathophysiology of sickle-cell disease. In sickle-cell disease, low oxygen tension promotes red blood cell sickling and repeated episodes of sickling damage the cell membrane and decrease the cell's elasticity. These cells fail to return to normal shape when normal oxygen tension is restored. Consequently, these rigid blood cells are unable to deform as they pass through narrow capillaries, leading to vessel occlusion and ischemia. Thus, the hemolytic anemia dominates the clinical features with characteristic crises: 1. Infarcts in bones, lungs, spleen (vaso-occlusive crisis) 2. Sequestration in visceral organs 3. Aplastic crises (infections; decreased Hb and reticulocytes) 4. Hemolytic crises (decreased Hb but increased reticulocytes) 25% of Africans are heterozygote for the disease and are therefore resistant to malaria, since the sickle red blood cells are not conducive to the parasites. In areas where malaria is common there is a survival value in carrying the sickle-cell genes. Apart from middle Africa, sickle cell anemia is also common in south Europe and in some parts of the Saudi Arabia. Diagnosis 1. In HbSS, Hb levels are in the range of 68 g/dL with a high reticulocyte count. 2. A blood film may show features of hyposplenism (target cells and Howell-Jolly bodies). 3. Sickling of the red blood cells, on a blood film, can be induced by the addition of sodium metabisulfite. The presence of sickle Hb can also be demonstrated with the "sickle solubility test". A mixture of Hb S in a reducing solution (such as sodium dithionite) gives a turbid appearance, whereas normal Hb gives a clear solution. 4. Abnormal Hb forms can be detected on electrophoresis

Avi Sayag

Clinical Biochemistry

5. The diagnosis can be confirmed with high-performance liquid chromatography (HPLC). 6. An acute sickle-cell crisis is often precipitated by infection. Therefore, a urinalysis to detect an occult urinary tract infection, and chest X-ray to look for occult pneumonia should be routinely performed. It should be noted that the sickle cell trait is a benign condition with no anemia. The RBCs appear to be normal, and crisis can be caused by extreme stress (anoxia, infection).

Note the presence of the A band as opposed to the previous one where no A band is present (this is because in homozygotes only HbS is present, while in heterozygotes 40% is HbS and the remainder is HbA). Hb C disease Hb C is an abnormal Hb with substitution of a lysine residue for a glutamic acid residue at the 6th position of the -globin chain. This mutated form reduces the normal plasticity of host erythrocytes causing a hemoglobinopathy. In those who are heterozygous for the mutation, about 2844% of total hemoglobin (Hb) is HbC, and no anemia develops. In homozygotes, nearly all Hb is in the HbC form, resulting in mild hemolytic anemia. Target cells (codocytes), microspherocytes and HbC crystals are found in a blood smear from a homozygous patient. The reticulocyte count is high, as well as serum bilirubin. Hb E disease Lysine substitutes glutamate on position 26 in the chain. Most common in south east Asia. Homozygotes present with mild hypochromic, microcytic anemia with target cells. Thalassemias Thalassemia is an inherited autosomal recessive blood disease. In thalassemia, the genetic defect results in reduced rate of synthesis of one of the globin chains that make up hemoglobin. Reduced synthesis of one of the globin chains causes the formation of abnormal Hb molecules, and this in turn causes the anemia which is the characteristic presenting symptom of the thalassemias. The cause of the disease might be a missing gene, improper processing of mRNA, premature termination of protein synthesis, frameshift mutation, etc. The thalassemias are classified according to which chain of the hemoglobin molecule is affected. In thalassemias, production of the globin chain is affected, while in thalassemia production of the globin chain is affected. globin chains are encoded by a single gene on chromosome 11; globin chains are encoded by two closely linked genes on chromosome 16. Thus in a normal person with two copies of each chromosome, there are two loci encoding the chain, and four loci encoding the chain. -thalassemia: There are four genetic loci for globin, two of which are maternal in origin and two of which are paternal in origin. The severity of the thalassemias is correlated with the number of affected globin loci: the greater the number of affected loci, the more severe the manifestations of the disease will be. If 1 copy is defective- silent -thalassemia carrier; no significant signs or symptoms (except maybe low MCV and low MCH)

Avi Sayag

Clinical Biochemistry

If 2 copies are defective thalassemia triat. Two loci permit nearly normal erythropoiesis, but there is a mild microcytic, hypochromic anemia. If 3 copies are defective mild-to-moderate hemolytic anemia (HbH disease), resulting in poor O2 delivery (too high affinity to O2). If 4 copies are defective - the fetus cannot live once outside the uterus and may not survive gestation: most such infants are dead at birth with hydrops fetalis, and those who are born alive die shortly after birth. They are edematous and have little circulating hemoglobin, and the hemoglobin that is present is all tetrameric chains. -thalassemia: as mentioned, there are 2 copies of the -chain gene: If 1 copy is defective minor -thalassemia mostly asymptomatic (perhaps mild microcytic, hypochromic anemia may develop). HbF is increased and HbA2 > 3.5%; target cells are present as well as ovalocytes, poikilocytosis and basophil stippling. If 2 copies are defective major -thalassemia appears after birth. Severe microcytic anemia develops (Cooley's anemia), and the patient depends on constant blood transfusion. This offered treatment is life-saving, but results in iron overload. The patient may die at 20-25 years of age. The ultimate cure is bone marrow transplantation. The Hb levels are 30-40 g/L, MCV 51-61 fL, reticulocytes 1-8%, and both HbF and HbA2 are elevated. The bone marrow shows erythroid hyperplasia (E:M is 20:1) seFe is elevated, Tfsat is > 80%. There is also thalassemia intermedia in which the morphology is the same as for the major form, but the HbF is 2-100%, the HbA2 does not exceed 7%. Diagnosis

Sideroblastic anemia Sideroblastic anemia is caused by abnormal production of red blood cells usually as part of myelodysplastic syndrome, which can evolve into hematological malignancies (especially acute myelogenous leukemia). The body has iron available but cannot incorporate it into hemoglobin. Sideroblasts are seen, which are nucleated erythrocytes with granules of iron in their cytoplasm The problem lies in a failure to completely form heme molecules, whose biosynthesis takes place partly in the mitochondrion. This leads to deposits of iron in the mitochondria that form a ring around the nucleus of the developing red blood cell. It can be inherited (X-linked) or acquired. If acquired, it can be primary, as part of a myelodysplastic syndrome, or secondary due to: Toxins: lead or zinc poisoning

Avi Sayag

Clinical Biochemistry

Drug-induced: ethanol, isoniazid, chloramphenicol, cycloserine Nutritional: pyridoxine or copper deficiency Genetic: ALA synthase deficiency Diagnosis Specific test: Prussian Blue stain of RBC in marrow. Shows ringed sideroblasts. Increased ferritin levels Decreased total iron-binding capacity Hematocrit of about 20-30% Serum Iron: High High transferrin saturation MCV is usually normal or slightly increased; although it may occasionally be low, leading to confusion with iron deficiency. With lead poisoning, there is coarse basophilic stippling of red blood cells on peripheral blood smear.

Summary and presentation of the topic: General features of Hb types, %, structure Disorders: HbS, HbC, HbC-S, HbE, thalassemia, sideroblastic anemia Diagnosis of HbS: Hb, ret, blood film, Na-metabisulfite, sickle solubility test, electrophoresis, HPLC (sickle cell trait) Diagnosis of thalassemia: history, clinical signs, blood (Hb, MCV, MCH, RDW, ret), electrophoresis, HPLC, -chain sequencing Diagnosis of sideroblastic anemia: Prussian blue stain, ferritin, iron binding capacity, HTC, Fe, Tfsat, MCV, lead poisoning: basophilic stippling on peripheral smear.

Avi Sayag

Clinical Biochemistry

Topic 11
Laboratory diagnostics of hemolytic anemias
An increased rate of red cell destruction; A compensatory increase in erythropoesis that results in reticulocytosis; The retention by the body of the products of red cell destruction (including iron). Because the iron is conserved and recycled readily, red cell regeneration can keep pace with the hemolysis. Consequently, these anemias are almost invariably associated with a marked erythroid hyperplasia within the marrow and an increased reticulocyte count in peripheral blood. In severe hemolytic anemias, extramedullary hematopoiesis often develops in the spleen, liver, and lymph nodes. Destruction of red cells can occur within the vascular compartment (intravascular hemolysis) or within the cells of the mononuclear phagocyte (reticuloendothelial) system (extravascular hemolysis). Intravascular hemolysis can result from mechanical trauma (e.g., a defective heart valve) or biochemical or physical agents that damage the red cell membrane (e.g., fixation of complement, exposure to clostridial toxins, or heat). Regardless of the cause, hemolysis leads to hemoglobinemia, hemoglobinuria, and hemosiderinuria. The conversion of the heme pigment to bilirubin can result in unconjugated hyperbilirubinemia and jaundice. Haptoglobin, a circulating protein that binds and clears free hemoglobin, is often absent from the plasma. Extravascular hemolysis (more common) takes place largely within the phagocytic cells of the spleen and liver. The RES removes damaged red cells from the circulation. The red cells need to be highly deformable to travel in the splenic sinusoid. Therefore, any change in that feature leads to red cells being stuck in the spleen (and phagocytosed there). Extravascular hemolysis is not associated with hemoglobinemia and hemoglobinuria, but it often produces jaundice and, if long-standing, can lead to the formation of bilirubin-rich gallstones. Haptoglobin amounts are always decreased, because some hemoglobin invariably escapes into the plasma. In most forms of hemolytic anemia there is a reactive hyperplasia of the RES (splenomegaly). In chronic hemolytic anemias, changes in iron metabolism lead to increases in iron absorption from the gut. Because the pathways for the excretion of excess iron are limited, this often causes iron to accumulate, giving rise to systemic hemosiderosis. The intracorpuscular abnormalities can be either hereditary or acquired. The hereditary causes might be: Membrane defects: o In the setting of spherocytosis (spectrin deficiency), jaundice may follow, splenomegaly, gallstones and most probably anemia. Spherocytosis is AD. Microspherocytes and elevation of reticulocytes (5-20%) are evident in blood film. The mechanism underlying spherocytosis starts with spectrin deficiency. This leads to decreased protein density of the RBC's cytoskeleton and as a result parts of the erythrocyte's bilayer membrane are released as microvesicles. Thus, the surface area of the RBC is decreased (spherocytosis). This condition impairs the deformability of the red blood cell, and they are therefore entrapped in the spleen. Trapped cells cause splenic conditions leading to further loss of surface area, which then again lead to the release of RBCs' membrane as microvesicles. Other forms of hereditary spherocytiosis result from mutations that involve ankyrin, band 4.2 and band 3.

Avi Sayag
o

Clinical Biochemistry

Diagnosis: the classic laboratory features of HS include minimal or no anemia, reticulocytosis, an increased mean corpuscular hemoglobin concentration (MCHC), spherocytes on the peripheral blood smear, hyperbilirubinemia, and abnormal results on the osmotic fragility test (the most sensitive test to help detect HS is the incubated osmotic fragility test performed after incubating RBCs for 18-24 hours under sterile conditions at 37C - hemolysis of HS cells may be complete at a solute concentration that causes little or no lysis of normal cells). o RBC morphology is distinctive yet not diagnostic. o Other biochemical changes of hemolysis also are present, including increased LDH, increased unconjugated bilirubin, and decreased serum haptoglobin. o An increased MCHC obtained from an electronic cell counter is a characteristic feature of red cells in HS. MCHC values greater than the upper limit of normal (35-36%) are common. This increased MCHC is a result of mild cellular dehydration. The MCV in patients with HS is low. This relatively low MCV may reflect membrane loss and cell dehydration. o Further characterizing the specific membrane lesion by looking for abnormalities in spectrin, ankyrin, pallidin, or band 3 is possible. However, these studies are not routine and are available only in select research laboratories. o The initial workup if hemolysis is suggested should include the following: Hb, MCHC, MCV, Reticulocyte count Lactate dehydrogenase Fractionated bilirubin Haptoglobin Peripheral smear: Howell-Jolly bodies may be present, indicating remnant splenic tissue if the patient has had their spleen removed. Findings also may include megalocytosis. Vitamin B-12 and folate: This should be measured to determine the nutritional stores during recovery from an aplastic crisis. Herpes simplex virus, HPV type 19, and infectious mononucleosis: Testing for these may help identify an infectious etiology for the aplastic crisis. o Elliptocytosis due to a mutated band 4.1 o Membrane lipids: a--lipoproteinemia Enzymatic causes: G6PD (x-linked), glutathione synthetase, pyruvate kinase, hexokinase Disorders of hemoglobin synthesis (hemoglobinopathies): thalassemias, sickle cell anemia The acquired cause is due to Paroxysmal Nocturnal Hemoglobinuria (PNH): o The problem lies in the lack of anchoring proteins for DAF and MIC, leading to noninhibition of the complement membrane destruction complex. Extracorpuscular The acquired causes can be: Immune causes: autoimmune (as in transfusion reactions and SLE), alloimmune (as in the Rh disease of the newborn) or drug-associated (penicillin or methyl-DOPA). RBC fragmentation (physical damage on abnormal surfaces (artificial heart valves) or microangiopathic damage: disease of small blood vessels, in DM for example, which may lead to DIC, HUS or TTP (thrombic thrombocytopenia purpura). March hemolytic anemia Infections with meningococci, malaria or Clostridium perfringens Secondary to liver disease or renal disease. Due to chemical or physical agents such as snake venom, insect bite and burns.

Avi Sayag

Clinical Biochemistry

Topic 12
Laboratory diagnostics of megaloblastic anemias
Megaloblastic anemias are macrocytic (MCV>100 fL) and hyperchromic (MCH>31pg/RBC). The 2 main causes are vitamin B12 deficiency and folic acid deficiency. Folic acid deficiency Diet is not a common cause for folic acid deficiency. Although it is present in almost all foods, it is destroyed in 10-15 minute cooking. It is distributed widely in green leafy vegetables, citrus fruits, and animal products. Humans do not generate folate endogenously because they cannot synthesize PABA (p-aminobenzoic acid), nor can they conjugate the first glutamate. Folates are present in natural foods and tissues as polyglutamates because these forms serve to keep the folates within cells. In plasma and urine, they are found as monoglutamates because this is the only form that can be transported across membranes. Enzymes in the lumen of the small intestine convert the polyglutamate form to the monoglutamate form of the folate, which is absorbed in the proximal jejunum via both active and passive transport. Within the plasma, folate is present, mostly in the 5-methyltetrahydrofolate (5-methyl THF) form, and is loosely associated with plasma albumin in circulation. The 5-methyl THF enters the cell via a diverse range of folate transporters with differing affinities and mechanisms (i.e., ATPdependent H+ co-transporter or anion exchanger). Once inside, 5-methyl THF may be demethylated to THF, the active form participating in folate-dependent enzymatic reactions. Cobalamin (B12) is required in this conversion, and in its absence, folate is "trapped" as 5-methyl THF. From then on, folate is no longer able to participate in its metabolic pathways, and megaloblastic anemia results. Large doses of supplemental folate can bypass the folate trap, and megaloblastic anemia will not occur. The biologically active form of folic acid is tetrahydrofolic acid (THF), which is derived by the 2-step reduction of folate involving dihydrofolate reductase. THF plays a key role in the transfer of 1-carbon units (such as methyl, methylene, and formyl groups) to the essential substrates involved in the synthesis of DNA, RNA, and proteins. More specifically, THF is involved with the enzymatic reactions necessary to synthesis of purine, thymidine, and amino acid. Manifestations of folate deficiency thereafter, not surprisingly, would involve impairment of cell division, accumulation of possibly toxic metabolites such as homocysteine, and impairment of methylation reactions involved in the regulation of gene expression, thus increasing neoplastic risks. A healthy individual has about 500-20,000 mcg of folate in body stores. Humans need to absorb approximately 50-100 mcg of folate per day in order to replenish the daily degradation and loss through urine and bile. Otherwise, signs and symptoms of deficiency can manifest after 4 months. Signs and symptoms: Anemia: weakness, vertigo, tinnitus, palpitations, angina, pallor with slightly icteric skin and eyes. GI: glossitis (sore tongue), cheilosis angularis, diarrhea, weight loss There are no neurological signs. B12 deficiency Vitamin B12 is absorbed in the ileum complexed to intrinsic factor secreted from the parietal cells of the stomach. It is transported in the blood bound to transcobalamin II. In its methylated form it facilitates the conversion of homocysteine to methionine a reaction which is carried out by the conversion of methyl-THF to THF. As mentioned, THF is required to synthesize purines (thymidine). Pathophysiology: vitamin B12 deficiency is caused by failure to absorb the vitamin due to autoantibodies directed against the gastric parietal cells, the intrinsic factor, the IF-B12 complex or against the receptors in the ileum.

Avi Sayag

Clinical Biochemistry

Signs and symptoms: Anemia: weakness, vertigo, tinnitus, palpitations, angina, pallor with slightly ichteric skin and eyes. GI: glossitis (sore tongue), cheilosis angularis, diarrhea, weight loss Neurological signs: numbness and parasthesias in the extremities, ataxia, poor finger coordination, sphincter disturbances, forgetfulness, severe dementia, and psychosis. Lab: MCV > 100 fL (macrocytic anemia) In the peripheral smear macro-ovalocytes can be found as well as hypersegmented granulocytes, and the reticulocyte count is low. Unconjugated serum bilirubin is elevated as well as LDH1. Autoantibodies can also be found in the serum or in the gastric juice. In the bone marrow there are megaloblasts and ineffective erythropoesis. Endoscopy may reveal atrophy of the gastric mucosa that may lead to achlorhydria. This poses an increased risk for gastric carcinoma. Serum B12 can be directly measured (plasma immunoassay) Schilling test is positive and helps to differentially diagnose GI disease from pernicious anemia: i. First, IM injection of B12 is given to saturate the transcobalamin II stores. At the same time, the patient is given labeled B12 per os (the most commonly used radiolabels are 57Co and 58Co). ii. Urine is then collected over 24 hours. A normal result is documented when more than 10% of the given labeled B12 is excreted in the urine. iii. If less than 10% is collected, the test is repeated with a modification: labeled B12 and IF are given per os, and urine is collected over 24 hours. If more than 10% is excreted this time, then pernicious anemia is diagnosed. However, if the result is still not normal, then a GI disease is suspected and should be further investigated. Summary and presentation of the topic: Classify megaloblastic anemia (hyperchromic, macrocytic) and give values of MCV and MCH Mention the 2 main causes: folate and B12 deficiency Speak about folate: o Forms and evolution: poly mono (methyl-FH4) FH4 (active form) o Absorbance (prox. jejunum, active + passive) o Participation in metabolism and the role of B12 o Reference range: 3-20 g/L o Signs and symptoms of folic acid deficiency o Diagnosis: plasma immunoassay (RBC is better, as the concentration in RBCs reflects the body's folate reserves, range >140g/L , while plasma concentration reflects recent dietary intake). Speak about B12: o Absorbance in the ileum + IF o Transport in the blood with transcobalamine II o Functions of B12 o Normal range: 130-700 ng/L o Recommended daily intake: 3-5g (the liver stores ~3mg enough for years) o Sources of deficiency (autoimmune, gastrectomy, Crohn's, low intake) o Signs and symptoms of deficiency o Diagnosis: plasma immunoassay, Schilling test Lab: Hb, MCV, MCH, B12, folic acid + normal ranges + lab procedures Additional tests: endoscopy, BM examination, blood smear

Avi Sayag

Clinical Biochemistry

Topic 13
Major laboratory characteristics of acute and chronic lymphoid leukemias
ALL Most common between 2-10 years of age. 85% B-cells, 15% T-cells. More then 20% blasts in peripheral blood and BM. Risk factors: Down syndrome, Fanconi's anemia, AT, chemical drugs, in utero radiation. Involves the enlargement of lymph nodes, spleen and liver. Complete remission in 90% of cases and while 66% undergo full recovery. FAB classification: o L1: small blasts, homogenous population, narrow cytoplasm. o L2: larger blasts, heterogeneous population, prominent nucleolus, wider cytoplasm. o L3: homogenous population, middle-large blasts, basophilic cytoplasm with vacuoles (Burkitt lymphoma). Another classification according to the immunophenotyping: o Pre-B-cell ALL: CD19, CD79a o Common ALL: CD19, CD79a, CD10 o Late pro-B-cell ALL: CD19, CD79a, CD10, -chain o B-cell ALL: with surface Ig. B-cell markers: CD 19, CD 20, CD 22, CD 79a, HLA-DR (young: CD 10) Prognosis: o L3 - unfavorable. o CD 10, T-cell ALL - unfavorable. o Adult > 30 G/L and children > 50 G/L - unfavorable. o Age: less then 2 years, older then 10 years, older then 35 (adult) unfavorable. o Less then 45 chromosomes unfavorable. o More then 50 chromosomes favorable. o Translocations t(8:14), t(9:22), t(1:19), t(4:11) unfavorable. o Translocations t(12:21) favorable. CLL In the spleen the germinal center is surrounded by two separate zones: the mantle zone (close to the germinal center and IgD positive) and the marginal zone. Both zones are populated by B-lymphocytes (the marginal zone lymphocytes have irregular nuclei and the cytoplasm appears more empty). In the lymph node, there is a mantle zone, and B-cells surround it. However, they do not form a special zone, but are rather mixed with the lymphocytes in the mental zone. CLL is the leukemic counterpart of SLL (small lymphocytic lymphoma) Usually affects people over 50. Not aggressive. Often asymptomatic, those who are symptomatic have general symptoms: general symptoms of malignancy, lymphadenopathy, hepatosplenomegaly, leukocytosis (around 200000/L), hypogammaglobulinemia (prone to infections), anemia, autoimmune hemolytic anemia, and thrombocytopenia. The organs most commonly involved are the bone marrow and the blood (these are the primary sites), the lymph nodes, spleen, liver, skin and tonsils. Does not transform into ALL Clonal expansion of mature lymphocytes (98% B-cell, 2% T-cell). Gumprecht shadow. More than 30% lymphocytic infiltration of the bone marrow.

Avi Sayag

Clinical Biochemistry

In the serum of these patients there are low levels of Igs, paraproteins are rare and uric acid level is elevated. Immunophenotyping: B-cell markers (CD19, CD20, CD22), T-cell markers (CD5), CD23. Cytogenetics: 12 trisomy; deletions 11,13,17; no translocation involving CycD (no t(4:11) ), as it must be differentially diagnosed from mantle zone lymphoma (centrocytic lymphoma) which is characterized by t(4:11)(cycD+), CD5+, no +12, and CD43. CLL/SLL ALL 98% B cells 85% B cells No translocations associated t(4:11), t(8:14), t(1:19), t(9:22), t(12:21) +12, deletions 11, 13, 17 Chromosomes < 45, >50 CD19, CD20, CD22, CD23, CD5 CD19, CD20, CD22, CD10 Associated with hypo--glubolinemia and hyperuricemia Stem cell early pro-B late pro-B large pre-B small pre-B immature B mature B In general, ALL is characterized by arrested maturation somewhere between late pro-B and small pre-B/immature B, while CLL is characterized by arrested maturation and proliferation of activated B cell or memory B cell.

Characterization of leukemic cells by morphology (practical topic 7)


Myelopoiesis: Myeloblast promyelocyte myelocyte metamyelocyte band form (for PMNs) neutrophils/eosinophils/basophils 1. Myloblasts: > 10m, basophilic plasma, no granulations, loose chromatin in the nucleus, multiple nucleoli, a halo around the nucleus. 2. Promyelocyte: the largest cell in the lineage, many azurophilic granules, decreased nucleus:plasma ratio. 3. Myelocytes: less basophilic plasma, lower number of granules, no nucleolus. 4. Metamyelocytes: secondary/specific granules appear; the nucleus is indented and bean-shaped. 5. Band form: stick-shaped nucleus; grayish-blue granulation 6. Neutrophils: the nucleus has several lobes; bluish granulation. 7. Eosinophils: bi-lobed nucleus, pink plasma with reddish-brown large granules. 12-17m. 8. Basophils: rough, purple-black granulation that covers even the nucleus. 1014m. 9. Monocytes: grayish-blue plasma with azurophilic granules. The nucleus is bean-shaped. 12-20m. 10. Lymphoblasts: large nucleus:plasma ratio; purplsh plasma; nucleoli are present; it takes an expert to distinguish them from myeloblasts. 11. Small lymphocytes: small cells (10-12m); basophilic cytoplasm; the nucleus is condensed and purple. 12. Large lymphocytes (activated): loose chromatin and less basophilic cytoplasm. 13. Large granular lymphocytes: large cells, rough azurophilic granules. 14. Plasma cells: blue cytoplasm; dark-purple nucleus in the periphery. 15. Megakaryocyte: large cell; cloudy cytoplasm with azurophilic granulation; sometimes detached platelets can be seen in the surrounding; the nucleus is multilobed. 50-70m. As a general rule, when cells mature, their size decreases as well as the nucleus:plasma ratio. Nucleoli disappear from the nucleus, and the nuclear material becomes more condensed.

Avi Sayag

Clinical Biochemistry

Topic 14
Major laboratory characteristics of acute and chronic myeloid leukemias
AML AML primarily affects older adults, with the median age being 50 years (nonetheless, it can affect all age groups). The clinical signs and symptoms are usually related to marrow failure caused by the replacement of normal marrow elements by leukemic blasts. Fatigue and pallor, abnormal bleeding, and infections are common Splenomegaly and lymphadenopathy are in general less prominent than in ALL, but, rarely, AML presents as a discrete tissue mass (a so-called granulocytic sarcoma). Ideally, the diagnosis and classification of AML are based on the results of morphologic, histochemical, immunophenotypic, and karyotypic studies. Of these tests, karyotyping is most predictive of outcome: Most AMLs are associated with acquired mutations in transcription factors that inhibit normal myeloid differentiation, leading to the accumulation of cells at earlier stages of development. t(15;17) translocation occurs in acute promyelocytic leukemia. This translocation results in the fusion of the retinoic acid receptor (RARA) gene on chromosome 17 with the PML gene on chromosome 15. The chimeric gene(s) produce abnormal PML/RARA fusion proteins that block myeloid differentiation at the promyelocytic stage. Pharmacologic doses of retinoic acid, a vitamin A analogue, overcome this block and cause the neoplastic promyelocytes to terminally differentiate into neutrophils and die. Because neutrophils live, on average, for 6 hours, the result is the rapid clearance of tumor cells and remission in a high fraction of patients. The effect is very specific; AMLs without translocations involving RARA do not respond to retinoic acid. Classification: o M0 AML without maturation: 2% of AML cases; Diagnostic criteria: in the bone marrow there are no azurophilic granulation, no Auer rods in the blast cells; MPO is positive in less than 3% of cells, and immunophenotyping is needed. o M1 AML with minimal maturation: 10-20% of AML cases. Diagnostic criteria (in the bone marrow): azurophilic granules and/or Auer rods in less than 10% of cells; 90% of non-erythroid cells are myeloblasts; MPO (or Sudan black) is positive in more than 3% of blasts. o M2 AML with maturation: 30-45% of AML cases. Diagnostic criteria (in the bone marrow): azurophilic granules and/or Auer rods in more than 50% of cells; 20-90% of non-erythroid cells are myeloblasts; less than 20% of cells are monocyte precursors; MPO (or Sudan black) is positive o M3 AML promyelocytic (hypergranular) 10-15% of AML cases Diagnostic criteria (in the bone marrow): strong granulation and lots of Auer rods; more than 50% of cells are abnormal promyelocytes; DIC is characteristic of this class. o M4 AML myelomonocytic 15-20% of AML cases Diagnostic criteria (in the bone marrow): more than 20% of nucleated cells in the bone marrow should be myeloblasts and promyelocytes;

Avi Sayag

Clinical Biochemistry

more than 20% of bone marrow cells should be promonoblasts and monoblasts (stain with esterases such as NSE). Gum hypertrophy and tissue infiltration are characteristic of this class. o M5a AML monoblastic Diagnostic criteria (in the bone marrow): more than 80% of nonerythroid cells in the bone marrow belong to the monocytic lineage; more than 80% are monoblasts. Gum hypertrophy and tissue infiltration are characteristic of this class. o M5b AML monocytic Diagnostic criteria (in the bone marrow): more than 80% of nonerythroid cells in the bone marrow belong to the monocytic lineage; less than 80% are monoblasts; monocytes and promonocytes comprise more than 20%. Gum hypertrophy and tissue infiltration are characteristic of this class. o M6 AML erythroleukemia Diagnostic criteria (in the bone marrow): more than 50% of nucleated cells in the bone marrow are early erythroid precursors; more than 20% of non-erythroid cells in the bone marrow are myeloblasts; PAS+ erythroids. o M7 AML megakaryoblastic Diagnostic criteria (in the bone marrow): more than 20% of nonerythroid cells in the bone marrow are blasts showing megakaryocytic differentiation ("budding" cytoplasm). Prognosis in AML: o Age of onset below 2 years and above 60 years is not favorable; o Pathomechanisms involving deletions in 5, 7 chromosomes, t(9:22) and t(11q23) are nor favorable. However, t(8:21) in M2 and inversion in chromosome 16 in M4 are favorable. t(15:17) in M3 is intermediately unfavorable. CML It is most common between 40-60 years of age, and more males are affected. There are 3 phases to the disease: o The chronic phase: 1/3 of cases are asymptomatic, while those who are have general symptoms such as anemia, fatigue, dyspnea, tachycardia, hepatosplenomegaly, abdominal discomfort, weight loss, night sweats and bleedings. o The accelerated phase: in this phase, patients exhibit enhanced splenomegaly, subfever (or fever), there are 5-20% blasts in the bone marrow and in the peripheral blood, thrombocytopenia or thrombocytosis that is unresponsive to treatment, an increase in the WBC count, and other genetic alterations that were not present at time of diagnosis. o The blastic phase (the crisis phase): in this phase there are more than 20% blasts in the bone marrow and in the peripheral blood. 1/3 of cases will transform to ALL and 2/3 to AML. CML should be differentially diagnosed from leukemoid reaction: in leukemoid reactions there is a dramatic increase in granulocyte count due to inflammatory processes going on. Therefore, GAPA score is positive in these reactions (Granulocyte Alkaline Phosphatase Activity on a 0-4 scale). Pathogenesis: t(9:22) BCR-ABL forming mutated tyrosine kinase; 95% of cases show this Philadelphia chromosome.

Avi Sayag
Lab: o o

Clinical Biochemistry

Blood count: anemia; thrombocytosis; leukocytosis Bone marrow: hypercellular (myeloids > erythroids); eosinophils and basophils; megakaryocytes are smaller o Cytochemistry: decreases GAPA o Cytogenetics: Ph. Chromosome Therapy: o Drugs (cytostatics) o IFN- (arrests cell cycle) o TK inhibitors (imatinib) o BM transplantation (allo).

Characterization of leukemic cells immunophenotyping (practical topic 8)

by

cytochemistry

and

Cytochemical reactions For the identification of leukemic cells and subtype the evaluation of the intracellular material is needed (enzymes, stored substances), since undifferentiated cells display a lot of similarities. 1. Myeloperoxidase (MPO): used to differentiate AML from ALL. Positive reaction: grayish-black. 2. Sudan black: detects the phospholipids in leukocyte granules. Positive reaction: black. 3. Non-specific esterase: the most abundant activity is detected in monocytes and promyelocytes. Positive reaction: light brown. 4. PAS: detects the glycogen stored in the cytoplasm. Positive reaction: red. 5. Lysozyme: the enzyme dissolves the bacterial cell wall. Monoblasts and immature monocytes are positive. 6. Prussian blue: detects iron depletion. Positive in ring sideroblasts. Positive reaction: blue. 7. GAPA: granulocyte alkaline phosphatase converts -naphtyl-phosphate into a brownish precipitate in alkaline pH. The phosphatase reaction is low or absent in CGL, but its activity is enhanced in myeloproliferative disorders and in leukemoid reactions. 8. Acid phosphatase: positive reaction: red. Positive in MM and HCL. Sudan black: positive in M1, M2, M3, M4. M5: 0/+, all the rest: 0. MPO: positive in M3, M4. All the rest: 0/+ NSE: positive in M3, M4, M5. ALL: 0, all the rest: 0/+ PAS: positive in M6, ALL, all the rest: 0/+. Acid phosphatase: positive in MM and HCL (TRAP+). Type MPO Sudan PAS NSE Lysozyme AP AML- M1, 0/+ ++ 0/+ 0/+ 0 M2 M3 ++/+++ ++ 0/+ +++ 0 M4 + ++ 0/+ ++ 0/+ M5 0/+ 0/+ 0/+ +++ ++ M6 0/+ 0 + 0/+ 0 M7 0/+ 0 0/+ 0/+ 0 ALL 0 0 + 0 0 MM +++ HCL ++ (TRAP)

Immunophenotyping

Avi Sayag

Clinical Biochemistry

This method is based on the detection of cell surface markers and intracytoplasmic markers. It is based on the identification of specific CD markers on pathological cells by fluorophor labeled monoclonal Abs. Since these antigens are already present on immature cells, the method allows the analysis of undifferentiated cells and the determination of subgroups of leukemias. The primary diagnostic goal in acute leukemias is the characterization of the blast cells and the detection of abnormal marker expressions. The most important CDs characteristic for cell lines are as follows: B-cells: CD19, CD20, CD22, HLA-DR CD10: cALLA (common acute lymphoid leukemia antigen) can be found on B cell precursors, it is absent on B cells in the peripheral blood of healthy individuals. T-cells: CD3, CD5, CD7, CD4, CD8 Myeloid cells: CD13, CD33, CD15, MPO Monocytes: CD14, HLA-DR Megakaryocytes: CD41, CD42, CD61 Stem cell marker: CD34: not detectable in the peripheral blood and less than 5% in the bone marrow of healthy individuals.

Avi Sayag

Clinical Biochemistry

Topic 15
Laboratory diagnostics of quantitative platelet disorders
Platelet disorders Qualitative Platelet function disorders Thrombocytopenia : (topic 19) Reduced production Increased destruction Loss from the body Abnormal distribution/increased trapping in the spleen Quantitative Thrombocytosis

Thrombocytopenia Thrombocytopenia is characterized by spontaneous bleeding, a prolonged bleeding time, and a normal PT and APTT. Normal platelet count 150-400 G/L Thrombocytopenia less than 100 G/L Thrombocytopenia with tendency to bleed less than 50 G/L Thrombocytopenia with spontaneous bleeding less than 10 G/L (in some sources less than 20 G/L). Larger hemorrhages into the central nervous system are a major hazard in patients with markedly depressed platelet count. In most cases in which the cause is accelerated destruction, the bone marrow reveals a compensatory increase in the number of megakaryocytes. Hence, bone marrow examination can help to distinguish the two major categories of thrombocytopenia. It is also worth emphasizing that thrombocytopenia is one of the most common hematologic manifestations of AIDS. 1. Decreased production Congenital Neonatal Acquired in newborns As part of a general bone May-Hegglin anomally: a Typical depression with genetic disorder of platelets infected with rubella, and in marrow that causes them to be babies of mothers who take selective megakaryocyte depression. Causes include: abnormally large. The cause thiazides or tolbutamine. is a mutation in the gene of 1. Chemotherapy/radiation non-muscle myosin heavy 2. Alcohol (suppresses chain IIA. The pathogenesis production of megak.) is unknown. It is 3. Cytotoxic drugs 4. Aplastic anemia characterized by hypoplasia 5. Pernicious anemia of megakeryocytes, Dohle 6. Infection with CMV, bodies (small inclusions in PMNs) and giant platelets. EBV, VZV, rubella, measles vaccine 7. Infiltration of BM with tumor cells. 8. Myeloid dysplasia with primary myelosclerosis.

Avi Sayag

Clinical Biochemistry

2. Increased destruction This can be due to 2 reasons: 1. Destruction by immune mechanism: a. Chronic idiopathic thrombocytopenia purpura (ITP) i. The most common form in women between 15-50 years old that suffer from petechial hemorrhages and are easily bruised. ii. Can be secondary to SLE, HIV, CLL, etc (but then it is not idiopathic). iii. Platelets can be sensitized by Ab directed against GpIIb-IIIa iv. Premature removal by the RES life-span is reduced to few hours. v. The platelet count is 10-50 G/L. vi. Platelets are large. vii. Lab detection of Ab in the serum or on platelets. viii. There is normal to increased number of megakaryocytes. b. Acute ITP i. Most common in children ii. 75 % of them get it after vaccination or infection. iii. There is a spountandeous remission. iv. 5-10% become chronic. c. Drug induced thrombocytopenia i. Abs against the drug and the carrier protein are produced the circulating immune complexes are adsorbed into the platelets platelets are removed by the RES or lysed by complement. ii. Quinine, quinidine, heparin iii. Count: less then 10 G/L. iv. Normal to increased number of megakaryocytes. d. Neonatal alloimmune: the mother becomes sensitized against platelet-specific antigen of the fetus. e. Neonatal autoimmune: ITP develops in pregnant women; Ab crosses the placenta. f. Post-transfusion thrombocytopenia: Anti-Pl Abs develop following transfusion. g. Secondary autoimmune: in CLL, SLE, etc. 2. Destruction by non-immune mechanism: a. In pregnancy of pre-eclampsia (ischemia vascular injury activation of DIC) b. Thrombotic thrombocytopenia purpura (TTP) i. Thrombi in capillaries and arterioles RBCs and PLTs are mechanically destroyed by the thrombi intravascular hemolysis reticulocytosis ii. Women between 20-50 are usually affected. iii. Severe organ damage. iv. Count less then 20 G/L. v. Pathomechanism: 1. Large vWF deficiency in a MMP called ADAMTS13 which degrades very-high-molecular-weight multimer of vWF allows multimers of vWF to accumulate in plasma. 2. Endothelial damage. 3. Defective prostacyclin production. c. In hemolytic uremic syndrome (HUS) i. Hemolytic anemia (decreased Hb, increased ret., schistocytes) ii. Renal failure (elevated urea, creatinine, RBCs, proteins) iii. Thrombocytopenia iv. Disease of childhood (6 months 4 years).

Avi Sayag

Clinical Biochemistry

v. Often follows acute viral infection or E. coli (verotoxin damages the endothelium bleeding, activation and consumption of PLTs) vi. Resembles TTP with no neurological symptoms. vii. High mortality rate. d. DIC 3. Loss from the body 4. Disorders related to distribution/dilution i. Increased pooling in splenomegaly. ii. Hypothermia iii. Extracorporal circulation iv. Massive blood transfusion Thrombocytosis 1. Reactive thrombocytosis: blood loss, surgery, post-splenectomy, iron deficiency due to blood loss, inflammation, stress, exercise. 2. Myeloproliferative disorders: polycytemia vera, myelofibrosis with myeloid metaplasia. 3. Essential thrombocytemia: megakaryocyte proliferation with high PLT count (500-2000G/L); recurrent hemorrhage and thrombosis; abnormally large PLT and megakaryocyte fragments in blood film; may transform to polycytemia vera, myelofibrosis or acute leukemia, but may remain stationary for many years.

Avi Sayag

Clinical Biochemistry

Topic 16
Inheritance of ABO blood group system and its clinical significance
The ABO antigens are added stepwise to proteins or to lipids on the erythrocytes and they appear on day 40 of gestation. The substrate molecule is L-fucose (the H antigen). The A antigen is N-acetyl-galactosamine (GalNAc); The B antigen is galactose (Gal);

A and B genes code for transferase enzymes, such that transferase A is alpha 1-3-Nacetylgalactosaminyltransferase and transferase B is alpha 1-3-galactosyltransferase. The antigens are found not only on RBCs, but also on most body cells (leukocytes, platelets, etc.) The gene coding for blood type lies on chromosome 9q34. However, other separate genes on chromosome 11 and 19 actually interact with the blood type gene, determining our ability to secrete our ABO blood type antigens into our body fluids and secretions. This is called the secretor gene, and by testing for this gene we can determine whether we are secretors or non-secretors. In the genetics of the secretor system two options exist. A person can be either a secretor (Se) or a non-secretor (se). This is completely independent of whether one's blood type is A, B, AB, or O. Thus a person could be an A secretor or an A nonsecretor, a B secretor or a B non-secretor, etc. Secretors: in a simplified sense, a secretor is defined as a person who secretes one's blood type antigens into body fluids and secretions like the saliva, the mucus in the digestive tract and respiratory cavities, etc. Non-Secretors: non-secretors on the other hand put little to none of their blood type into these same fluids. As a general rule, in the US about 15-20% of the population are non-secretors with the remaining 80-85% being secretors. Aside from the physical implications centering around whether you have blood type antigens in your body fluids or not, the secretor genetics have additional significance through the effects of gene linkage: in other words, the outcome of your secretor genetics links to other seemingly unrelated genes and influences their function. Your secretor status drastically alters the carbohydrates present in your body fluids and secretions in addition to several important aspects of your metabolism and resistance. These factors include the activity of an enzyme called intestinal alkaline phosphatase, the overall composition of bacteria in your intestinal ecosystem, your propensities toward blood clotting, your level of carbohydrate tolerance, and your resistance to certain parasites and yeast (based on these features, new diet regimes have been proposed tailored to the blood type of those who are desperately willing to try anything to lose weight). The natural ABO antibodies are IgM, which are produced after the first few months of life (after the 4th month). These antibodies work in "cold" temperatures (room temperature) and may fade in old age. Other irregular Ab are produced after immunization (incompatible

Avi Sayag

Clinical Biochemistry

transfusion or pregnancy). Most complications after such incompatible transfusions are caused by these Ab (mostly IgG warm Ab that cross the placenta). Antigens and antibodies: Blood group Ag on RBC Ab in serum Genotype A A anti B AA or Ao B B anti A BB or BO AB A and B none AB O none (H) anti A and anti B - OO Subtype A The A blood type contains about 20 subgroups, of which A1 and A2 are the most common. 80% of which are A1. These cells carry about 1 million antigens on a single RBC. The A2 cells, however, carry only 200,000 antigens on a single RBC. A1 and A2 differ in such a way that type A2 people can produce anti-A1 antibodies. As mentioned, there are weaker subgroups in the A subtype (about 18) due to mutations in the A gene that may lead to dysfunctional transferase (3--N-acetylgalactosaminyl-transferase). The clinical significance of the ABO group manifest in mismatched transfusions, although rare. However, should they occur, they may be life-threatening, as they may lead to intravascular hemolysis. It is more severe in group O patients, as they have very reactive antiA and anti-B antibodies. Universal donors and recipients: O group carries no A or B antigens. O The packed and processed units have little or no antibody content. The AB group has no antibodies whatsoever, and therefore cannot lyse any transfused cells. Other antibodies, however, may be present! Prevalence: A B Bororo (brazil) and Peru (Indian) 100% type O North American Indians 82% type A Type O is the most prevalent in all parts of the world, except N. American AB Indians. The Bombay phenotype Individuals with the rare Bombay phenotype (hh) do not express antigen H on their red blood cells. As H antigen serves as precursor for producing A and B antigens, the absence of H antigen means the individuals do not have A or B antigens as well (similar to O blood group). However, unlike O group H antigen is absent, hence the individuals produce isoantibodies to antigen H as well as to both A and B antigens. In case they receive blood from O blood group, the anti-H antibodies will bind to H antigen on RBC of donor blood and destroy the RBCs by complement-mediated lysis. Therefore, Bombay phenotype can receive blood only from other hh donors (although they can donate as though they were type O). It should be mentioned that the transferase enzyme IS produced in this phenotype. Summary and presentation of the topic: Present the blood types and the biochemistry of the various types + prevalence Explain the difference between them in terms of chemistry, Ag, Ab and transferases Chromosome 9q34 and interactions with chromosomes 11 and 19 Se and se (secretor and non-secretor). The significance of being an secretor or a non-secretor. Subtype A distribution of A1 and A2 Clinical significance: blood transfusion universal donors and recipients Bombay phenotype

Avi Sayag

Clinical Biochemistry

Topic 17
Inheritance and clinical significance of Rh blood group system
The Rh blood group system is the second most important system and is the most complex. It is important because it is associated with hemolytic transfusion reactions and with development of severe hemolytic disease of the newborn (HDN). Rh antigens are proteolipids and lack carbohydrate. Rh antigens are present on RBCs only, and there are 20,000 antigen sites on each erythrocyte. They are relatively small (32KDa). The inheritance of Rh antigens is determined by a complex of 2 closely linked genes on chromosome 1. One gene codes for the protein carrying D expression; the other codes for the proteins carrying C or c and E or e expression. Rh-positive individuals have both a D and a CE gene while Rh-negative individuals have only a CE gene. Depending on which genes are present on a chromosome, 8 common antigen combinations or haplotypes are possible: Dce, DCe, DcE, DCE, dce, dCe, dcE, dCE. Common phenotypes and genotypes are given below. D antigen is the most important Rh antigen. Presence of a single D antigen confers upon an individual the designation Rh-positive; its absence means that the person is Rh-negative. Eighty five percent of Caucasians, 92% of African Americans and 99% of Asian Americans are Rh positive. The letter d is commonly used to indicate the lack of D in Rh-negative individuals, but neither d antigen nor anti-d has been detected. The Fisher-Race nomenclature has been more widely adopted over the more complex Wiener nomenclature for Rh antigens. However, an abbreviated version of the Wiener system is useful to describe Rh genotypes. Wiener is covenient because it uses a single letter, R or r, with superscripts to name a 3 locus haplotype. It is possible to translate from one nomenclature to the other by remembering a few rules: In the Wiener system, D is indicated by an uppercase R and the absence of D is indicated by lower case r. In the Wiener system, superscripts or numbers are used to indicate which Cc or Ee genes are present. Numbers are used with R and primes are used with r. In the Fisher Race system, loci are lined up in the order Dd, Cc, Ee (e.g. DCE). In the Wiener system, the Dd position is numbered 0, Cc position is 1 and Ee position is 2. The Wiener superscript of 0, 1, 2 indicates which of the Fisher Race loci is in its uppercase form (D, C, or E). For example, 1 or prime indicates that C is capitalized, while a 2 or double prime indicates that E is capitalized. Rh Nomenclature and Haplotype Frequencies Fisher Race Wiener Dce Ro DCe R1 DcE R2 DCE Rz dce r dCe r' dcE r" dCE ry The most common haplotye in Caucasians and Asian Americans is DCe (R1), while the most common phenotype in African Americans is Dce (Ro). Red blood cells that fail to react with all Rh antibodies are called Rh null. An individual's genotype can only be determined with certainty by performing DNA analysis and family studies. Unlike the ABO naturally occurring antibodies, Rh antibodies are produced in response to an incompatible transfusion or pregnancy. The D antigen is the most immunogenic of the Rh antigens, causing immunization at least 80% of the time when a D-negative person receives a single unit of D-positive blood. Anti-c is the second most important Rh antibody. Although anti-E is more common than anti-c, anti-E is frequently a naturally occurring antibody. Anti-c and anti-e only occur after an antigenic stimulus.

Avi Sayag

Clinical Biochemistry

Weak D (Du): some individuals have a weak expression of the D antigen for either which of 3 reasons: Individuals who lack part of the D antigen (partial D) have a weak expression on their RBCs. If they are immunized, they produce antibodies to the portion they lack. The D gene encodes all epitopes of the D antigen, but the antigen number on RBCs is less than normal. In some cases a C transposition to a D gene occurs (Dce/Ce or DCe/Ce) which weakens the expression of D. Significance: 80% of Rh(D)- persons exposed to Rh(D)+ blood will develop anti-D antibodies. Anti-D antibodies can also be stimulated by pregnancy with an Rh(D)+ baby. Sensitization can be prevented by the use of anti-D Ig antenatally and postnatally. Rh(D)- females of childbearing potential should never be given Rh(D)+ blood. Antibodies to Rh antigens are primarily IgG antibodies which can cross the placenta. IgG is too small to make bridges between RBCs, so agglutination doesn't occur in saline. These antibodies work best in "warm" temperatures. The D antigen is the most important cause of Hemolytic Disease of the Newborn (HDN). Other antigens can also cause it (C, c, E, e) as well as other blood groups (rare). Finally, it should be noted that the inheritance of ABO and Rh are not linked and are inherited independently. Summary and presentation of the topic: Rh Ags are proteolipids found only on RBCs, with 20000 Ags on each RBC Inheritance 2 genes on chromosome 1 (D and CE) 8 combinations Fisher-Race vs. Wiener Most common DCe (R1) Caucasians and Asian Americans Most common Dce (R0) African Americans Anti-Rh Abs are produced in response to transfusion, pregnancy, etc. D Ag is the most immunogenic c Ag is the second most immunogenic, and NOT naturally occurring Anti-e is not naturally occurring Anti-E is frequently naturally occurring and more common than anti-c. 3 causes of weak D (Du) Significance of Rh: transfusion, pregnancy, transfusion to women in childbearing age Abs are IgG Rh and ABO are not genetically linked

Avi Sayag

Clinical Biochemistry

Determination of ABO and Rh blood group (practical topic 9)


ABO determination Abs in the ABO group are naturally occurring ones: cross-reacting carbohydrate structures on environmental agents stimulate the thymus-independent production of IgM anti-A and/or antiB in individuals who are not tolerant to these antigens. The IgM Abs then directly agglutinate the appropriate antigen-positive RBCs, preferentially at room temperature. The Landsteiner rule: sera of healthy adults contain ABO Ab that reacts with the ABO Ags missing from the person's RBCs, but it must not contain any Ab that reacts with the Ags on the person's RBCs. Blood group Ag on RBC Ab in serum Genotype A A Anti-B AA or AO B B Anti-A BB or BO AB A and B None AB O None Anti-A and anti-B OO Bedside blood group determination can be one-sided or two-sided. 1. One-sided This examination is based on agglutination by addition of anti-A, anti-B and anti-AB antisera at room temperature and in saline medium. We put one drop of each anti serum in its place on the right around 2 cm apart, and then we put one drop of the patient's serum. Then, we put one drop of the patient's blood and saline opposite to each antiserum and serum of the patient. We mix them well using the corner of slide, caring not to contaminate one with the other. We allow the slide to stand for 30 seconds at room temperature and then tilt the slide backward and forward by 30. In the one-sided method, we determine the presence/absence of antigens on RBCs and only that! Hence, one-sided method. 2. Two-sided In this method, we determine both the Ags on the RBCs and the Abs in the serum of the patient (hence "two-sided"). We divide a tile into 2 parts by an imaginary line. We put one drop of anti-A, anti-B and antiAB sera with proper distance from each other on the left side, and mark their places for safety (see figure on page 55 of the practice book). We then make a 10% suspension of the blood sample in physiological saline and put one drop opposite each of them. On the right side we put 4 drops of the requested serum with sufficient space from each other. We put one drop of known A1, A2, B and O test RBCs opposite the blood sera. We mix them carefully and let them stand for 10 minutes at room temperature. We again tilt it and read the results visually. Rh determination The test is performed on a glass slide or white glass bottle filled with hot water and with a surface temperature of 37-42C. We place one drop of anti-D serum, one drop of the appropriate control reagent and to both drops we add one drop of a well-mixed 50% suspension of the investigated RBCs in their own serum or plasma. We then mix the RBC suspension and the reagent, and put it in a wet chamber at 37C for 20 minutes. We gently tilt the slide and observe for agglutination. After 2-3 minutes we record and interpret the results. We should use Rh+ and Rh- controls with the same method at the same time. Papain enzyme treatment of RBCs cleaves the sialoglycoproteins from the RBCs, and the net surface charge decreases. This enhances the agglutination reaction. It has a special importance in cases when allo-Abs or auto-Abs are bound to the surface of the RBCs as a result of previous incompatible transfusion. Rh+: the suspension is agglutinated and the autocontrol is homogenous. Rh-: if the suspension ad autocontrol are the same and homogenous. In case of an uncertain result (positive autocontrol, weak reaction, etc.), the sample should be sent to special departments for further examination.

Avi Sayag

Clinical Biochemistry

Topic 18
Coagulopathies, laboratory control of anticoagulant treatment
The coagulopathies can be inherited, acquired, iatrogenic or therapeutic. The coagulopathies can be caused by 3 main mechanisms: 1. Decreased levels or absence of clotting factors: In this category, the decreased levels can be caused by inherited disorders, acquired disorders (e.g. in liver failure which leads to decreased synthesis of clotting factors as well as in consumption coagulopathies) and by therapeutic mechanisms (agents leading to thrombolysis). 2. Synthesis of abnormal clotting factors In this category, the synthesis of abnormal clotting factors can be caused by congenital defects, by acquired disorders (such as in dysfibrinogemia in liver diseases and in vitamin K deficiency), by therapeutic agents (e.g. Syncumar, aka Coumarin) and by iatrogenic factors (such as administration of cephalosporin). 3. Inhibitors of coagulation In this category, the coagulopathies can be caused by neutralizing or non-neutralizing antibodies that are directed against clotting factors. This condition can be accompanied by inherited factor deficiency but not necessarily. However, not only antibodies can cause coagulopathies, but global inhibitors can as well (e.g. heparin). Bearing in mind the coagulation pathway, 4 main screening tests of blood coagulation should be mentioned: APTT, PT, TT and bleeding time.

Avi Sayag

Clinical Biochemistry
Extrinsic pathway PT 8-12 seconds + 4 seconds prolonged The prothrombin time is most commonly measured using blood plasma. Blood is drawn into a test tube containing liquid citrate. The blood is mixed, then centrifuged to separate blood cells from plasma. An excess of calcium is added (thereby reversing the effects of citrate), which enables the blood to clot again. Tissue factor (also known as factor III or thromboplastin) is added, and the time the sample takes to clot is measured optically or using the KC-1. TF is both the receptor and activator of FVII. If the clotting does not occur within 100 seconds, the result is said to be PT > 100 sec. Factors II, V, VII, IX and X are vit.-K dependent, therefore the PT test is good to monitor coumarin therapy. If so, the result should be given as

Intrinsic pathway APTT 28-40 seconds + 9 seconds prolonged After collecting blood samples in vacu-tubes with oxalate or citrate to arrest coagulation by binding calcium, the specimen is then delivered to the laboratory. In order to activate the intrinsic pathway, phospholipid, an activator (such as silica, celite, kaolin, ellagic acid), and calcium (to reverse the anticoagulant effect of the oxalate) are mixed into the plasma sample . The test is termed "partial" due to the absence of tissue factor in the reaction mixture. The time is measured until a thrombus forms. If the clotting does not occur within 200 seconds, the result is said to be APTT > 200 sec. Causes of prolonged APTT: 1. Deficiency or decreased "intrinsic factors": hemophilia A and B 2. Presence of heparin in the sample 3. Presence of inhibitors directed against clotting factors or phospholipids (lupus anticoagulant) 4. Inappropriate ratio of Nacitrate:blood (citrate in excess) 5. Consumption coagulopathies When do we use APTT? 1. To monitor unfractionated heparin therapy (UFH)2 2. Control of fibrinolytic therapy: before thrombolytic therapy, screening for hemorrhagic diathesis should be performed. 3. Diagnosis of DIC 4. Diagnosis of thrombophilia 5. In liver diseases 6. If a patient with severe bleeding is treated only with RBC concentrate, and does not receive the plasma clotting factors.

PT pt INR = PT contorl

ISI

. ISI is the International

Sensitivity Index. The smaller the ISI is, the more sensitive the reagent is. The INR should be kept between 2-3. INR of patients with prosthetic heart valve should be between 2.53.5. PT determination and INR should be performed every 2 weeks during the first 6 weeks of therapy, and then, if the INR is stable, once in a month. If INR is > 5, there is a risk of spontaneous bleeding. Causes of prolonged PT: 1. Coumarin therapy 2. Hereditary/acquired absence of "extrinsic" factors/abnormal synthesis 3. Inappropriate ratio of Nacitrate:blood (citrate in excess) 4. Fibrinolytic therapy When do we use PT? 1. To monitor coumarin therapy 2. Before thrombolytic therapy: screening for hemorrhagic diathesis should be performed. 3. Diagnosis of DIC 4. Liver disease

APTT taken after: UFH continuous I.V bolus heparin Subcutaneous heparin

0.5-1 hr X

2-3 hr X

4-6 hr

Avi Sayag

Clinical Biochemistry
Thrombin time (TT): TT evaluates the last phase of the clotting cascade and represents the time (in seconds) that elapses between the addition of thrombin (usually bovine thrombin) and the onset of clotting. The values of both the control's and the patient's plasma are reported. The reference interval is 14-22 seconds. TT is considered prolonged if the patient's TT exceeds the control value by 8 seconds. If clotting does not occur within 100 seconds, the result is given as TT > 100 seconds. Causes of prolonged TT: i. Heparin treatment ii. Pathologic levels of fibrinogen/fibrin split products (acute DIC, primary hyperfibrinolysis, dissolution of a thrombus) that inhibit the thrombin activity and fibrin polymerization iii. Severe hypofibrinogenemia or afibrinogenemia iv. Dysfibrinogenemia v. Certain hepatic diseases due to hypo- or dysfibrinogenemia. vi. If a patient with severe bleeding is treated only with RBC concentrate, and does not receive the plasma clotting factors. Bleeding time: apply 40 mmHg tourniquet pressure to the upper arm and maintain it during the entire process. Wipe the inner surface of the forearm with ethanol and, by avoiding larger visible veins, cut the skin using a special disposable device. The device is attached to the forearm without pressure, and pushing a trigger two 5-mm blades are released that make two 1-mmdeep cuts. Dry the blood with a sterile blotting paper every 30 seconds without touching the wound. The time when the last drop of blood is visible on the blotting paper is the bleeding time. The reference interval is 2.5-9.5 minutes. If bleeding does not stop within 20 minutes, the result is given as bleeding time > 20 minutes. Unfortunately, in most labs and hospitals the bleeding time is determined by pricking the fingertip. This is a completely unreliable method of no clinical significance, for the bleeding time in this case depends on the thickness of the skin of the fingertip and on the depth of the pricking, rather than on platelet function. Bleeding time is the most important screening test of platelet function, as bleeding of a small wound stops when platelets adhere to the injured vessel wall forming a primary platelet plug. Bleeding time is normal in coagulopathies with the exception of afibrinogenemia (i.e. it is normal in hemophilias!) Diseases that cause prolonged bleeding time include: i. Thrombocytopenia ii. DIC iii. Aspirin and other cyclooxygenase inhibitors can prolong bleeding time significantly. iv. While warfarin and heparin have their major effects on coagulation factors, an increased bleeding time is sometimes seen with use of these medications as well. v. People with von Willebrand disease usually experience increased bleeding time, as von Willebrand factor is a platelet agglutination protein, but this is not considered an effective diagnostic test for this condition. vi. It is also prolonged in hypofibrinogenemia.

Several coagulopathies should be mentioned: 1. Hemophilia A 2. Hemophilia B 3. Afibrinogenemia and dysfibrinogenemia 4. Other factor deficiencies 5. Acquired coagulopathies

Avi Sayag

Clinical Biochemistry

Hemophilia A
This bleeding disorder is caused by a mutation in FVIII gene, which is located on chromosome X (Xq28) and is the most common hemophilia. FVIII is a big glycoprotein synthesized in the liver, and perhaps in endothelial cells. It is a cofactor of FX activation (along with FIX). Thrombin is required to activate FVIII. The deficiency in FVIII can be inherited (a mutated FVIII gene) or acquired (antibodies directed against FVIII). Symptoms: hemophilia leads to a severely increased risk of bleeding from common injuries. The first symptoms that should suspect of hemophilia are bleeding at labor and during delivery, circumcision, vaccination and onset of walking. The sites of bleeding include the GI and the brain (less common) and the joints and muscles (most common). Hemarthrosis occurs primarily in the knee and elbow joints. As RBCs are lysed, iron is deposited in the synovium. This leads to chronic synovitis, synovial fibrosis, joint stiffness, limited motion and pain. Diagnosis: APTT is elevated, PT is normal, TT is normal and bleeding time is normal. Factor assay can also be performed. Hemophilia A should be differentially diagnosed from: 1. von Willenbrand disease; 2. Combined FVIII and FV deficiency; and 3. Consumption coagulopathies. 4. Hemophilia B 5. Vitamin K deficiency 6. Vitamin K antagonist drugs. Treatment: administration of FVIII.

Hemophilia B
Hemophilia B is caused by a mutation in the FIX gene located on the X chromosome (Xq27). FIX is synthesized in the liver and its function depends on vitamin K. Its cleavage is carried via FXI and FVII. In the presence of FVIII and Ca+2 it cleaves FX. Symptoms: Factor IX deficiency leads to an increased propensity for hemorrhage. This is in response to mild trauma or even spontaneously, such as in joints (haemarthrosis) or muscles. Diagnosis: APTT is elevated, with normal PT, normal TT and normal bleeding time. Factor assay can also be carries out. Hemophilia B should be differentially diagnosed from: 1. Liver disease 2. Vitamin K deficiency 3. Vitamin K antagonist drugs. 4. von Willebrand disease 5. Afibrinogenemia/dysfibrinogenemia 6. Fibrinolytic diseases 7. Hemophilia A Both hemophila A and B can be classified as severe (if the activity/level of the defective factor is less than 1% - leading to spontaneous bleeding); moderately severe (1-5% - bleeding due to minor trauma or surgery); or mild (5-30% - bleeding due to major trauma and surgery).

Afibrinogenemia and Dysfibrinogenemia


In this autosomal recessive disorder there is failure of the fibrinogen chains to assemble, or fibrinogen fails to be secreted (in afibrinogenemia there is no fibrin at all, whereas in dysfibrinogenemia the levels are low). Both are characterized by severe bleeding diathesis and platelet function disorder, as well as by spontaneous abortion, as there is a need for more than 1g/L of fibrinogen to maintain pregnancy. Diagnosis: APTT, PT, Fibrinogen levels, TT, and Bleeding time are all abnormal.

Avi Sayag

Clinical Biochemistry

Other factor deficiencies: FXI deficiency: APTT is elevated (only) Deficiency in FX, FVII, FV, FV-FVIII (combined), prothrombin, combined FII-FVIIFIX-FX. In these deficiencies the APTT and the PT are elevated (only). FXIII deficiency 2 plasmin inhibitor deficiency.

Acquired coagulopathies
1. Neutralizing and non-neutralizing antibodies which inhibit the function of clotting factors and accelerate their clearance. These antibodies can occur spontaneously or after replacement therapy of a certain factor that is absent in the patient's blood. 2. Non-antibody inhibitors (such as heparin) 3. Liver diseases which lead to a decreased factor synthesis 4. Vitamin K deficiency due to malabsorption or elimination of the intestinal bacterial flora. 5. Vitamin K antagonists: coumarin (a precursor of warfarin (Coumadin)), cephalosporin. The APTT and PT will be elevated. For APTT, PT and TT: 1. Blood is collected into a vacutainer tube containing 0.105M Na-citrate as an anticoagulant. Citrate prevents coagulation by forming complex with calcium that is required for some steps in the clotting process. Clotting test should be performed within 2 hours following blood collection. 2. To obtain platelet poor plasma, samples should be centrifuged at 3000/min (1500g) for 15 minutes at room temperature. After centrifugation, the upper 3/4 of platelet poor plasma should be transferred into plastic tubes. 3. Magnetic sensor coagulometer (we use the KC-1 in our lab): a cup with a steel ball is rotated in a heated compartment of the coagulometer and the ball is held on its place by a magnet. The plasma and reagents are pipetted into this cup. The timer of the instrument is started by the addition of the starting reagent with an automatic pipette connected to the instrument. The clot formed during coagulation pulls the steel ball out of place, and this displacement detected by a magnetic sensor. The timer then stops, and the clotting time is recorded.

For fibrin monomer test and D-dimer test, see Topic 22, as part of DIC panel.

Avi Sayag

Clinical Biochemistry

Topic 19
Platelet function disorders
Normal blood flow is laminar, such that platelets flow centrally in the vessel lumen, separated from the endothelium by a slower moving clear zone of plasma. vWF gene is located on chromosome 22. The product (vWF) is synthesized and stored in endothelial cells (Weibel Palade bodies), in megakaryocytes (synthesis) and platelets (storages in -granules) and is also found in the ECM (subendothelium) with collagen fibers. The half-life of vWF is 12-20 hours and ADAMTS13 inhibits vWF by cleaving it. The functions of vWF are: Supports platelets adhesion to collagen and other subendothelial structures, especially at high-shear rates. The platelets bind via the GpIb receptors to the vWF located in the subendothelium and exposed at endothelial injuries. Supports platelets aggregation at high-shear rates. Associates with FVIII in the circulation and protects it from degradation (it also promotes the secretion of FVIII). Von Willebrand Disease An autosomally (dominant mostly) inherited disorder of platelet adhesion, with reduced amount/function of vWF. The disease is marked by spontaneous bleeding from mucous membranes, excessive bleeding from wounds, menorrhagia, and a prolonged bleeding time in the presence of a normal platelet count. Individuals with von Willebrand disease have a compound defect involving platelet function and the coagulation pathway. The amounts of factor VIII are only moderately depressed, and it is the defect in platelet function that dominates the clinical picture. Lab: 1. Anamnesis with focus on bleeding time (should be prolonged in vWD) 2. PFA-100 (Platelet Function Analyzer): The PFA-100 test is a new in vitro test of platelet function. The test measures the time taken for blood, drawn through a fine capillary, to block a membrane coated with collagen and epinephrine (CEPI) or collagen and ADP (CADP). This is referred to as the Closure Time (CT) and is measured in seconds. The test is therefore a combined measure of platelet adhesion and aggregation. If CEPI is <180 seconds - normal platelet function. A normal CEPI value excludes the presence of a significant platelet function defect. CEPI >180 s; CADP <116 s (normal value) - "Aspirin Effect". If this result is normal, the most likely explanation is that the patient has ingested aspirin or similar medication. CEPI >180 s; CADP >116 seconds - abnormal platelet function. However, thrombocytopenia (<100K) and anemia (Hct <0.28) should first be excluded. 3. Platelet count (normal values are 150-400 G/L) 4. APTT 5. RIPA (Ristocetin Induced Platelet Aggregation): Ristocetin induces the binding of vWF to platelets, and induces their aggregation. 6. vWF:Ag this test measures the antigenic characteristics of vWF (measured by immunoassay). 7. vWF:RCo (Ristocetin cofactor) this test measures the binding capacity of plasma vWF to platelets in the presence of Ristocetin (it is measured by agglutination of isolated platelets). 8. vWF:CB this test measures the binding capacity of plasma vWF to collagen. 9. vWF:FVIII this test measures the binding capacity of vWF to FVIII 10. Examining the structure of the multimeric vWF (by SDS-agarose elctrophoresis) 11. vWF gene analysis (chromosome 22)

Avi Sayag

Clinical Biochemistry

Types of vWF disease: Type I: partial quantitative deficiency in vWF. Autosomal dominant (AD). The multimer distribution is normal, but there is a reduced amount of vWF:Ag and FVIII. In type IIA, the high-molecular-weight multimers (the largest) are not synthesized, leading to a true deficiency. Thus, the activity of vWF:Rco is reduced. AD. In type IIB, functionally abnormal high-molecular-weight multimers are synthesized but are rapidly removed from the circulation. These high-molecular-weight multimers cause spontaneous platelet aggregation (a situation reminiscent of the very-high-molecular-weight multimer aggregates that are seen in TTP), and indeed some individuals with type IIB von Willebrand disease have chronic mild thrombocytopenia that is presumably caused by platelet consumption. AD. The vWF:Rco is normal but the RIPA may be enhanced. Type IIM: vWF defect associated with specific defects in platelet/vWF interaction, but with a normal range of multimers. AD. Type IIN: vWF defect resulting from defective vWF binding to FVIII. This leads to low levels of FVIII. AR. Type III: severe quantitative disorder resulting from a huge reduction in or absence of vWF in the plasma and in the platelets (less than 5%). FVIII will also be reduced. AR.

Individuals with blood group O have lower levels of vWF than those with blood group A, B and AB (more than 65% of patients with vWD have blood group O). In some cases, the disease can be acquired due to: 1. Reduced rate of synthesis; 2. Increased rate of clearance from the circulation (might result in abnormal multimeric structure); 3. Antibodies directed against vWF (that might be inhibitory or non-inhibitory) The acquired form may accompany several diseases: 1. Hypothyrodism (like in type I vWD); 2. Wilms tumors (tumors of the kidneys); 3. Congenital cardiovascular defects (in case of aortic valve stenosis it may lead to GI bleeding Heyde's syndrome); 4. Absorption on certain tumors; 5. Autoimmune vWD.

Avi Sayag

Clinical Biochemistry

Topic 20
Inherited thrombophilias
Thrombophilia is the propensity to develop thrombosis (blood clots) due to an abnormality in the system of coagulation. Hereditary defects in one or more of the clotting factors can cause the formation of potentially dangerous thrombosis. The most common symptoms of thrombophilia are DVT at a relatively young age (<55 years of age) Pulmonary embolism Thrombosis at an unusual location (not in the lower limbs) Recurrent thrombosis Familial occurrence (inherited thrombophilias) Causes of inherited thrombophilias: 1. Antithrombin III deficiency 2. Protein C deficiency 3. Protein S deficiency 4. Prothrombin mutation (20210A allele) 5. Elevated factor VIII 6. Activated protein C resistance (FV Leiden) the most common cause 7. Dysfibrinogenemia Antithrombin III deficiency The physiological target proteases of antithrombin are those of the intrinsic pathway: Xa, IXa, XIa, XIIa and thrombin, and also VIIa from the extrinsic pathway. The incidence of inherited antithrombin deficiency has been estimated to be between 1:2000 and 1:5000 in the normal population. Maintenance of an adequate level of antithrombin activity, which is at least 70% that of a normal functional level, is essential to ensure effective inhibition of blood coagulation proteases. Typically as a result of type I or type II antithrombin deficiency, functional antithrombin levels are reduced to below 50% of normal. Type I antithrombin deficiency is characterized by a decrease in both antithrombin activity and antithrombin concentration in the blood of affected individuals. Most cases of type I deficiency are due to point mutations, deletions or minor insertions within the antithrombin gene. Type II antithrombin deficiency is characterized by normal antithrombin levels but reduced antithrombin activity in the blood of affected individuals. Diagnosis: antithrombin III activity should be measured first. If low, then antithrombin antigen is measured to look for mutations consistent with type II disease. Protein C deficiency The prevalence of protein C deficiency has been estimated to be about 0.2% to 0.5% of the general population (that is, between 1:200 and 1:500). Protein C deficiency is associated with an increased incidence of venous thromboembolism (relative risk 8-10), whereas no association with arterial thrombotic disease has been found. The main function of protein C is its anticoagulant property as an inhibitor of coagulation factors V and VIII. There are two main types of protein C mutations that lead to protein C deficiency: Type I: Quantitative defects of protein C (low production or short protein half life) Type II: Qualitative defects, in which interaction with other molecules is abnormal. Defects in interaction with thrombomodulin, phospholipids, factors V/VIII and others have been described. The majority of people with protein C deficiency lack only one of the functioning genes, and are therefore heterozygous. Protein S deficiency Protein S, a vitamin K-dependent physiological anticoagulant, acts as a nonenzymatic cofactor to the activated protein C in the proteolytic degradation of factor Va and factor VIIIa.

Avi Sayag

Clinical Biochemistry

Decreased (antigen) levels or impaired function (activity) of protein S, leads to decreased degradation of factor Va and factor VIIIa and an increased propensity to venous thrombosis. In healthy individuals, approximately 30-40% of total protein S is in the free state. Only free protein S is capable of acting as a cofactor in the protein C system. There are three types of hereditary protein S deficiency: Type I - decreased protein S activity: decreased total protein S (=both bound and free protein S) levels AND decreased free protein S levels (quantitative defect) Type II - decreased protein S activity: normal free protein S levels AND decreased total protein S levels (qualitative defect) Type III - decreased protein S activity: decreased free protein S levels AND normal total protein S levels (quantitative defect) Protein S deficiency can also be acquired due to vitamin K deficiency or treatment with warfarin, systemic sex hormone therapy and pregnancy, liver disease, and certain chronic infections (for example HIV). Vitamin K deficiency or treatment with warfarin generally also impairs the coagulation system itself (factors II, VII, IX and X), and therefore predisposes to bleeding rather than thrombosis. Protein S deficiency is the underlying cause of a small proportion of cases of DIC, DVT and pulmonary embolism. Hereditary protein S deficiency is an autosomal dominant condition. Prothrombin 20210A mutation This mutation in the gene encoding the clotting factor prothrombin is found in about 1 in 50 persons in the US. The mutation gives rise to slowed mRNA degradation and to an increase in circulating prothrombin levels. This appears to create a hypercoagulable state. The mutation is inherited in an autosomal dominant manner. Testing for prothrombin mutation G20210A is therefore useful in determining a person's predisposition to thrombosis and can assist in determining the need for anticoagulant therapy. People who have prothrombin mutation G20210A have a 2-to-3 fold increase in the risk of DVT. Persons who have this mutation plus the Factor V Leiden mutation have a 10-to-20 fold increase in thrombotic risk. Prothrombin mutations have also been linked with thrombotic events other than DVT, including recurrent miscarriages. In addition, there may be interactions with other risk factors for thrombosis (e.g. pregnancy, oral contraceptives). Activated Protein C Resistance (FV Leiden) Factor V Leiden is the name given to a variant of human factor V that causes a hypercoagulability disorder in 1:20 Caucasians in N. America. In this disorder factor V and FVIII cannot be inactivated by activated protein C due to structural changes in these factors. It is named after the city Leiden (The Netherlands), where it was first identified in 1994. Diagnosis: APC added to the plasma degrades these factors and prolongs clotting time tests that include these factors. In case of APC resistance, structural changes in factor V (and rarely in FVIII) prevent the effect of APC and the prolongation of the clotting test becomes more moderate. FV is a cofactor of FX that converts prothrombin to thrombin. FV is activated by thrombin and inactivated by protein C. Protein C cleaves FV at Arg306, Arg506 and Arg679. In FV Leiden mutation guanine exchanges adenine at nucleotide 1691 resulting in Arg506Gln. The test sample is Na-citrate anticoagulated plasma. The patient's plasma should be diluted 5fold by FV deficient plasma. The samples are tested for APTT in the presence and absence of APC. The 2 samples are first incubated for 5 minutes (after diluting them). Calcium is added to both samples, but one gets APC and the other one does not. APTT is measured for both samples. The normal ratio should be > 2. A ratio < 2.0 suggests APC resistance (sample with APC divided by the sample without APC). Molecular genetic test for FV Leiden mutation DNA is prepared from peripheral WBCs. The exon containing the FV Leiden mutation is amplified by PCR, and the products are digested by specific restriction endonucleases. The digestion products are separated by agarose gel electrophoresis. Interpretation: when Leiden mutation is present, the restriction endonuclease loses one of its cleavage sites on the PCR product. (Note: APC resistance is measured by APTT-based assay, while Leiden mutaion is based on a molecular genetic method).

Avi Sayag

Clinical Biochemistry

Topic 21
Acquired thrombophilias
Acquired thrombophilias can occur in the settings of: Post-operative conditions Pregnancy, estrogen therapy, pills (oral contraceptives) Prosthetic valves Anti-phospholipid syndrome Atrial fibrillation Immobilization Initial phase of syncumar therapy Heparin-induced thrombosis Varicose veins Malignancy Thrombus and malignancy In cancer patients, thromboembolism is the second most frequent cause of death. Thrombosis and embolism may indicate occult malignancy. Malignant diseases are frequently associated with thromboembolic complications o Promyelocytic leukemia o Primary cerebral tumors o Pancreas carcinoma (mucin-secreting adenocarcinoma) Hypercoagulability in cancer patients results from the response of the host (monocytes/macrophages are stimulated by tumor cells, which provoke a procoagulant activity) as well as from features of malignant cells: they express tissue factor and platelet activating substances. Thrombosis promoting factors in cancer patients: o Surgical intervention o Aggressive chemotherapy o Hormone therapy o Permanent central venous catheter Anti-phospholipid syndrome (APS) An autoimmune condition in which a group of autoAb (antiphospholipid) play a direct role in the pathogenesis of thrombosis, fetal loss and other symptoms. APS can be primary or secondary (related to SLE, other autoimmune diseases, neoplasias, etc.) Clinical criteria for APS: o Vascular thrombosis: one or more episodes of arterial/venous/small vessel thrombosis confirmed by imaging or Doppler (or histopathology) o Pregnancy morbidity: 3 or more unexplained consecutive miscarriages or 1 or more unexplained fetal deaths at the 10th week of gestation or afterwards. One or more premature births at week 34 or before, associated with severe preeclampsia or placental insufficiency. Lab: o Lupus anticoagulant (LA): present in the plasma at 2 or more occasions, at least 12 weeks apart, and detected according to the guidelines of the ISTH (International Society on Thrombosis and Haemostasis). o Anticardiolipin Ab (ACA): IgG and/or IgM isotypes present in medium or high titer at 2 or more occasions, at least 12 weeks apart, and measured by a standardized ELISA for 2 glycoprotein I-dependent ACA. o Anti-2-glycoprotein-I-Ab: IgG and/or IgM isotypes present in medium or high titer at 2 or more occasions, at least 12 weeks apart, and measured by a standardized ELISA. LA or other AP-Ab are directed against epitopes on certain proteins, most commonly against 2-glycoprotein I and prothrombin. The epitopes become exposed upon binding to anionic phospholipids or negatively-charged surfaces.

Avi Sayag

Clinical Biochemistry

LA: these are immunoglobulins (IgG and/or IgM) that interfere with phospholipiddependent clotting tests. They are directed against phospholipid-protein complexes (epitope on the protein) but not against clotting factors. In vitro, they are anticoagulants, but in vivo they may predispose for thrombosis. Lab diagnosis of LA: o Prolongation of APTT o Demonstration of an inhibitor by mixing studies. The inhibitor has to be demonstrated to be phospholipid-dependent by: Relative correction by increasing the amount of phospholipids The effect is increased by diluting the phospholipid. Mechanism of thrombophilias induced by AP-Ab: o Inhibition of protein C/protein S pathway o Interference with the synthesis of PgI2 (aka prostacyclin a potent inhibitor of platelet aggregation and a powerful vasodilator) o Inhibition of the binding of heparan-sulphate with AT-III on the surface of endothelial cells o Induction of the expression of tissue factor in endothelial cells o Involvement in Ab-induced platelet activation.

Avi Sayag

Clinical Biochemistry

Topic 22
Consumption coagulopathies. DIC
There are 4 consumption coagulopathies: 1. Acute DIC; 2. Large thrombus; 3. Primary hyperfibrinolysis; and 4. Thrombolysis The bleeding symptom is due to the consumption of clotting factors. Disseminated Intravascular Coagulation An acute, subacute, or chronic thrombohemorrhagic disorder. DIC occurs as a secondary complication in a variety of diseases. It is caused by the systemic activation of the coagulation pathways, leading to the formation of thrombi throughout the microcirculation. As a consequence of the widespread thromboses, there is consumption of platelets and coagulation factors and, secondarily, activation of fibrinolysis. Thus, DIC can give rise to: either tissue hypoxia and microinfarcts caused by many microthrombi; or a bleeding disorder related to o Pathologic activation of fibrinolysis o The depletion of the elements required for hemostasis (hence the term consumptive coagulopathy). This is probably a more common cause of bleeding than all of the congenital coagulation disorders all together. Two major mechanisms can trigger DIC: the release of TF and massive endothelial damage. Release of tissue factor The cytoplasmic Mucin-secreting granules of acute adenocarcinoma promyelocytic cells. Some leukemia cells: tumors express treatment leads to tissue factor on destruction of the cell granulocytes membrane proteolytic enzymes (mainly lung are released adenocarcinoma vessel damage and pancreatic TF is exposed. adenocarcinoma)

The placenta in obstetric complications; amniotic fluid embolism; abruptio placentae (wherein the placental lining has separated from the uterus of the mother. Trauma, hypertension, or coagulopathy, contributes to the avulsion of the anchoring placental villi from the expanding lower uterine segment, which, in turn, leads to bleeding into the decidua basalis. This can push the placenta away from the uterus and cause further bleeding); dead fetus syndrome; septic abortion;

In gram-negative and gram-positive sepsis (important causes of DIC), endotoxins or exotoxins cause increased synthesis, surface expression, and release of tissue factor from monocytes. Furthermore, activated monocytes release IL-1 and TNF, both increase the expression of tissue factor on endothelial cells and simultaneously decrease the expression of thrombomodulin. The net result is the enhanced activation of the extrinsic clotting system and the decrease of inhibitory pathways that tend to prevent coagulation (protein C and S).

Avi Sayag

Clinical Biochemistry

Widespread endothelial cell damage Severe endothelial cell injury can initiate DIC by causing the release of tissue factor and by exposing subendothelial collagen and von Willebrand factor (vWF), which act together to promote platelet aggregation and the activation of the intrinsic coagulation cascade. Widespread endothelial injury can be produced by: 1. The deposition of antigen-antibody complexes (e.g., in SLE), 2. By temperature extremes (e.g., following heat stroke or burns), or 3. By infections (e.g., meningococci and rickettsiae). Endothelial injury is an important consequence of endotoxemia; therefore, DIC is a frequent complication of gram-negative sepsis.

DIC has two phases: The thrombotic phase: Fibrin deposition

Obstructed vessels ischemia

Hemolysis of RBCs (RBCs are traumatized in passing though narrowed vessels)

The hemorrhagic phase:

Depletion of PLTs

Release of plasminogen activators

Fibrinolysis (FDP produced) FV and FVIII are cleaved by plasminogen Inhibit PLT aggregation Antithrombin activity Impair fibrin polymerization

Consumption of factors
Lab

1. INR and PT: a high INR level such as INR=5 indicates that there is a high chance of bleeding, whereas if the INR=0.5 then there is a high chance of having a clot. Normal range for a healthy person is 2-3, and for people on warfarin therapy or prosthetic valves: 2.53.5. PT: 8-12 seconds (+ 4 seconds prolonged) 2. APTT: 28-40 seconds (+8 seconds prolonged). 3. Platelet count (reveals thrombocytopenia. Range: 150-400 G/L); 4. Soluble fibrin test: fibrin monomers and oligomers at low concentrations form complexes with fibrinogen and remain soluble. The advantage of measuring soluble fibrin over fibrinopeptide A to detect thrombin action on fibrinogen is the considerably longer half-life of soluble fibrin in the circulation.

Avi Sayag

Clinical Biochemistry

5. D-dimers: these are products of fibrin degradation (among other fibrin degradation products FDPs). Their detection is made possible using monoclonal antibodies that do not cross react with fibrinogen. 6. TT: 14-22 seconds (+ 8 seconds prolonged) 7. Fragmentocytes (aka schistocytes) 8. Tests indicating the consumption of antithrombin III (ATIII). 9. Other markers for the intravascular activation of blood coagulation are the prothrombin fragments 1 and 2, the fibrinopeptide A and the thrombin-antithrombin (TAT complex). As prothrombin is cleaved it forms thrombin and the remaining is 2 fragments. Thrombin formation results in irreversible TAT complex. Fibrinopeptide A is cleaved off by thrombin from the N-terminal end of A-chain of the fibrinogen (the products of this cleavage are FPA, FPB and fibrin monomers). The first 6 comprise the DIC panel and the last 3 complete the diagnostic procedure. Primary hyperfibrinolysis Fibrinolysis is responsible for fibrin breakdown. Hyperfibrinolysis occurs when fibrinolytic activity is greater than fibrin formation such that clot integrity is threatened. The central event of fibrinolysis is the generation of plasmin, which cleaves fibrin and fibrinogen. Free plasmin is rapidly inhibited by its inhibitor 2 antiplasmin. Fibrinolytic activity is initiated by the plasminogen activators: t-PA and u-PA, which convert plasminogen to plasmin. t-PA is released by endothelial cells, has a short half-life of 3-5 minutes and is regulated by specific inhibitors: PAI (plasminogen activator inhibitor) types 1 and 2. PAI-1 is the main systemic inhibitor and is produced by several cell types including endothelial cells, smooth muscle cells, fibroblasts and hepatocytes. PAI-2 is found in the placenta. Platelets are the source of 90% of the circulating PAI-1, which is released at the site of a forming thrombus. Hyperfibrinolysis occurs in the setting of imbalance between fibrinolytic activators and their inhibitors. The consequences of hyperfibrinolysis affect other aspects of hemostasis. Plasmin may reduce platelet adhesion and aggregation by degradation of GpIb receptors and IIb/IIIa receptors. The consumption of the clotting factors due to the direct effect of plasmin and the formation of fibrinogen degradation products, which inhibit fibrin polymerization, results in poor fibrin generation. Fibrinolytic activation has been separated into primary and secondary: the primary form represents fibrinolytic activity independent of other factors, whereas the secondary form is a consequence of activation of coagulation and thus thrombin generation which stimulates the endothelium to produce increased amount of t-PA. Chronic liver disease is a common cause of hyperfibrinolysis, and is characterized by both primary and secondary hyperfibrinolytic changes. There is reduced clearance of t-PA, and reduced concentrations of 2-antiplasmin due to diminished protein synthesis. In addition, primary hyperfibrinolysis can occur in the setting of prostate cancer and neoplasia as tumors produce plasminogen activators (AML M3 tumors produce uPA). Among the lab marker specified for DIC, only fibrin degradation product (FDP) is positive.

Detection of fibrin monomers (topic 13 in the practical topics)


Fibrin monomers are coupled to human RBCs (type O, Rh negative) in the reagent. In the presence of soluble fibrin monomer complexes in the test plasma, hemagglutination will occur. Positive and negative fibrin monomer controls are supplied by the manufacturer. The method is fast, simple and specific.

Avi Sayag

Clinical Biochemistry

Detection of D-dimers (topic 12 in the practical topics)


In coagulation, the activated factor XIII crosslinks fibrin, which results in high molecular weight crosslinked fibrin polymers. The secondary fibrinolysis degrades fibrin. Fibrinogen is cleaved to D and E fragments. If fibrin was cross-linked before fibrinolysis, the products of fibrinolysis are E fragment and D-dimer. The reagent contains latex conjugated with specific antibodies directed against the D-dimer domain of the cross-linked fibrin. In the presence of excess amount of D-dimers, the latex beads will agglutinate and the test is considered positive. The D-dimer test detects only the presence of split products of fibrin cross-linked by FXIII. Thus, the D-dimer test is positive only in secondary hyperfibrinolysis. If the sample is agglutinated, the D-dimer level is said to be > 2mg/L. In normal conditions, agglutination cannot be detected and the D-dimer is said to be < 0.25mg/L. In the clinical practice, the only acceptable method for D-dimer measurement is the quantitative D-dimer determination where the agglutination is evaluated by automated methods. The presence of rheumatoid factor (anti-human IgG) can lead to false positive results.

FX Prothrombin Prothrombin fragments 1 & 2 + thrombin TAT

Fibrinogen

FPA, FPB, fibrin monomers

Avi Sayag

Clinical Biochemistry

Topic 23
Laboratory tests of glomerular and tubular function
Assessing renal function, we have to test 3 aspects: 1. The glomerular function (the glomerular filtration and integrity); 2. The renal tubular functions; and 3. The renal endocrine function. Estimation of the glomerular function The estimation consists of 4 parameters: 1. Clearance determinations 2. Plasma creatinine 3. Plasma urea 4. Plasma level of low molecular weight proteins: 2-microglobulins, retinol binding protein, cystatin C. We shall consider each of these: 1. Clearance Determinations We can use exogenous or endogenous substances that are completely filtered, not reabsorbed and not secreted in the tubules. Whichever substance chosen, we calculate its concentration in the urine collected, in the plasma collected, and we measure the urine output over 24 hours. In order to avoid problems, the collection has to be carried out twice. The exogenous substances are inulin and Cr-EDTA, and the endogenous one is creatinine. The formula for clearance is given by: C =

UxV . P

Creatinine clearance in adults is normally of the order of 120 mL/min, corrected to a standard body surface area of 1.73 m2. It should be noted that the clearance formula is only valid for a steady state, that is, when renal function is not changing rapidly. The accurate measurement of creatinine clearance is difficult, especially in outpatients, since it is necessary to obtain a complete and accurately timed sample of urine. The usual collection time is 24 h, but patients may forget the time or forget to include some urine in the collection. Creatinine is actively secreted by the renal tubules and, as a result, the creatinine clearance is higher than the true GFR (by 15%). The difference is of little significance when the GFR is normal, but when the GFR is low (<10 mL/min), tubular secretion makes a major contribution to creatinine excretion and creatinine clearance significantly overestimates the GFR. The effect of creatinine breakdown in the gut also becomes significant when the GFR is very low. Certain drugs, including spironolactone, cimetidine and amiloride, decrease creatinine secretion and thus can reduce creatinine clearance. Lastly, in the calculation of creatinine clearance, two measurements of creatinine concentration and one of urine volume are required. Each of these has an inherent imprecision that can affect the accuracy of the overall result. Even in wellmotivated subjects, studied under ideal conditions, the coefficient of variance of measurements of creatinine clearance can be as high as 10%, and it can be two or three times greater than this in ordinary patients. The Cockcroft-Gault formula gives the creatinine clearance:

(140 age) xKgxK where seCreatinine( mol / L)

K (constant) is 1.227 for men and 1.04 for women. As mentioned, the creatinine clearance estimates the GFR. However, the GFR can be estimated using the 4-variable Modification of Diet in Renal Disease study group, or in short, the 4-v MDRD formula: GFR= 186 x(0.0113Scr ) 1.154 x( age) 0.203 . The result should be multiplied by 0.742 if the patient is a female, and by 1.21 if the patient is black. Scr is serum creatinine given in mol/L. As implied by the name of the formula, the GFR is tested in renal disorders, and it is valid only at high plasma creatinine concentrations. Another estimation tool to calculate GFR is the Mayo Quadratic formula. This formula was developed in an attempt to better estimate GFR in patients with preserved kidney function. It is well recognized that the MDRD formula tends to underestimate GFR in patients with

Avi Sayag

Clinical Biochemistry

preserved kidney function. Notwithstanding, the formula estimates the GFR in patients suffering from renal disorders, and it is valid at low plasma creatinine concentrations (the quadratic formula). None of the formulae is valid for children under 18 years of age and for pregnant women! The reference range for GFR is 90-120 ml/min normalized to body surface of 1.73m2. GFR between 60-90 ml/min/1.73m2 suggests mild kidney disorder. GFR between 30-60 ml/min/1.73m2 suggests moderate kidney disorder. GFR between 15-30 ml/min/1.73m2 suggests severe kidney disorder. GFR < 15 ml/min/1.73m2 suggests end-stage kidney disorder.

2. Plasma Creatinine
Measurement of plasma creatinine concentration is a reliable test of glomerular function. Creatinine is a break-down product of creatine phosphate in muscle, and is usually produced at a fairly constant rate by the body (depending on muscle mass). Meat intake may increase creatinine serum level by 10%. It is not reabsorbed by the renal tubules, but a small amount is secreted. The reference range- for men: 62-106 mol/L; for women: 44-97 mol/L. The standard laboratory measurements for creatinine can suffer from interference, for example from bilirubin and ketones3. The laboratory should be able to advise on whether this may be a problem in individual cases. Several factors influence plasma creatinine concentration: It decreases with age and among females. It also decreases among vegetarians and in cases of malnutrition. Immediately after surgery and in patients treated with corticosteroids the concentration also decreases. It increases among blacks, in cooked meat, among athletes (increased muscular mass) and if certain medications are taken: creatinine levels may increase when ACE inhibitors (ACEI) or angiotensin-II receptor blockers (ARBs) are used in the treatment of chronic heart failure (CHF). Using both ACEI & ARB concomitantly will increase creatinine levels to a greater degree than either of the two drugs would individually. An increase of <30% is to be expected with ACEI or ARB use. Obesity does not affect creatinine concentration. 3. Plasma Urea Urea is synthesized in the liver, primarily as a by-product of the deamination of amino acids. Its elimination in the urine represents the major route for nitrogen excretion. It is filtered from the blood by the glomeruli (90%) but significant tubular reabsorption occurs through passive diffusion (neither active reabsorption nor secretion occurs). Plasma urea concentration is a less reliable indicator of renal glomerular function than creatinine. Urea production depends on non-renal factors! The reference interval is 2.9-8.2 mmol/L. Urea production is increased by a high protein intake, in catabolic states, and by the absorption of amino acids and peptides after gastrointestinal hemorrhage. Dehydration and urinary stasis also increase its levels. Conversely, production is decreased in patients with a low protein intake and sometimes in patients with liver disease. Tubular reabsorption increases at low rates of urine flow (e.g. in fluid depletion) and this can cause increased plasma urea concentration even when renal function is normal.

Bilirubin decreases creatinine and ketones increase creatinine.

Avi Sayag

Clinical Biochemistry

Lab methods for the determination of urea and creatinine (Practical topic 14)
Urea There are 2 methods to determine the urea concentration: 1. Direct chemical methods 2. Indirect enzymatic method 1. Direct chemical method This method is outdated because they cannot be automated. The direct chemical reaction of urea and special chemical substances results in the formation of colored products that can be measured by spectrophotometry. 2. Indirect enzymatic method The ammonia, formed after the reaction of urease on urea, is determined. Urea + H2O ------------> 2NH4+ + CO2 (carried by urease) From this point, there are 3 ways to determine urea concentration: - Enzymatic UV kinetic method: 2NH4+ + 2-ketoglutarate + 2NADH ---------> 2 glutamate + 2NAD + 2H2O From ammonia and -ketoglutarate, in the presence of NADH and under the effect of glutamate dehydrogenase, NAD and glutamate are formed. The decrease in the amount of NADH correlates with the amount of ammonia formed. - Conductometric method: The amount of ammonia formed from urea can be detected by the change of conductivity in the solution. - Ionselective electrode: The changes in the electrode potential are proportional to the ammonia concentration (ammonium ions). Creatinine There are 2 methods: 1. Methods based on Jaffe reaction 2. Enzymatic methods 1. Methods based on Jaffe reaction Creatinine in alkaline solution gives an orange condensation product with picric acid. There are 2 types of reactions: End-point Jaffe reaction: one reaction with deproteinization and the other one is without deproteinization. In the first method we deproteinize the solution, because proteins interfere with the reaction. This makes the method laborious. The other method, in which we give up the deproteinization process, is considered outdated. Thus, the end-point Jaffe reaction is no longer practical. (In other words, in the oral exam, just mention the reaction and move on. In the written SCT, remember that it exists). Kinetic Jaffe reaction: the kinetic Jaffe reaction is a modified end-point Jaffe reaction. It is fast, automated, and measures the rate of color development. The Jaffe reaction is not specific for creatinine because other serum components (proteins, glucose, ascorbic acid and -ketoacids) also react with picric acid. The reaction rate in case of creatinine and interfering substances is different, so prior deproteinization is not necessary. A part of non-specific reactions are completed within 30 seconds, others only cause interference after 2 minutes. The change in absorbance after 30 seconds to 2 minutes is mainly caused by creatinine. In case of diabetic ketoacidosis, falsely high creatinine concentration can be detected due to ketone bodies, whereas in icteric patients (jaundice) bilirubin and its metabolites may cause falsely low results.

Avi Sayag

Clinical Biochemistry

2. Enzymatic methods Here, too, there are 2 options: Partial enzymatic method: creatinine is degraded under the effect of creatinine aminohydrolase producing ammonia. The ammonia can be detected by ionselective electrode. The other possibility is to measure the amount of chromogens from Jaffe reaction before and after the enzyme reaction. The difference will be proportional to the creatinine concentration. Completely enzymatic UV method: creatinine is converted to creatine under the effect of creatine hydrolases, which can be detected by creatine kinase.

(cont. of topic 23:) 4. Cystatin C


This low molecular weight peptide (13 kDa) is produced by all nucleated cells. It is a cysteine protease inhibitor. Due to its low Mw and high isoelectric point (9.2), it is cleared from the plasma by glomerular filtration only and its plasma concentration reflects the GFR. However, its plasma concentration is more variable than that of creatinine; it is increased in malignancy and by treatment with corticosteroids. Although advocated as being a more sensitive and specific indicator of moderately impaired renal function than creatinine, measurement of cystatin C does not at present have a clear role in the assessment of patients with suspected renal impairment

Assessment of glomerular integrity


Impairment of glomerular integrity results in the filtration of large molecules that are normally retained and it manifests as proteinuria. 'Clinical proteinuria' is proteinuria that can be reliably detected by dipstick testing of urine, and is >300 mg/L. With severe glomerular damage, RBCs are detectable in the urine (hematuria). While hematuria can occur as a result of lesions anywhere in the urinary tract, the RBCs often have an abnormal morphology in glomerular disease. The presence of red cell casts (cells embedded in a proteinaceous matrix) in urinary sediment is strongly suggestive of glomerular dysfunction. Between 7-10 g of proteins are filtered per day; however, only less than 150 mg is excreted! (thus, most is reabsorbed). Half of the excreted proteins is given by Tamm-Horsfall protein: a human gene. This gene encodes uromodulin, the most abundant protein in normal urine. Uromodulin may act as a constitutive inhibitor of calcium crystallization in renal fluids. Excretion of uromodulin in urine may provide defense against urinary tract infections caused by uropathogenic bacteria. Defects in this gene are associated with the autosomal dominant renal disorders: medullary cystic kidney disease-2 (MCKD2) and familial juvenile hyperuricemic nephropathy (FJHN). Less than 30 mg of albumin is excreted. Proteinuria can result from 3 causes: 1. Increased filtered load; 2. Decreased tubular reabsorption; 3. Postglomerular secretion/leakage; We shall consider each cause separately: 1. Increased filtered load The increased load can be the result of: 1. Increased glomerular permeability there is a progressively increasing excretion of higher Mw proteins as the permeability increases; 2. Increased plasma concentration of a relatively freely-filtered proteins (such as BenceJones proteins and myoglobin); 3. Decreased number of glumeroli thus, there is an increased filtered load per nephron. 2. Decreased tubular reabsorption The decreased reabsorption can be the result of: 1. Damage to the proximal tubules the indication to such damage is the presence of low Mw proteins, such as 1-microglobulins in the urine;

Avi Sayag

Clinical Biochemistry

2. Enzymuria damage of the tubular epithelial cells results in increased cell turnover and cell lysis. Therefore, enzymes such as those in the brush border, in the cytosol and lysosomes are released. 3. Postglomerular secretion/leakage This can be the result of: 1. Increased protein secretion by the tubular system (Tamm-Horsfall glycoprotein) 2. Leakage of various plasma proteins into the urinary space as a result of tubulointerstitial damage (in cases of inflammation, e.g.). Proteinuria is screened by using dip-stick, which detects albumin sensitively (>200 mg/L), and is less sensitive to other proteins such as Bence Jones protein. False positive reactions can be obtained in alkaline urine and if X-ray contrast medium is used. External causes of proteinuria should be excluded (fever, strenuous exercise, orthostatic proteinuria). The source of proteinuria can be tubular damage or glomerular damage. If the source is tubular, then low Mw proteins will be detected in the urine (the high Mw ones will not be filtered in the glomerulus, as it is intact). In particular, retinol binding protein and 1microglobulin will be present. If the glomerulus is the source of the proteinuria, then higher Mw proteins will also be detected in the urine. Hematuria can be the manifestation of glomerular disease, tubulointerstitial disease or postrenal disease. It can manifest in the presence of RBCs in the urine, which are detected as sediments seen by phase-contrast microscopy, or in hemoglobinuria, which is detected by chemical methods that detect Hb. Estimation of the renal tubular function 1. Detection of low-Mw molecules (1-microglobin and retinol binding protein). 2. Assessment of renal concentrating ability: the osmolality of the urine, or its specific gravity, is measured following water deprivation. 3. Tests for renal tubular acidosis: the excretion of fractional bicarbonate is measured, and the ammonium chloride loading test is performed: the acid loading test (pH) measures the ability of the kidney tubules to acidify urine when there is increased plasma acidity. The patient is told to take ammonium chloride capsules by mouth for 3 days. Then, urine and blood samples are taken. (The blood sample is needed to show that the ammonium chloride made the blood slightly acidic.) The laboratory measures the level of acid found in both samples. 4. Tests for aminoaciduria: amino acids are measured using the HPLC method 5. Measurement of glucose in the plasma and urine (renal glucosuria) Urinalysis what can be detected using the dipstick method? (check this website for clear details: http://www.irvingcrowley.com/cls/urin.htm)

Avi Sayag

Clinical Biochemistry

Examination of urine (general and sediment analysis) (Practical topic 15)


1. Types of samples: random, first-morning urine (the most preferred sample. Concentrated, acidic, enhances the stability of the cellular elements), collected (24 hours, day, night. Urine sediment analysis must not be performed from collected sample!) 2. Collection of urine: mid-stream specimen. Collection via a catheter, collection bags (infants, children), collection via urostomy4. In rare cases, suprapubical aspiration is performed (when other techniques are not applicable). 3. Storage: urine examination and urine sediment analysis are better completed within 30 minutes. Urine samples can be kept at 4C (refrigerator) for maximum 2 hours. Urine examination by test strips The samples should be first examined macroscopically. The color of urine sampled from a healthy person is yellowish. For urine analysis by test strips, fresh (not older than 2 hours), uncentrifuged, carefully mixed samples have to be used. Submerge the sample for 1-2 seconds and remove the unnecessary urine off the strip. After 1-2 minutes, the color reactions can be visually evaluated. Remember that vitamin C (ascorbic acid) and certain drugs (captopril, phenazopyridin) can alter the results (inquire the patient about these drugs). The reactions on the urine test strips are based on dry chemical principles (the reagents are impregnated into a membrane attached to a plastic base). The parameters measured: 1. Specific gravity: detects the ion concentration of the urine (in the presence of cations, protons are released through a complex formation reaction, which lead to a change in color of the bromothymicblue indicator). Reference range: 1.005-1.03 2. pH: the indicators are methyl-red, phenophtalein and bromothymicblue. Reference range: 5-6 (higher values are recorded in vegetarians and after a long period of sample standing). 3. WBC: the esterase enzyme (present in granulocytes) cleaves indoxylester into indoxyl, which forms a purple color with a diazonium acid. The healthy urine does not contain enough WBCs to induce this color reaction (if the sample is not fresh, we may get a false positive result, because the esterase from leukocytes may pass out of the cells). 4. Nitrite: the most pathogenic bacteria (e.g. E. coli) in the urine reduce nitrate into nitrite, which can be visualized by the Griess probe (false negative results in the presence of vitamin C; false positive in the presence of phenazopyridine an antiinflammatory drug). 5. Proteins: the test strip contains bromophenolblue indicator set to an acidic pH by a buffer. When there are no proteins, the color of the indicator at pH 3 is yellow, and in the presence of proteins, the color changes into greenish-blue, depending on the type of the protein and its concentration. The indicator is much more sensitive to albumin than to globulin. 6. Glucose: detected by glucoseoxidase/peroxidase/chromogen-sulphate method. In healthy sample glucose is not detectable. 7. Ketone bodies: acetone and acetoacetate can be detected but not -hydroxybutyrate (this is kind of problematic if ketoacidosis is suspected, because -hydroxybutyrate is the most abundant ketone body in this condition).

A urostomy is a stoma (artificial opening) for the urinary system. A urostomy is made in cases where long-term drainage of urine through the bladder and urethra is not possible, e.g. after extensive surgery or in case of obstruction.

Avi Sayag

Clinical Biochemistry

8. Urobilinogen: it is important to keep the sample at dark to avoid the effect of direct sunshine that can yield false negative results. If UBG is present in high concentration, it can be observed macroscopically by the yellowish-brown color. Chemically, the presence of UBG gives a red color due to the formation of diazonium salt. 9. Bilirubin: the detection is based on the transformation of diazonium salt into a colored substance. As bilirubin dissociates by light, the sample has to be kept at dark prior to the test. 10. Blood: Hb (and myoglobin) catalyze the oxidation of tetramethylbenzidine. If the sample stands for too long before analysis, false positive results may be obtained due to hemolysis. Urine sediment analysis 1. Microscopic evaluation of urine sediment: 10mL of urine is put into a tube, centrifuged for 5 minutes, the supernatant is suctioned, the sediment is stirred, pipetted onto a slide, covered by a coverslip and examined under the microscope. 2. Analysis of sediment under the microscope: evaluate 10 low-power fields (count the casts); evaluate 10-20 high-power fields (count all cells and other elements). Reference range: 1 isomorphic RBC, 1-2 leukocytes, 1 hyaline cylinder in 1 HPF, 1 superficial urothelium per 4-5 HPF. 3. The analysis can be automated: in one class of analyzers, the cellular components flow by an optical lens. The other types of analyzers are based on a flow cytometry principle: the cellular elements are labeled with 2 types of fluorescent dyes by the machine one labels nucleic acids and the other labels the negatively-charged cell membrane, the nuclear membrane and the mitochondria. After hydrodynamic focusing, we measure the impedance of the cellular components, and a laser beam determines the scattering and fluorescent parameters of each element. According to this data, the computer of the analyzer can identify the right type of the cellular components, and can calculate the number of these elements in 1 L of urine. 4. Components of the urine sediment: there are organic components, inorganic components and artifacts/contaminating substances Organic components
RBCs 4-7m Isomorphic cells indicate postrenal bleeding Acantocytes suggest glomerular disease Ghost cells (lysed cells) DD: calcium-oxalate monohydrate crystals, bubbles, fat drops and yeast (2% acetic acid does not lyse yeast particles) Cellular elements WBCs Mainly PMNs, but all can be present Usually form clumps Usually associated with infections May be due to contamination Epithelium Tubular epithelium larger than WBCs (13m) Can appear during fever, ATN, interstitial nephritis, acute rejection of kidney transplantation Urothelium large (30m): can be: Superficial type: oval with smaller nucleus Deeper type: smaller cells Squamous epithelium: large (50-60m) usually suggests contamination

Casts
Hyaline casts contain only the base matrix of Tamm-Horsfall protein. Barely visible. Granular casts lysosomes of tubular cells or degraded cells (acute renal disease) Waxy casts chronic renal disease (sharp edges with indented borders) Fatty casts contain lipid particles (nephrosis) Cellular casts RBCs (renal parenchymal bleeding), WBCs (acute pyelonephritis, acute interstitial nephritis), tubular epithelial cells (glomerular disease) Hb and myoglobin casts renal parenchymal bleeding Bacterial and yeast casts among immunocompromised patients with pyelonephritis

Avi Sayag

Clinical Biochemistry
Microorganisms

Bacteria: if they are associated with high WBC count TB Yeasts mainly candida Trichomonas vaginalis Parasites

infection; if WBCs are present without bacteria

Inorganic components
Crystals Urine sediment analysis suggests the possibility of renal calculus only if the urine is examined right after urine passing and only if the repeated analysis of the sediment still shows crystals in bulk quantity. Cystine Uric acid Ca+2 oxalate Ca+2 oxalate Struvite (Mgmonohydrate NH4-Phosphate) Amorphous The major components are urates (in acidic urine) and phosphates (in alkaline urine). They appear as granular particles, often in clumps: "sedimentum lateritium"

Artifacts/contamination Vaginal floor Sperm Feces Hair Pollens Glass particles Bubbles Textile filaments

Avi Sayag

Clinical Biochemistry

Topic 24
Clinical biochemistry of acute & chronic renal failure (ARF CRF); tubulopathies
ARF is a rapid loss of renal function due to damage to the kidneys, resulting in retention of nitrogenous (urea and creatinine) and non-nitrogenous waste products that are normally excreted by the kidney. It is a serious disease and treated as a medical emergency. Signs of renal failure are: Symptoms manifest when the renal functional capacity decreases below 50-60% and the GFR falls below 50 ml/min. It thus follows that the kidneys have considerable reserve capacity! Uremia: manifests as nausea, vomiting and lethargy Disorders of micturition: frequency of urination increases, nocturia, retention of urine and dysuria (pain when urinating) Disorders of urine volume: polyuria, oliguria and anuria Alteration in urinary composition: hematuria, proteinuria, bacteriuria, leukocyturia and calculi Pain (not necessarily present) Edema: due to hypoalbuminemia and retention of salt and water ARF can be completely resolved, develop into CRF or lead to death in 50% of cases! ARF is usually categorized according to pre-renal, renal and post-renal causes: 1. Pre-renal ARF Pre-renal (causes in the blood supply): Hypovolemia: usually from shock, dehydration, fluid loss or excessive diuretics use, trauma, burns and surgery. When cardiac output is decreased (as in cardiogenic shock, in congestive heart failure and pulmonary embolism) it may result in ARF. Hepatorenal syndrome in which renal perfusion is compromised in liver failure Vascular problems, such as atheroembolic disease and renal vein thrombosis (which can occur as a complication of the nephrotic syndrome). The effective plasma volume is also decreased due to sepsis and shock. Infection: usually sepsis and systemic inflammation due to infection. When there is renal hypoperfusion, the GFR decreases. Parallelly, there is intense renal vasoconstriction that leads to redistribution of renal blood flow. This preserves tubular function. If hypoperfusion is left untreated, acute tubular necrosis develops. In case of hypovolemia, RBF decreases, which leads to increased levels of renin, angiotensin and aldosterone. Aldosterone increases tubular reabsorption of Na+, thus leading to decreased Na+ in the urine. Elevated levels of Na+ increase plasma osmolality, which activates the hypothalamus, leading to secretion of ADH from the neurohypophysis. There is water retention to correct for the hypovolemia, and small amount of concentrated urine is excreted. Regardless of this mechanism to correct for the hypovolemia, the decreased plasma volume results in decreased GFR. This leads to increased serum urea and creatinine. In addition, less sodium is delivered to the distal tubule (because less sodium is filtered and NOT because more is reabsorbed in the proximal tubule due to aldosterone secretion, as aldosterone acts on the distal tubule and the collecting ducts). Since less sodium reaches the distal tubule, less K+ and H+ are secreted (normally, in the distal tubule, sodium is reabsorbed actively and potassium and hydrogen ions are secreted passively). This leads to hyperkalemia and acidosis.

Avi Sayag

Clinical Biochemistry

2. Renal (intrinsic) ARF Toxins or medication (e.g. some NSAIDs, aminoglycoside antibiotics, cephalosporin, iodinated contrast, lithium, phosphate nephropathy due to bowel preparation for colonoscopy with sodium phosphates) Rhabdomyolysis - the resultant release of myoglobin in the blood affects the kidney; it can be caused by injury (especially crush injury and extensive blunt trauma), statins, stimulants and some other drugs. Hemolysis - the hemoglobin damages the tubules; it may be caused by various conditions such as sickle-cell disease, and lupus erythematosus. Multiple myeloma, either due to hypercalcemia or "cast nephropathy" (multiple myeloma can also cause chronic renal failure by a different mechanism). Acute glomerulonephritis which may be due to a variety of causes, such as anti glomerular basement membrane disease/Goodpasture's syndrome, Wegener's granulomatosis or acute lupus nephritis with SLE. Renal hypoperfusion: due to hypotension, bleeding and sepsis. Ischemia may result from low cardiac output, burns and trauma. Intrarenal obstruction: Bence-Jones proteinuria. Some features distinguish pre-renal ARF from renal ARF: 1. In prerenal ARF, the GFR is decreased while the tubules are preserved. In the renal form, tubular necrosis is frequent. Although glomerular damage is uncommon in intrinsic ARF, the GFR falls as a result of glomerular hypoperfusion. 2. In pre-renal ARF, urine [Na] is less than 20 mmol/L, while in the intrinsic form it rises above 40 mmol/L. 3. In pre-renal ARF, urine urea is concentrated by a factor of 20, that is, the urine-toplasma urea concentration is more than 20:1 (it doesn't contradict the fact that the plasma concentration of urea is elevated in pre-renal ARF). In the intrinsic form, the ratio is less than 10:1. 4. The urine-to-plasma osmolality ratio is more than 1.5:1 in the pre-renal form. In the intrinsic form, the ratio is less than 1.1:1. 5. In the intrinsic form, proteinuria is always present. There are typically three phases to the course of acute tubular necrosis (renal ARF): the initial oliguric phase, a diuretic phase and a recovery phase. The oliguric phase typically lasts for 810 days but sometimes is much shorter or persists for several weeks. When it occurs, the oliguric phase is followed by a diuretic phase, with increasing urine volume. This is the result of an increase in GFR, and initially there is often little improvement in tubular function. The composition of the urine is similar to that of protein-free plasma. During this phase, urine volume may exceed 5 L/day and, because of its high ionic concentration, there is a considerable risk of both dehydration and depletion of sodium and potassium. Although the onset of the diuretic phase often heralds clinical improvement, plasma concentrations of urea and creatinine do not fall immediately since the GFR is still much lower than normal and insufficient to allow excretion of the surplus. The persisting high urea concentration in the blood, and hence in the glomerular filtrate, contributes to the diuresis by an osmotic effect. The acidosis also persists until tubular function is restored. Plasma Ca+2 concentration may rise during this phase, particularly after crush injuries, owing to the release of Ca+2 from damaged muscles. Temporary persistence of any elevation in the plasma concentration of parathyroid hormone will stimulate calcitriol synthesis and this may also contribute to hypercalcaemia. Gradually, in the recovery phase, as the tubular cells regenerate and tubular function is restored, the diuresis subsides and the various abnormalities of renal function resolve. Patients who survive the acute illness usually recover completely. Some residual impairment of renal function is often demonstrable but it is not usually of clinical significance. Post-renal ARF In post-renal ARF the hydrostatic pressure increases and opposes glomerular filtration. If this state lasts for too long, it may lead to secondary renal tubular damage.

Avi Sayag

Clinical Biochemistry

Causes include: Medication interfering with normal bladder emptying (e.g. anticholinergics). Benign prostatic hypertrophy or prostate cancer. Kidney stones. Abdominal malignancy (e.g. ovarian cancer, colorectal cancer). Obstructed urinary catheter. Common to all these is the presence of any obstruction to bladder outflow or ureteric obstruction. In summary, in ARF there is an increase in K+, H+, urea, creatinine, phosphate, Mg+2 and uric acid, while there is a decrease in Na+, HCO3-, and Ca+2. Chronic Renal Failure (CRF) CRF is a progressive, irreversible loss of renal function over a period of months or years. The symptoms of worsening kidney function are unspecific, and might include feeling generally unwell and experiencing a reduced appetite. Often, CRF is diagnosed as a result of screening of people known to be at risk of kidney problems, such as those with hypertension or diabetes, glomerulonephritis and pyelonephritis, renal vascular diseases and those with a blood relative with CRF. CRF may also be identified when it leads to one of its recognized complications, such as cardiovascular disease, anemia or pericarditis. CRF is identified by a blood test for creatinine. Higher levels of creatinine indicate a falling GFR and as a result a decreased capability of the kidneys to excrete waste products. Creatinine levels may be normal in the early stages of CRF, and the condition is discovered if urinalysis shows that the kidney is allowing the loss of protein or RBCs into the urine. To fully investigate the underlying cause of kidney damage, various forms of medical imaging, blood tests and often renal biopsy are employed to find out whether there is a reversible cause for the kidney malfunction. Recent professional guidelines classify the severity of chronic kidney disease in 4 stages: Decreased renal reserve: when 50-75% of the renal function are preserved; Renal insufficiency: when 25-50% of the renal function are preserved; Renal failure: when 10-25% of the renal function are preserved; Renal end-stage: when less than 10% of renal function are preserved (uremic syndrome). There is no specific treatment unequivocally shown to slow the worsening of chronic kidney disease. If there is an underlying cause to CRF, such as vasculitis, this may be treated directly with treatments aimed to slow the damage. In more advanced stages, treatments may be required for anemia and bone disease. Severe CRF requires one of the forms of renal replacement therapy; this may be a form of dialysis, but kidney transplant is ideal. Signs and symptoms Initially, it is without specific symptoms and can only be detected as an increase in serum creatinine or protein in the urine. As the kidney function decreases: Blood pressure is increased due to fluid overload and production of vasoactive hormones, increasing one's risk of developing hypertension and/or suffering from congestive heart failure. Urea accumulates, leading to azotemia and ultimately uremia (symptoms ranging from lethargy to pericarditis and encephalopathy). Urea is excreted by sweating and crystallizes on skin ("uremic frost"). Hyperkalemia with a range of symptoms including malaise and potentially fatal cardiac arrhythmias. When K+ levels are above 6.5mmol/L it is an indication for hemodialysis. Erythropoietin synthesis is decreased (potentially leading to anemia, which causes fatigue). Fluid volume overload - symptoms may range from mild edema to life-threatening pulmonary edema. Hyperphosphatemia - due to reduced phosphate excretion, associated with hypocalcemia (due to vitamin D3 deficiency). The major sign of hypocalcemia is tetany.

Avi Sayag

Clinical Biochemistry

Later, this progresses to tertiary hyperparathyroidism, with hypercalcaemia, renal osteodystrophy and vascular calcification that further impairs cardiac function. Metabolic acidosis, due to accumulation of sulfates, phosphates, uric acid etc. This may cause altered enzyme activity by excess acid acting on enzymes and also increased excitability of cardiac and neuronal membranes by the promotion of hyperkalemia due to excess acid. People with chronic kidney disease suffer from accelerated atherosclerosis and are more likely to develop cardiovascular disease than the general population. Patients afflicted with CRF and cardiovascular disease tend to have significantly worse prognoses than those suffering only from the latter. Insulin is also increased as a response to the hyperkalemia (it activates the Na-K pump, in order for K+ to enter the cell). Testosterone and estrogen are also decreased. HCO3- and Na+ are also decreased In summary: Increased: K+, urea, creatinine, phosphate, Mg+2, H+, insulin, uric acid; Decreased: Na+, HCO3-, estrogen, testosterone, Ca+2, erythropoietin, Hb Several toxins are potential uremic toxin: 1. Urea: in very high concentrations it may lead to fatigue, vomiting and headache 2. Creatinine: affects glucose tolerance and RBC survival 3. Uric acid: uremic pericarditis 4. Cyanate: causes irreversible carbamylation of proteins, drowsiness and hyperglycemia 5. Polyols: may cause peripheral neuropathy 6. Phenols: may cause membrane toxicity 7. Medium Mw molecules might be responsible for most of symptoms 8. 2-microglobulins: may cause renal amyloidosis CRF may necessitate dialysis or later kidney transplant. Thus, the clinical and biochemical monitoring of patients include: 1. Monitoring of graft functions and graft rejection. The rejection manifests as oliguria and fever; 2. Increase in creatinine levels (due to rejection or ciclosporin toxicity) Most patients with CRF become hypocalcaemic and, in time, many develop renal osteodystrophy.

Avi Sayag

Clinical Biochemistry

Renal Tubular Diseases Renal tubular disorders can be congenital or acquired; they can involve single or multiple aspects of tubular function. The congenital conditions are inherited and all are rare: their clinical sequelae relate to the consequences of loss of substances that are normally completely or partially reabsorbed by the tubules. 1. The Fanconi syndrome This is a generalized disorder of tubular function characterized by glycosuria, amino aciduria, phosphaturia and acidosis. It can occur secondarily to a variety of conditions. One of these is cystinosis, or Lignac-Fanconi disease, a rare inherited disease in which there is a defect in the transport of cystine out of lysosomes. This leads to cystine accumulation and the deposition of cystine crystals in many body tissues, including the kidneys. Affected infants fail to thrive, develop rickets and polyuria with dehydration and eventually progress to renal failure. There is no specific treatment. Cystinosis should not be confused with cystinuria, a disorder of tubular transport. Cystinosis is one of idiopathic inherited metabolic diseases along with galactosemia, fructose intolerance, glycogen storage disease, tyrosinemia, and Wilson's disease. In Fanconi's syndrome, apart from the idiopathic inherited metabolic diseases, there are also: - Nephrotoxins (heavy metals and drugs) - Paraproteinemia - Amyloidosis 2. Renal tubular acidosis (RTA) In this condition there is acidosis due to: 1. Increased loss of bicarbonate (impaired reabsorption in the proximal tubule) and/or 2. Insufficient acidification of distal tubular fluid (impaired secretion of hydrogen ions in the distal tubule) There are 4 types: RTA-1 - distal renal tubular acidosis: it can be either inherited (autosomal dominant) or acquired (sporadic, non-familial). Other causes are drugs (such as amphotericin, gentamycin and lithium) and autoimmune diseases. There is a defect in H+ excretion and the urine cannot be acidified. Consequences include growth retardation, rickets, hypophosphatemia, osteomalacia, hypercalciuria, and often hypokalaemia. In general, hyperkalaemia is more usual in acidotic states, but in these types of RTA, the impaired ability of the kidneys to excrete H+ necessitates increased K+ excretion when Na+ is reabsorbed in the distal tubules, and this may cause H+ depletion and hypokalaemia. Treatment of type 1 RTA involves the administration of bicarbonate in sufficient quantities to buffer normal H+ production and K+ supplements. Diagnosis involves the ammonium chloride loading test, and the urinary pH is more the 5.5. RTA-2 proximal renal tubular acidosis: the lesion in proximal RTA is impairment of bicarbonate reabsorption. It can be primary/idiopathic or secondary to Fanconi's syndrome. It can also be associated with other non-inherited diseases such as MM, Sjogren syndrome, renal transplantation, hyper-D-vitaminosis, nephrotic syndrome, etc. Bicarbonate can be completely reabsorbed if the plasma bicarbonate concentration is low, and thus patients may excrete normal amounts of acid but at the expense of systemic acidosis. At nearly normal bicarbonate levels, bicarbonate is lost, but the urine pH does not fall below 5.5! Treatment consists of administering large amounts of bicarbonate. It is characterized by normokalemia or hypokalemia, hypocalcemia, hypophosphatemia, glycosuria, aminoaciduria, Na+ and K+ wasting. RTA-3 combined renal tubular acidosis RTA-4 selective aldosterone deficiency: It is associated with hypoaldosteronism, either secondary to adrenal disease, or to renal disease in which there is decreased renin secretion (hyporeninaemic hypoaldosteronism, e.g. in diabetic nephropathy) or resistance to aldosterone (e.g. in obstructive nephropathy). In contrast to the other types of RTA, there is hyperkalaemia. The urine can be maximally acidified, but the pH may decrease below 5.5.

Avi Sayag

Clinical Biochemistry

The diagnosis of RTA requires a high index of suspicion. Typically, there is hyperchloraemia and a normal anion gap. 3. Defects of urinary concentration Impairment of urinary concentration is a feature of nephrogenic diabetes insipidus, a group of primary tubular disorders. It is also a feature of cranial diabetes insipidus and chronic renal failure and can occur with hypercalcaemia, hypokalaemia and certain drugs, notably lithium. In inherited nephrogenic diabetes insipidus, ADH secretion is normal, but there is a mutation either affecting its receptor (the V2 receptor) or aquaporin 2. Hypercalcaemia and hypokalaemia interfere with the intracellular cAMP-mediated signalling pathway that leads to the insertion of aquaporins into the cell membranes of the collecting ducts. 4. Glycosuria Renal glycosuria can also occur in association with other tubular abnormalities, for example as part of the Fanconi's syndrome. 5. Amino aciduria: can also be part of Fanconi's syndrome. 6. Hypophosphataemic rickets This condition, also known as vitamin D-resistant rickets, has a dominant X-linked pattern of inheritance. A defect in tubular phosphate reabsorption leads to severe rickets. This does not respond to treatment with vitamin D alone, even if administered in massive doses, but can be treated effectively with a combination of oral phosphate supplements and vitamin D, usually given as a 1-hydroxylated derivative. Hypophosphataemic rickets should not be confused with inherited vitamin D-dependent rickets type I, an autosomal recessive condition. The defect is in the 1-hydroxylation of 25-hydroxycholecalciferol. This condition can be treated with 1-hydroxylated derivatives of vitamin D alone.

Avi Sayag

Clinical Biochemistry

Topic 25
Disturbances of acid-base balance
Generally, acid is a proton donor, while a base is a proton acceptor. The plasma pH is normally between 7.35 and 7.45. That is, ~40 nmol/L H+ are in the plasma. Below 7.35 acidosis occurs (below 7.25 it's a severe one) and above 7.45 alkalosis occurs (above 7.55 it's a severe one). Death occurs when pH drops below 6.8 or rises above 7.8. pH is measured using ion-selective glass electrode. Optimal pH provides conditions for optimal intracellular functions, such as enzymes, and the intracellular trapping of metabolite intermediates is maximized at a neutral intracellular pH. The main intermediate groups are phosphate, ammonium and carboxylic acids. There are 2 types of acids produced intracellularly: 1. Volatile acids (respiratory acids): CO2 derived from H2CO3. Between 12000 and 13000 mmol of CO2 is produced per day (basal production). 2. Fixed acids (metabolic acids): these are not excreted by the lungs, and are referred by their anions: lactate, phosphate, sulphate, acetoacetate, -hydroxybutyrate. In order to keep the balance, the amount of acid excreted must equal the amount of acid produced per day. As acids are produced intracellularly, they are buffered within the cell, and leave to the extracellular space while buffering is kept during transport. Then, acids are excreted. In the interstitial fluid, bicarbonate buffers metabolic acids. In the blood, bicarbonate buffers metabolic acids, and Hg buffers CO2. Proteins and phosphates are also present in these 2 compartments, but they are insignificant. In the intracellular fluid, however, proteins and phosphates are important buffers. In the urine, phosphate and ammonia are important buffers, while in the bone calcium carbonate is a the major player (in chronic metabolic acidosis). The ECF buffering comprises 43% of the total buffering capacity, and the buffers are the bicarbonate and proteins. The ICF buffering comprises the remaining 57%, and the buffers are proteins, phosphate and bicarbonate. This is due to entry of hydrogen ions via the Na-H exchangers (36%), K+-H+ exchangers (15%) and other transporters (6%). The respiratory part in keeping the acid-base balance is determined by the arterial pCO2 (changes in arterial pCO2 changes arterial pH). Central and peripheral chemoreceptors sense these changes (the PCR can also detect pO2, but the CCR can only detect pCO2) and signal the respiratory centers in the medulla (the apneustic center and the inspiratory center). These centers (mainly and directly through the inspiratory center) control the respiratory muscles, and lead to increased minute ventilation (hyperventilation) that ultimately reduces arterial pCO2 to its initial setpoint. This mechanism takes some 2-3 minutes to control the pH, and is thus very fast. The renal compensatory mechanism, however, is slower (takes 2-3 days to control pH). The mechanism compensates for the pH change by reabsorbing the filtered bicarbonate and by excretion of the fixed acids. These occur in the proximal and distal tubules. In the proximal tubule bicarbonate is reabsorbed (85%) and ammonium is produced. In the distal tubule bicarbonate is reabsorbed to a much lesser extent (0-5%), ammonium is added to the urine, and the H+ is buffered mainly by phosphate. Creatinine also contributes to the formation of titrable acidity. The remaining bicarbonate is reabsorbed in the thick ascending limb of the loop of Henle. The anion gap is a good indicator of metabolic acidosis. It is the difference between the sum of the concentrations of the principal cations (Na+ and K+) and the principal anions (Cl- and HCO3-). It represents the concentration of all the unmeasured anions in the plasma. Proteins (negative charge) account for about 10% of plasma anions and make up the majority of the unmeasured anions under normal circumstances. The acid anions (lactate, acetoacetate, sulphate) produced during metabolic acidosis are not measured as part of the usual lab tests. As known, H+ reacts with HCO3- and the CO2 produced in this reaction is excreted via the lungs. The net effect is a decrease in the concentration of the measured anions (bicarbonate) and an increase in the concentration of the unmeasured anions (the acid anions). Thus, the anion gap increases. The reference range for anion gap is 8-16 mmol/L.

Avi Sayag

Clinical Biochemistry

The metabolic acidosis can be organic (the lost bicarbonate is replaced by the acid anion, which is not normally measured) or inorganic (as in HCl infusion for example). The anion gap can help differentiate between these two: if the anion gap is normal, inorganic metabolic acidosis is present, as the chloride anions replace the lost bicarbonate; if the anion gap is increased, organic acidosis is present. The anion gap also helps determine the severity of the metabolic acidosis and follow the response to treatment. Blood gas analysis is another measure to determine alkalosis or acidosis (and whether they are respiratory or metabolic): pH: normally between 7.35 and 7.45; pCO2: gives information about the respiratory side. In arterial sample the reference range is 34-46 mmHg; pO2: normal reference range > 60 mmHg Standard bicarbonate: this is the concentration of bicarbonate in the plasma of fully oxygenated blood. In other words, it tells us what the bicarbonate would be if there were no respiratory disturbance. Therefore, it gives us information about the metabolic side. Normal reference range 22-26 mmol/L Actual bicarbonate: reflects the metabolic side. Normal reference range 25 mmol/L The difference between the actual and the standard bicarbonate concentrations indicates a respiratory alkalosis or acidosis: o When the actual bicarbonate is higher than the standard bicarbonate, it is a sign of respiratory acidosis. o When the actual bicarbonate is lower than the standard bicarbonate, it is a sign of respiratory alkalosis. Buffer base (BB): reflects the overall base contents of the organism. It gives information about the metabolic side. Normal reference range 44-52 mmol/L Base excess (BE): reflects the metabolic side. Normal reference range (-2.5)- (+2.5) mmol/L. Positive values mean base excess or lack of acids. In order to calculate how much bicarbonate is needed to be administered to correct for metabolic acidosis (ml) = BE x 0.3 x body weight Causes of acidosis and alkalosis Acidosis Alkalosis Increased intake of basic material (increased Increased intake of acids Increased acid production (diabetic intake of bicarbonate in ulcers or ketoacidosis, alcoholic ketoacidosis, overcompensation with bicarbonate) increased protein metabolism) Impaired excretion of bicarbonate (kidney Impaired H+ removal (renal disease) disese) Loss of base (diarrhea, renal diseases, loss of Loss of acids (vomiting) GI fluid) Hyperventilation (hysteria, fever, intracranial inflammatory disease, chest wall irritation, Impaired removal of CO2 (lung disease) irritation of the breathing center, inappropriate mechanical ventilation)

Avi Sayag

Clinical Biochemistry

Topic 26
Predominant water depletion, isoosmolar volume depletion
Predominant water depletion can occur when there is loss of fluid with Na+ concentration lower than that of the plasma, or when there is deficient water intake. Excessive loss of water without any sodium loss is unusual, except in diabetes insipidus, but, even if there is loss of sodium as well, provided that this is small, the clinical consequences will be related primarily to the water depletion. There are 2 main scenarios of water depletion: 1. Water depletion in the presence of normal homeostatic mechanisms, in which excessive water loss is due to sweating, vomiting, diarrhea, excessive respiratory loss and excessive burns, or due to deficient water intake due to inadequate supply and mechanical obstructions. 2. Water depletion in the absence of (or failure) of the homeostatic mechanisms for water retention, in which there is inadequate response to thirst, diabetes insipidus, osmotic diuresis and nephrogenic diabetes insipidus5. Features of predomiant water depletion With normal homeostatic mechanisms With deficient homeostatic mechanisms Hyperosmolality Hypovolemia (late stage) Oliguria Polyuria (but not in case of damage to the thirst center) Hypernatremia Hemoconcentration Mild uremia Decreased urinary Na+ Concentrated urine Diluted urine (only in diabetes insipidus and nephrogenic diabetes insipidus) If the patient is unconscious, predominant water depletion manifests as: 1. Pyrexia (fever) 2. Overbreathing 3. Osmotic diuresis (osmotic diuresis is increased urination caused by the presence of certain substances in the small tubes of the kidneys. The excretion occurs when substances of high molecular weight, such as glucose, enter the kidney tubules. The substances cause an increase in the osmotic pressure within the tubule, causing retention of water within the lumen, and thus reduce the reabsorption of water, increasing urine output). 4. Diabetes insipidus 5. Inability to drink The block in predominant water depletion is between the hypothalamus and the secretion of ADH. Hypernatremia accompanies predominant water depletion (details in next topic). Isoosmolar volume depletion can occur due to: 1. Loss of small-intestinal secretion (in the presence of normal homeostatic mechanisms). This can occur if there is a fistula in the small intestine, or if there is obstruction or paralytic ileus in the small intestine. 2. Tubular damage with normal glomerular function (in the absence of normal homeostatic mechanisms). This can occur during the recovery phase of acute renal failure or when there is polyuric chronic renal failure.

Diabetes insipidus is a condition characterized by excessive thirst and excretion of large amounts of severely diluted urine, with reduction of fluid intake having no effect on the latter. There are several different types of DI, each with a different cause. The most common type is central diabetes insipidus, caused by a deficiency of ADH, also known as antidiuretic hormone (ADH). The second common type of DI is nephrogenic diabetes insipidus, which is caused by an insensitivity of the kidneys to ADH.

Avi Sayag

Clinical Biochemistry

The hypovolemia that results from this condition manifests in hypotension, oliguria, uremia and hemoconcentration. The management of water depletion involves treatment of the underlying cause and replacement of the fluid deficit. Water should preferably be given either orally or via a nasogastric tube. If this is not possible, either 5% dextrose or, if there is also some sodium depletion, dextrose-saline should be administered (4% dextrose, 0.18% NaCl) intravenously. The aim should be to correct approximately two-thirds of the deficit in the first 24 h and the remainder in the next 24 h, but plasma osmolality should not be allowed to fall too rapidly. However, 2 side effects should be borne in mind during fluid replacement: 1. If the Na+ concentration is low in the fluid replacement, predominant sodium depletion can pursue; 2. Overcorrection with protein-free fluid will lead to increased hydrostatic pressure (and reduced oncotic pressure), which will eventually lead to increased loss of fluid.

Avi Sayag

Clinical Biochemistry

Topic 27
Water and sodium excess; predominant sodium depletion
Predominant Na+ depletion is one of 3 causes of hyponatremia, the second being water excess and the third excess of water and Na+. Hyponatremia As mentioned, there are three causes for hyponatremia: 1. Depletion of Na+ (hypovolemic hyponatremia) 2. Excess of water (euvolemic hyponatremia) 3. Excess of water and sodium (hypervolemic hyponatremia) Each one will be considered separately: 1. Predominant sodium depletion This condition can be caused due to 2 main reasons: a. Volume replacement with fluid of incorrect composition: in the first phase of this condition the clinical symptom is hypovolemia alone, while in the second phase hypoosmolality and polyuria are the dominant symptoms (hyponatraemia is a very common finding in postoperative patients on IV fluid infusions. It usually reflects excessive administration of hypotonic fluids (5% dextrose or 'dextrose-saline') at a time when the ability of the body to excrete water is depressed as part of the normal metabolic response to trauma, which includes increased release of ADH). The lab findings in the first phase are normonatremia, hemoconcentration, increased urea and decreased urinary Na+ concentration, while in the second phase there are hyponatremia, hemodilution, decreased urea and increased urinary [Na+]. b. Failure of homeostatic mechanisms for Na+: this cause is primarily due to Addison's diseases or pseudoaddison's disease. Addison's disease is a rare endocrine disorder in which the adrenal gland produces insufficient amounts of steroid hormones. Pseudoaddison's disease is a condition in which there is no dysfunction of the adrenal gland, but failure of the renal tubules to respond to aldosterone (the distal tubules and the collecting ducts). The clinical signs span from hypovolemia to late hypoosmolality (same as in the previous cause), and the lab findings are hemoconcentration, mild uremia, late hyponatremia and inappropriately high urinary [Na+] (again, same as in the previous condition). 2. Predominant excess of water There are 4 main factors leading to predominant excess of water: Glomerular dysfunction if fluid of low [Na+] is replaced in excess to fluid loss; Inappropriate ADH secretion (SIADH); Oxytocin or other narcotics given in infusion (in 5% glucose or dextrose-saline); Psychogenic polydipsia Water excess gives rise to a dilutional hyponatremia with reduced plasma osmolality. It can occur acutely purely due to excessive water intake, but this is rare. Normal kidneys are capable of excreting 1 L of water per hour: water intoxication and hyponatremia will thus be seen only when very large quantities of fluid are ingested rapidly, as is seen in some patients with psychoses. It can also occur in people who drink large quantities of weak beer. Far more frequently, however, the acute development of water excess and hyponatremia is a result of a combination of excessive hypotonic fluid intake and impairment of diuresis. Since osmolality is normally precisely controlled, the persistence of dilutional hyponatremia implies a failure of diuresis, which must be due to either continued (and inappropriate) production of ADH (or the presence of a drug having a ADH-like action) or an impairment of the renal diluting mechanism. SIADH is essentially a diagnosis of exclusion. It is frequently diagnosed on insufficient evidence with no regard to other possible causes of hyponatremia. Both clinical information and laboratory data are important. It is essential to measure urine and plasma osmolalities: the urine may not be more concentrated than the plasma but must be less than maximally dilute (osmolality >50 mmol/kg). Edema is not a feature of SIADH: the excess of water is shared by the ICF and the ECF and the effect on ECF volume is insufficient to cause edema.

Avi Sayag

Clinical Biochemistry

Measurement of ADH concentration is seldom helpful in differential diagnosis: raised values are present in the majority of patients with hyponatremia, irrespective of the cause. There is undoubtedly more than one type of SIADH. Tumors may produce the hormone (ectopic production), but patients with many other conditions can also fulfill the diagnostic criteria for SIADH. In some of these, there may be an inappropriate stimulus to ADH release, such as stimulation of volume receptors during artificial ventilation, and in others the 'osmostat' appears to be reset, so that osmolality is still controlled but at a lower level. Decreased intracellular organic solute ('osmolyte') content may be one mechanism whereby the osmostat can be reset. Patients have been described in whom suppression of ADH release when osmolality falls is incomplete (a 'ADH leak'), while in others the production of ADH is entirely normal and antidiuresis must be presumed to reflect an abnormal response to the hormone. 3. Water and Na+excess Water excess This is usually related to an impairment of water excretion. However, the limit to the ability of the healthy kidneys to excrete water is about 20 mL/min and, occasionally, excessive intake is alone sufficient to cause water intoxication. This can sometimes occur in patients with psychiatric disorders. It has also been described in people drinking large amounts of beer with a low solute content, because this results in a low osmotic load for excretion and there is a minimum osmolality below which the urine cannot be diluted further. Increased thirst can occur in organic brain disease (particularly trauma, and following surgery). Hyponatremia is invariably present in water overload. The increased water load is shared by the ICF and ECF. The clinical features of water overload are related to cerebral over-hydration, the incidence and severity depending upon the extent of the water excess and its time course. A patient with a plasma Na+ concentration of 120 mmol/L, in whom water retention has occurred gradually over several days, may be asymptomatic, while one in whom this is an acute phenomenon may show signs of severe water intoxication. Na+excess Na+ excess can result from increased intake or decreased excretion. The clinical features are related primarily to expansion of ECF volume (peripheral edema, pulmonary edema, venous congestion and hypertension). When related to excessive intake (e.g. the inappropriate use of hypertonic saline), a rapid shift of water from the intracellular compartment may also cause cerebral dehydration. When Na+ overload is due to excessive intake, hypernatremia is usual. Na+overload is more usually due to impaired excretion than to excessive intake. The most frequent cause is secondary aldosteronism. This is seen in patients who, despite clinical evidence of increased ECF volume (e.g. peripheral edema), appear to have a decreased effective arterial blood volume, for example due to venous pooling or a disturbance in the normal distribution of ECF between the intravascular and extravascular compartments. This phenomenon is particularly associated with cardiac failure, hypoalbuminaemia and hepatic cirrhosis, damage to renal vessels, and pregnancy. The clinical symptoms of such patients are determined by the primary condition. Lab findings may reveal mild hyponatremia or normonatremia, a decrease in urinary Na+ concentration, hypokalemia (if loop diuretics are administered) and other findings related to the primary abnormality. That many such patients with Na+ excess are, paradoxically, hyponatraemic, implies the coexistence of a defect in free water excretion. This is probably in part due to an increase in ADH secretion as a result of the decreased effective blood volume. Also, the decrease in GFR and consequent increase in proximal tubular Na+ reabsorption decreases the delivery of Na+ and Cl- to the loops of Henle and distal convoluted tubules. This reduces the kidneys' diluting capacity, thereby compromising water excretion. Renal disease is a relatively uncommon cause of Na+ excess, as is increased mineralocorticoid secretion due to primary aldosteronism (PA) (as in Conn's syndrome). It is noteworthy, however, that edema is not a feature of Conn's syndrome. The clinical features of PA are hypertension and hypokalemia. Lab findings will reveal hypokalemia, elevated plasma HCO3-, plasma Na in the upper limit, decreased urinary sodium concentration, elevated aldosteron levels and decreased renin levels.

Avi Sayag

Clinical Biochemistry

Measurement of serum Na+ and K+ (practical topic 16)


There are 4 methods to measure these ions: 1. Atomic absorption spectrophotometry This is the reference method. The electrons of the atoms in the atomizer can be promoted to higher orbitals for a short amount of time by absorbing a set quantity of energy (i.e. light of a given wavelength). This amount of energy (or wavelength) is specific to a particular electron transition in a particular element, and in general, each wavelength corresponds to only one element. This gives the technique its elemental selectivity. This method is no longer used in routine diagnostics. 2. Flame photometry If the salt of a metal is vaporized into a flame at an appropriate temperature, a part of the metal's atoms will be excited. Light emitted from the thermally excited ions directed into a photometer results in an electric signal. The calibrators contain Na+ and K+ in known concentrations. In practice, lithium salt of a constant concentration is mixed into the diluting solution. Then, signals of the Na+ and K+ are compared to the lithium (lithium is an ideal internal standard as it is not present in the serum of normal individuals and its light emission if different from that of Na+ and K+). Reference values: Na+: 135-145 mmol/L; K+: 3.6-5 mmol/L. 3. Ionselective electrodes This electrode is a membrane producing electric potential, which is proportional to the concentration of the ion. The membrane is a thin film made of solid material or plastic into which components responsible for selectivity are mixed. In cases of an ion selective membrane electrode, a membrane separates the internal electrolyte (known concentration of ions) from the external electrolyte (unknown concentration), and it is selectively permeable for an ion. This selective permeability can generate a measurable change in membrane potential that depends on the ion activity in the sample. In direct potentiometry the ion activity is measured in undiluted samples, while in the indirect potentiometry samples are diluted before analysis. Reference values: Na+: 137-150 mmol/L; K+: 3.5-5.3 mmol/L. False results: homolytic samples (K+ concentration in RBCs is 23 greater than that of plasma); stored sample; contamination with Na/K infusion. Pseudohyponatremia is a falsely low [Na+] in a hyperlipemic or hyperproteinemic sample when measured by flame photometry or indirect potentiometry. 4. Spectrophotometry

Avi Sayag

Clinical Biochemistry

Topic 28
Hypokalemia
Hypokalemia refers to the condition in which the concentration of K+ in the blood is low. Normal serum K+ levels are between 3.5 to 5.2 mmol/L; at least 95% of the body's K is found inside cells, with the remainder in the blood. This concentration gradient is maintained principally by the Na+/K+-ATPase pump. Pathophysiology K+ is essential for many body functions, including nerve activity and skeletal and cardiac muscle contraction and rhythm. The electrochemical gradient of K+ between the intracellular and extracellular space is essential for nerve function; in particular, K+ is needed to repolarize the cell membrane to a resting state after an action potential has passed. Decreased K+ levels in the extracellular space will cause hyperpolarization of the resting membrane potential. This hyperpolarization is caused by the effect of the altered K+ gradient on resting membrane potential as defined by the Goldman equation. As a result, a greater than normal stimulus is required for depolarization of the membrane in order to initiate an action potential. In certain conditions, this will make cells less excitable. However, in the heart, it causes myocytes to become hyperexcitable. Lower membrane potentials in the atrium may cause arrhythmias because of more complete recovery from Na+-channel inactivation, making the triggering of an action potential more likely. In addition, the reduced extracellular K+ (paradoxically) inhibits the activity of the IKr K current and delays ventricular repolarization. This delayed repolarization may promote reentrant arrhythmias. Regularly, K+ depletion leads to hypokalemia. However, an exception to this rule is hypokalemia without K+ depletion. This occurs when there is ECF-to-ICF shift of K+. Causes Hypokalemia can result from one or more of the following medical conditions: 1. Perhaps the most obvious cause is insufficient consumption of K+ (that is, a low-K+ diet). However, without excessive K+ loss from the body, this is a rare cause of hypokalemia. Chronic starvation, anorexia nervosa and alcoholism reduce K+ intake. 2. A more common cause is excessive non-renal loss of K+, often associated with heavy fluid losses that "flush" K+ out of the body. Typically, this is a consequence of vomiting, diarrhea, laxatives, intestinal fistula6, mucus-secreting villous adenomas (K+ loss is secondary to mucus secretion), excessive perspiration, or losses associated with surgical procedures 3. Certain medications can accelerate the removal of K+ from the body, including: 1. Thiazide diuretics such as hydrochlorothiazide: act on the distal convoluted tubule, where it blocks the reabsorption of Na+. Thus, in the more distal part of the tubule there is an accelerated reabsorption of Na+ ions into the bloodstream in exchange for K+. 2. Loop diuretics such as furosemide: for the same reason (act on the thick ascending loop of Henle) 3. Carbonic anhydrase inhibitors, such as acetazolamide: in the proximal convoluted tubule, this enzyme promotes reabsorption of bicarbonate. Inhibiting this enzyme will result in non-reabsorption of NaHCO3. It will lead to metabolic acidosis, and to an accelerated reabsorption of Na+ ions into the bloodstream in exchange for K+ in the more distal part of the tubule . 4. Osmotic diuretics, such as mannitol: they act on the descending loop of Henle. The drug annexes water to it along K+. 5. Amphotericin B: the drug is toxic to the kidney (RTA-1). Nephrotoxicity can lead to increased K+ secretion and thus to hypokalemia. 4. A special case of K+ loss occurs with diabetic ketoacidosis. In addition to urinary losses from polyuria and volume contraction, there is also obligate loss of K+ from kidney tubules as

A fistula is a narrow passage or duct formed by disease or injury, as one leading from an abscess to a free surface, or from one cavity to another.

Avi Sayag

Clinical Biochemistry

a cationic partner to the negatively charged ketone, -hydroxybutyrate. Note that this is a special case of hyperkalemia with K+ depletion. 5. Hypomagnesemia can cause hypokalemia. Magnesium is required for adequate processing of K+. This may become evident when hypokalemia persists despite K+ supplementation. Other electrolyte abnormalities may also be present. 6. Alkalosis can cause transient hypokalemia by two mechanisms. First, the alkalosis causes a shift of K+ from the plasma and interstitial fluids into cells; perhaps mediated by stimulation of Na+-H+ exchange and a subsequent activation of Na+/K+-ATPase activity. Second, an acute rise of plasma HCO3- concentration (caused by vomiting, for example) will exceed the capacity of the renal proximal tubule to reabsorb this anion, and K+ will be excreted as an obligate cation partner to the bicarbonate. It should be noted that metabolic alkalosis is often present in states of volume depletion, and thus alkalosis is typically not the main cause of hypokalemia seen in volume-depleted states. 7. Disease states that lead to abnormally high aldosterone levels can cause hypertension and excessive urinary losses of K+. These include renal artery stenosis and tumors (generally nonmalignant) of the adrenal glands. Hypertension and hypokalemia can also be seen with a deficiency of the 11-beta-hydroxysteroid dehydrogenase type 2 enzyme which allows cortisol to stimulate aldosterone receptors. This deficiency can either be congenital or caused by consumption of glycyrrhizin, which is contained in extract of licorice, sometimes found in herbal supplements, candies and chewing tobacco. It follows that increased secretions of mineralocorticoids may lead to hypokalemia (hyperaldosteronism, Cushing syndrome with steroid therapy, ACTH therapy or ectopic ACTH secretion, renal hemangiopericytoma, and licorice/tobacco chewing). 11--OH-steroid DH -----------| cortisol aldosterone R hypokalemia (aldosteron R increased Na+ reabsorption increased K+ secretion hypokalemia) Rare hereditary defects of renal salt transporters, such as Bartter syndrome or Gitelman syndrome, can cause hypokalemia, in a manner similar to that of diuretics. As opposed to disease states of primary excess of aldosterone, blood pressure is either normal or low in Bartter's or Gitelman's. Rare hereditary defects of muscular ion channels and transporters that cause hypokalemic periodic paralysis can precipitate occasional attacks of severe hypokalemia and muscle weakness. These defects cause a heightened sensitivity to the normal changes in K+ produced by catechols and/or insulin and/or thyroid hormone, which lead to movement of K+ from the extracellular fluid into the muscle cells. Lastly, renal dysfunction leads to hypokalemia: renal tubular acidosis types I and II (RTA-I and RTA-II), recovery phase of acute oliguric renal failure, chronic pyelonephritis, polycystic kidney, interstitial nephritis, and amphotericin B. Summary of Causes of Hypokalemia: There are 3 main causes: low intake, a shift from the ECF to the ICF, and increased loss in the urine: 1. Decreased intake: due to starvation or clay ingestion (perhaps K+ is trapped there) 2. ECF ICF : there are 4 main causes: 1. Metabolic alkalosis 2. Hormonal: insulin (stimulates the Na-H antiporter and secondarily the Na-K pump), -antagonists, and 2 agonists (2 receptors participate in glycogenolysis. Stimulating them will result in hyperglycemia, and increased insulin level. Moreover, hyperglycemia will lead to osmotic diuresis and K+ wasting). 3. Anabolic states: vitamin B12/folic acid, GM-CSF pernicious anemia treated with vitamin B12/folic acid, or neutropenia treated with GM-CSF, lead to rapid cell growth and an increased anabolic state. Since K+ is the major intracellular cation, there is an increased shift of K+ into the cells. 4. Others: hypokalemic periodic paralysis, hypothermia, barium toxicity.

Avi Sayag

Clinical Biochemistry

3. Increased loss: this can be further subdivided into: 1. Non-renal loss: GI loss (diarrhea) or integumentary loss (sweating) 2. Renal loss: due to increased K+ secretion (RTA1, RTA2, aldosteron excess, diabetic ketoacidosis, hypomagnesemia, amphotericin B) or increased distal flow (diuretics, osmotic diuresis). Signs and symptoms Mild hypokalemia is often without symptoms, although it may cause a small elevation of blood pressure, and can occasionally provoke cardiac arrhythmias. Moderate hypokalemia, with serum K+ concentrations of 2.5-3 mEq/L, may cause muscular weakness, myalgia, and muscle cramps (owing to disturbed function of the skeletal muscles), and constipation (from disturbed function of smooth muscles). With more severe hypokalemia, flaccid paralysis, hyporeflexia, and tetany may result. There are reports of rhabdomyolysis occurring with profound hypokalemia with serum K+ levels less than 2 mEq/L. Respiratory depression from severe impairment of skeletal muscle function is not uncommon. Other symptoms include impaired glucose tolerance, symptoms of extracellular alkalosis, renal tubular damage, impairment of memory, disorientation and confusion. Some (ECG) findings associated with hypokalemia are flattened (or low) T waves, increased U waves, ST segment depression, and prolongation of the QT interval. The prolonged QT interval may lead to arrhythmias. Treatment The most important treatment in severe hypokalemia is addressing the cause, such as improving the diet, treating diarrhea or stopping an offending medication. Patients without a significant source of K+ loss and who show no symptoms of hypokalemia may not require treatment. Mild hypokalemia (>3.0 mEq/L) may be treated with oral K+ chloride supplements (KlorCon, Sando-K, Slow-K). As this is often part of a poor nutritional intake, K+-containing foods may be recommended, such as leafy green vegetables, tomatoes, citrus fruits, oranges or bananas. Both dietary and pharmaceutical supplements are used for people taking diuretic medications. Severe hypokalemia (<3.0 mEq/L) may require intravenous supplementation. Typically, saline is used, with 20-40 mEq KCl per liter over 3-4 hours. Giving intravenous K+ at faster rates (20-25 mEq/hr) may predispose to ventricular tachycardias and requires intensive monitoring. A generally safe rate is 10 mEq/hr. Difficult or resistant cases of hypokalemia may be amenable to a K+-sparing diuretic such as amiloride. In contrast to the more commonly used diuretics like hydrochlorothiazide and furosemide, these K+-sparing diuretics actually reduce the kidney's excretion of K+. When replacing K+ intravenously, infusion via central line is encouraged to avoid the frequent occurrence of a burning sensation at the site of a peripheral IV, or the rare occurrence of damage to the vein.

Avi Sayag

Clinical Biochemistry

Topic 29
Hyperkalemia
Extreme degrees of hyperkalemia are considered a medical emergency due to the risk of potentially fatal arrhythmias (levels above 8.5 mmol/L cardiac arrest). Signs and symptoms Symptoms are nonspecific and generally include malaise, palpitations and muscle weakness; mild hyperventilation may indicate a compensatory response to metabolic acidosis, which is one of the possible causes of hyperkalemia. Often, however, the problem is detected during screening blood tests for a medical disorder, or it only comes to medical attention after complications have developed, such as cardiac arrhythmia or sudden death. Diagnosis In order to gather enough information for diagnosis, the measurement of K+ needs to be repeated, as the elevation can be due to hemolysis in the first sample. The normal serum level of K+ is 3.5 to 5.2 mmol/L. Generally, blood tests for renal function (creatinine, blood urea nitrogen), glucose and occasionally creatine kinase and cortisol) will be performed. Calculating the trans-tubular K+ gradient can sometimes help in distinguishing the cause of the hyperkalemia. In many cases, renal ultrasound will be performed, since hyperkalemia is highly suggestive of renal failure. Also, ECG may be performed to determine if there is a significant risk of cardiac arrhythmias. Causes include: 1. Ineffective elimination from the body Renal insufficiency Medication that interferes with urinary excretion: ACE inhibitors and angiotensin receptor blockers K-sparing diuretics (e.g. amiloride and spironolactone) NSAIDs The calcineurin inhibitor immunosuppressants ciclosporin and tacrolimus The antibiotic trimethoprim The antiparasitic drug pentamidine Mineralocorticoid deficiency or resistance, such as: Addison's disease Aldosterone deficiency, including reduced levels due to heparin Some forms of congenital adrenal hyperplasia Type IV renal tubular acidosis (resistance of renal tubules to aldosterone) Gordon's syndrome (familial hypertension with hyperkalemia), a rare genetic disorder caused by defective modulators of salt transporters, including the thiazide-sensitive Na+-Clcotransporter. 2. Excessive release from cells Rhabdomyolysis, burns or any cause of rapid tissue necrosis, including tumor lysis syndrome. Massive blood transfusion or massive hemolysis Shifts/transport out of cells caused by acidosis, low insulin levels (which lead to decreased activity of the Na+-K+ pump), beta-blocker therapy, digoxin overdose (blocks the Na+-K+ pump), or the paralyzing anesthetic succinylcholine (depolarizing muscle relaxants can increase the plasma K+). 3. Excessive intake Intoxication with salt-substitute, K+-containing dietary supplements, or KCl infusion. Note that for a person with normal kidney function and nothing interfering with normal elimination, hyperkalemia by K+ intoxication would be seen only with large infusions of KCl or massive doses of oral KCl supplements. Administration of high doses of penicillin may also lead to hyperkalemia, since penicillin is given as K+ salt or Na+ salt. 4. Lethal injection Hyperkalemia is intentionally brought about in an execution by lethal injection, with KCl being the third and last of the three drugs administered to cause death.

Avi Sayag

Clinical Biochemistry

5. Pseudohyperkalemia Pseudohyperkalemia is a rise in the amount of K+ that occurs due to excessive leakage of K+ from cells, during or after blood is drawn. It is a laboratory artifact rather than a biological abnormality and can be misleading to caregivers. Pseudohyperkalemia is typically caused by hemolysis during venipuncture; excessive tournequet time or fist clenching during phlebotomy (which presumably leads to efflux of K+ from the muscle cells into the bloodstream); or by a delay in the processing of the blood specimen. It can also occur in specimens from patients with thrombocytosis, leukocytosis, or erythrocytosis (hematocrit>55%). The renal elimination of K+ is passive (through the glomeruli), and reabsorption is active in the proximal tubule and the ascending limb of the loop of Henle. There is active excretion of K+ in the distal tubule and the collecting duct; both are controlled by aldosterone. Increased extracellular K+ levels result in depolarization of the membrane potentials of cells. This depolarization opens some voltage-gated Na channels, but not enough to generate an action potential. After a short while, the open Na channels inactivate and become refractory, increasing the threshold to generate an action potential. This leads to the impairment of neuromuscular, cardiac, and gastrointestinal organ systems. Of most concern is the impairment of cardiac conduction which can result in ventricular fibrillation or asystole. During extreme exercise, K+ is released from active muscle and the serum K+ rises to a point that would be dangerous at rest. Patients with the rare hereditary condition of hyperkalemic periodic paralysis appear to have a heightened sensitivity of muscular symptoms that are associated with transient elevation of K+ levels. Episodes of muscle weakness and spasms can be precipitated by exercise or fasting in these subjects. ECG findings: peaked T waves. Treatment: Acute: When arrhythmias occur, or when K+ levels exceed 6.5 mmol/l, emergency lowering of K+ levels is mandated: Ca+2 supplementation (calcium gluconate) does not lower K+ but decreases myocardial excitability, protecting against life threatening arrhythmias. Insulin will lead to a shift of K+ ions into cells. Bicarbonate therapy is effective in cases of metabolic acidosis. Salbutamol (Ventolin), a 2-selective Refractory or severe cases may need dialysis to remove the K+ from the circulation.

Avi Sayag

Clinical Biochemistry

Topic 30
Pathogenesis of diabetes mellitus
Diabetes mellitus (DM) is characterized by a tendency to chronic hyperglycaemia with disturbances in carbohydrate, fat and protein metabolism that arise from a defect in insulin secretion or action or both. Diabetes can occur secondarily to other diseases (see table). Most cases of DM are primary. There are two distinct types. In type 1 DM, there is destruction of pancreatic cells, leading to a decrease in, and eventually cessation of, insulin secretion. Approximately 10% of all patients with diabetes have type 1. They have an absolute requirement for insulin. In type 2 DM, insufficient insulin is secreted to prevent hyperglycaemia, often because of resistance to its actions. Most patients with type 2 DM can initially be successfully treated by diet, with or without oral hypoglycaemic drugs, but many eventually require treatment with insulin to achieve adequate glycaemic control. Type 1 DM usually presents acutely in younger people, with symptoms developing over a period of days or only a few weeks. However, there is evidence that the appearance of symptoms is preceded by a 'prediabetic' period of several months during which growth failure (in children), a fall in insulin response to glucose and various immunological abnormalities can be detected. Type 2 DM tends to present more chronically in the middle-aged and elderly (although it is increasingly being diagnosed in obese young people), with symptoms developing over months or even longer. The prevalence of type 2 DM is over 10% in people over the age of 75 years. The previously used terms, 'insulin-dependent' and 'juvenile-onset' diabetes (for type 1) and 'non-insulin-dependent' and 'maturity-onset' diabetes (for type 2) are obsolete! It has become apparent that some young patients with diabetes are not insulin dependent, while approximately 10% of patients developing diabetes over the age of 25 have 'latent autoimmune diabetes of adulthood' (LADA). Patients with LADA may be misclassified as having type 2 DM. However, in comparison with patients with true type 2 DM, they tend to present at a younger age, are less likely to be overweight, have plasma markers of autoimmunity and, although often treated successfully with diet alone or diet and oral agents initially, develop a requirement for insulin, often within a year of diagnosis. Type 1 diabetes mellitus Type 1 DM is an autoimmune disease. There is a familial incidence, though to a lesser extent than with type 2 DM, and there is a strong association with certain histocompatibility antigens, for example HLA-DR3, DR4 and various DQ alleles. An individual's HLA antigens are genetically determined but it is clear that type 1 DM is a genetically heterogeneous disorder. Environmental factors are also important and there is considerable circumstantial evidence that viral antigens (e.g. Coxsackie B) may initiate the autoimmune process in some genetically susceptible individuals. Proteins in cows' milk have also been implicated. The pancreatic islets of newly diagnosed patients with type 1 DM show characteristic histological features of autoimmune disease. Islet cell antibodies (ICA) are frequently present in the plasma (and may be detectable long before the condition presents clinically), together with antibodies to insulin and glutamic acid decarboxylase (GAD), which, like ICA, are sensitive markers of risk of progression to clinical diabetes in the apparently healthy members of patients' families. It is thought that -cell destruction is initiated by activated T-lymphocytes directed against antigens on the cell surface, possibly viral antigens or other antigens that normally are either not expressed or not recognized as 'non-self'. Type 2 diabetes mellitus Type 2 is characterized by impaired insulin secretion, insulin resistance (a defect downstream the insulin receptor), excessive hepatic glucose production, and abnormal fat metabolism. The exact pathogenesis of type 2 DM is uncertain. It is undoubtedly a heterogeneous disease. In established cases, -cell dysfunction with an inadequate insulin response to hyperglycemia and insulin resistance usually coexist but it is not clear which is the primary defect: hyperglycemia itself causes insulin resistance and -cell dysfunction (glucotoxicity); so, too, does hyperlipidemia (lipotoxicity), which is frequently present in diabetes. Immune

Avi Sayag

Clinical Biochemistry

mechanisms are thought to contribute to the loss of insulin secretion in approximately 10% of patients. Type 2 DM shows a strong familial incidence. Several single gene defects have been identified in specific subsets of patients with type 2 DM, notably in the dominantly inherited forms that typically develop in the young (MODY, maturity-onset diabetes of the young). The commonest mutations responsible for MODY are in the glucokinase gene (MODY type 2: six types of MODY have been described, each due to a different mutation see table). Glucokinase is the rate-limiting enzyme of glucose metabolism in pancreatic -cells and through acting as a 'glucose sensor' is key to the regulation of pancreatic insulin secretion. Such specific mutations are, however, rare in type 2 DM considered overall, where the tendency to develop diabetes is polygenic and there is no clear pattern of inheritance. Environmental factors are also important. Many patients with type 2 DM are obese, particularly tending to have visceral obesity, which is known to cause insulin resistance. Reduced physical activity also causes insulin resistance, and various drugs, including corticosteroids, thiazides in high doses and some -adrenergic antagonists, are diabetogenic. The interaction between genetic and environmental factors in the pathogenesis of type 2 DM is exemplified by the high prevalence of the condition in certain ethnic groups (e.g. Pacific islanders) following the adoption of a westernized lifestyle, with good public health facilities and ready access to an assured food supply, in comparison with the prevalence in their aboriginal state. The suggestion is that their genotype evolved to maximize the storage of ingested energy as fat, to provide protection against famine, but that a continuous food supply leads to obesity and insulin intolerance (the 'thrifty genotype' hypothesis). There is also a 'thrifty phenotype' hypothesis, based on the observation that low birthweight is associated with an increased risk of later development of type 2 DM, the putative mechanism being cell dysfunction induced by fetal malnutrition. Type 2 DM is a progressive condition. Although there is evidence that it can be prevented in susceptible individuals by diet and exercise, by the time it presents clinically it will often have been present for several years. Aggressive treatment may slow its progression, but the tendency is for continuing loss of -cell function and increasing insulin deficiency.

Glucose and other nutrients regulate insulin secretion by the pancreatic beta cell. Glucose is transported by the GLUT2; subsequent glucose metabolism by the cell alters ion channel activity, leading to insulin secretion. The SUR (Sulfonylurea receptor) receptor is the binding site for drugs that act as insulin secretagogues. Mutations in the events or proteins underlined are a cause of MODY or other forms of diabetes.

I. Type 1: cell destruction C A U S E S Immune mediated Idiopathic

II. Type 2: Insulin resistance with def. Insulin resistance Impaired secretion

Genetic defects of cells MODY1 mutated HNF 4 MODY 2 glucokinase MODY 3 HNF 1 MODY 4 IPF-1 MODY 5 HNF 1 MODY 6 NeuroD1

Genetic defects in insulin action Type A insulin resistance Leprechaunism RMS LS

Diseases pancreas

of

III. Endocrinopathies

Druginduced

Infections

Other associated conditions Down's sy. Turner sy. Klinefelter's sy.

IV. Gestational

Pancreatitis Pancreatectomy Neoplasia CF Hemochromatosis

Acromegaly Cushing's Glucagonoma Pheochromocytoma Somatostatinoma Aldosteronoma Thyrotoxicosis

Pentamidine Nicotinic acid IFN Phenytoin Clozapine Thiazides

Rubella CMV Coxsackie

4% of pregnant women. 30-60% will develop DM later in life

MODY maturity onset of diabetes of the young HNF hepatocyte nuclear transcription factor IPF Insulin promoter factor Leprechaunism - (also known as Donohue syndrome) is an extremely rare and severe genetic disorder. Leprechaunism derives its name from the fact that those afflicted with the disease often have elfin features and are smaller than usual. Affected individuals have an insulin receptor with greatly impaired functionality. RMS Rabson-Mendenhall syndrome - rare insulin receptor disorder characterized by severe insulin resistance, developmental abnormalities, and acanthosis nigricans. LS Lipodystrophy syndrome - a disturbance of lipid metabolism that involves the partial or total absence of fat and often the abnormal deposition and distribution of fat in the body. Insulin resistance is a feature of this syndrome.

Topic 31
Lab diagnosis and management of diabetes mellitus
The aim of lab tests regarding DM is to diagnose DM, to monitor the therapy, to predict late complications, to diagnose metabolic complications and to manage them. The diagnosis of diabetes mellitus depends upon the demonstration of hyperglycaemia. In a patient with classic symptoms and signs, this may be inferred from the presence of glycosuria but glycosuria is not diagnostic of diabetes, even in the presence of classic clinical features. Therefore, urine samples should be collected (collected or random) as well as capillary/venous blood (random or in fasting). Note the reference range is somewhat different for capillary blood and venous blood. The diagnosis is made by detection of fasting plasma glucose (FPG) and Oral Glucose Tolerance Test (OGTT). The WHO has issued diagnostic criteria for DM based on 2 principles: 1. The spectrum of FPG and the response to OGTT varies among individuals, and 2. DM is defined as the level of glycemia at which diabetes-specific complications occur rather than on deviations from a population-based mean. The results may have some errors: samples stored at 20C will show a decrease of 0.4 mmol/L in glucose levels per hour of storage, while those stored at 4C will have an error of 0.1 mmol/L per hour of storage. The sampling may clearly be inappropriate if taken during glucose infusion. Apart from determination of glucose levels, antibodies can also be detected to determine the cause/type of diabetes: antibodies against Islet cell, antibodies against insulin, or antibodies against glutamic acid decarboxylase (GAD).

These values taken from Harrison do not correspond with those of the department. Here are the values of the department. Fasting plasma glucose cut-off values: Normal: < 6.1 mmol/L (if venous/capillary blood: < 5.6 mmol/L) Impaired fasting glycemia (IFG): 6.1-7 mmol/L (if venous/capillary blood: 5.6-6.1 mmol/L) DM: > 7 mmol/L (if venous/capillary blood: > 6.1 mmol/L) In a patient with typical features of DM (polyuria, polydipsia) a single measurement is sufficient: Fasting plasma glucose: > 7 mmol/L Random venous plasma: > 10 mmol/L Random capillary blood: > 11.1 mmol/L In the absence of symptoms, any of these limits must be exceeded on more than one occasion for the diagnosis to be made. Individuals who have fasting blood glucose concentrations that are elevated but not in the diabetic range have impaired fasting glycaemia (IFG). Their

Avi Sayag

Clinical Biochemistry

response to a glucose load should be tested to determine whether they have diabetes. Other indications include unexplained glycosuria7 particularly in pregnancy, the presence of clinical symptoms with normal fasting glucose, and diagnosis of acromegaly. The OGTT is performed as follows: The patient should eat normally for 3 days, with at least 250 grams of sugars; The patient fasts overnight Basal blood sample is taken for glucose determination 75g glucose are given in 250-300ml water over the course of 5 minutes (children are given 1.75g/kg, but it should not exceed 75g!) The patient then rests for 2 hours. Smoking is not allowed. He/she is allowed to drink water Blood sample is taken after 2 hours for glucose determination. Interpretation of results DM is diagnosed if the patient's glucose level exceeds 11.1 mmol/L. Impaired glucose tolerance: 7.8-11.1 mmol/L Impaired fasting glycemia: 6.1- 7.8 mmol/L Normal result: < 6.1 mmol/L Note again that the OGTT is not performed routinely! Management Education of patients is vital along with dietary control with or without oral hypoglycaemic agents in patients with type 2 DM (at least initially; insulin is often required later in the course of the condition), and with diet and insulin in patients with type 1 DM. Regular follow-up is essential to monitor treatment and detect early signs of complications, particularly retinopathy, which can, in many cases, be treated successfully, and nephropathy, since treatment may slow its progression. The aims of treatment are two-fold: to alleviate symptoms and prevent the acute metabolic complications of diabetes, as well as to prevent long-term complications. In type 1 DM, there is an increasing tendency to use 'basal-bolus' regimens of insulin injections, whereby a long acting insulin is given at night (to mimic the basal insulin secretion that occurs even during fasting) with boluses of short-acting insulin at meal times. Continuous subcutaneous insulin infusion is also being used, but is demanding for the patient. Hitherto, insulin has had to be given by injection, as it is degraded in the gut. Alternative routes of administration, for example inhaled, are now being developed and introduced. In patients with type 2 DM, improved control over that achieved with oral hypoglycaemics alone can often be achieved by giving a single injection of a long-acting insulin at night, continuing with the oral agents during the day, but standard insulin regimens may be required for some patients. Drugs used in the treatment of type 2 DM include metformin (a biguanide, decreases glycogenolysis); sulphonylureas (enhances insulin secretion by acting on SUR); thiazolidinediones (glitazones, agonists of the peroxisome proliferator-activated receptor (PPAR), which enhance the actions of insulin by increasing the sensitivity to it), and meglitinides (rapidly and short acting insulin secretagogues by stimulating the smaller subunit of the K+ channel). Monitoring The aim of monitoring DM is to control the metabolic balance and to predict complications. 4 tests are used for these purposes: 1. Blood glucose: monitored daily for children and adolescents with DM. Blood glucose should be more than 3 mmol/L, fasting blood glucose should be less than 7 mmol/L, and after a meal it can rise up to 10 mmol/L; 2. Urinary glucose: same as for blood glucose; During infections and poor metabolic control, daily urine tests for ketonuria should be performed.
Glycosuria is detected in the urine by dipstick (insensitive but specific). It occurs when plasma glucose is > 11 mmol/L and the GFR is normal, or when the maximal tubular reabsorption capacity for glucose is decreased (renal glycosuria).
7

Avi Sayag

Clinical Biochemistry

3. Glycated Hb (HbA1c): should be checked at least every 3 months. The levels should be between the mean of normal range and up to 1-2% above the range; The International Diabetes Federation and American College of Endocrinology recommend that HbA1c values be below 6.5%, while the American Diabetes Association recommends that the Hb A1c be below 7.0% for most patients. 4. Fructosamine: monitors the level of blood glucose in the past 3 weeks. There is no

standard reference range available for this test. The reference values depend upon the factors of patient age, gender, sample population, and test method. Hence each laboratory report will include their specific reference range for the test. Fructosamine = (HbA1c 1.61) x 58.82
The control of DM can be made at home (as mentioned in "Management"). If the metabolic control is good, then DM should be controlled 4 times a year for fasting plasma glucose (for type II DM), fructosamine and HbA1c, and once a year for renal functions and lipid parameters. The complications of DM are: 1. Diabetic retinopathy; 2. Diabetic neuropathy; 3. Diabetic nephropathy; In the initial stage, this condition is predicted by the presence of microalbuminuria, and in later stage by macroalbuminuria and creatinine. Microalbuminuria is screened and diagnosed when morning albumin is greater than 20 mg/L and the albumin/creatinine ratio is over 2.5 mg/mmol. In the control of microalbuminuria urine is collected over 24 hours. If more than 20 microgram/min (or 30 mg/day) are collected, microalbuminuria is diagnosed (control should be performed every 6 months). Note that microalbuminuria means the presence of small amount of albumin in the urine. 4. Cataract 5. Infections 6. Complications of pregnancy: pregnancy is NOT recommended if HbA1c is greater than 10%. The target values in pregnancy should be pursued: a. Fasting plasma glucose: < 5.6 mmol/L b. Post-prandial plasma glucose: < 8 mmol/L c. HbA1c/fructosamine: up to upper limit of normal range 7. Premature coronary artery disease; 8. Cerebral and peripheral vascular diseases: these are predicted and diagnosed by the determination of cholesterol, triglycerides, HDL, LDL, VLDV, chylomicrons. 9. Impaired wound healing; 10. The prediction of acute metabolic complications is made by the detection of ketone bodies in the urine. Screening: 1. All individuals above 45 should be screened for DM once in 3 years if the results are normal. 2. More frequent screening in the following cases: a. Obesity; b. A family member with DM; c. Ethnic groups of high risk; d. Women who had GDM or delivered a baby over 4500g; e. Hypertensive patients; f. HDL < 0.9 mmol/L, Tg > 2.8 mmol/L g. If previous results suggested IGT or IFG

Avi Sayag

Clinical Biochemistry

Determination of glucose in serum; point of care tests (practical topic 17)


Glucose in the serum can be detected using lab assays and quick tests. Lab assays: there are 4 assays available: 1. Glucose oxidase (GOD) assay; 2. Oxygen consumption in GOD reaction 3. Hexokinase assay 4. Glucose dehydrogenase assay 1. Glucose oxidase assay In this reaction D-glucose + O2 ------> gluconic acid + H2O2 (carried by GOD) Then, H2O2 +para-amino-phenazone + m-cresol -------> H2O + oxidized product (carried by peroxidase) GOD is highly specific for D-glucose, but the second step involving the peroxidase is less specific. The intensity of the color product is proportional to the glucose concentration up to 30 mmol/L. Above the linearity limit (30mmol/L) the sample has to be diluted and the result should be multiplied by the degree of dilution. After the reagent is added to the sample, it is mixed and kept at room temperature for 30 minutes. The absorbance is determined at 520nm against the reagent blank. Glucose (mmol/L) = (Abs of sample/Abs of standard) X 7.9 Disadvantages: Many compounds in the serum/urine (e.g. bilirubin, ascorbic acid, uric acid) can be oxidized by H2O2 leading to a negative bias (falsely low glucose value) Some compounds may react by oxidizing the indicator dye leading to a positive bias (falsely high glucose values). 2. O2 consumption The amount of O2 consumed in the GOD reaction can be measured by an oxygen sensitive electrode. This method is highly specific for glucose. 3. Hexokinase assay Glucose + ATP ------> G-6-P + ADP (carried by hexokinase) G-6-P + NADP ------> 6-phosphogluconate + NADPH + H+ (carried by G6PDH) 4. Glucose dehydrogenase (GDH) Glucose + NAD --------> D-gluconolactone + NADH + H+ Reference range: 3.6-6 mmol/L Falsely low values may be obtained due to glycolysis by blood cells. Glucose level may decrease if the plasma/serum is not separated from the cellular elements of blood within 1.5-2 hours following blood sampling. Glycolysis in whole blood causes a 5%/hour decrease in glucose level. This value may be higher in samples with high WBC count or in case of bacterial contamination. Samples may be stored for prolonged periods if glycolysis is prevented (by drawing blood into tubes containing NaF). Falsely high values may be obtained if: The absorbance of the blank of lipemic serum is not subtracted from the absorbance of the sample. The sample is drawn during glucose infusion or right after it. If fasting samples are taken after meal (inappropriate timing). Quick tests: using home monitoring devices, glucose values obtained from whole blood are about 12% lower than plasma values due to difference in water content (glucose is equally distributed between cellular water and plasma water, but the water content of RBCs is lower than that of the plasma. The device is generally a strip impregnated by a mixture of GOD and catalase that ensure the color reaction. The color is evaluated either by a color scale or by a reflectance photometer.

Avi Sayag

Clinical Biochemistry

Point of care tests (POCT) These tests can be carried out in an inpatient setting, outpatient clinics and by the patients themselves. The values measured by the POCT device should be compared with that of the central lab every 4 months. There are no strict professional or personnel requirements for POCT. If laboratory automatization and robotization reduce the turnaround time for a given test below 30 minutes, then POCT indication is questionable. Thus, for example, some professional guidelines suggest that if the test results required for MI diagnosis (CK-MB, troponin, IMA) cannot be delivered within 30 minutes, a POCT should be used. Advantages: o The test can be performed at the site of care. o A negligible amount of sample is needed (10-15L) o Treatment can be started immediately and its efficacy can be followed at the site of care. o The devices are compact and cheap. o They are movable, and usually do not require external power supply. o The greatest advantage is the very short turnaround time (1-15 minutes). Disadvantages: o The customized POCT is quite expensive compared to the serial measurements carried out in the central lab. o The accuracy of the POCT does not match that of the tests performed at the central lab. o As many people use the device, it leaves more room for errors. o The older POCT devices cannot always be connected to the central lab's information system. The most frequent POCT is the glucose test. The first POCT used 56 years ago is the pregnancy test. Other POCTs: cholesterol tests and triglyceride tests. The instrument measures the amount of light reflected by the dye and translates it into the analyte concentration.

Avi Sayag

Clinical Biochemistry

Topic 32
Acute metabolic complications of DM
The complications that might accompany diabetes mellitus are: Diabetic ketoacidosis with or without hyper- and/or hypokalemia (more frequently associated with type 1 DM) Non-ketotic hyperglycemia (hyperosmolal non-ketotic coma; more frequently associated with type 2 DM) Hypoglycemia CVA Lactic acidosis Diabetic nephropathy Lipoprotein metabolism in DM The first 4 can lead to coma. The first 2 are the 2 main acute complications. Diabetic ketoacidosis Coma can be induced by acidosis and hypovolemia. As the intracellular glucose level decreases, and in the absence of insulin to facilitate glucose intake into the cells (DM type 1), the cell starts to utilize fatty acids for energy production. Ketone bodies are formed and accumulate in the plasma (ketonemia). This directly can cause coma, and indirectly through hyperventilation and vomiting, can cause loss of water, Na+ and K+, which results in hypovolemia (that in turn leads to coma). The other pathway leading to coma is through glycogenolysis, gluconeogenesis and decreased glucose intake into the cells, all are the consequence of decreased insulin levels. These will cause hyperglycemia, which will induce osmotic diuresis and a loss of water, Na+ and K+. As in the previous pathway, this loss will lead to hypovolemia and coma. As a result of the hypovolemia, the GFR reduces and urea accumulates in the blood. Proteolysis will also lead to accumulation of amino acids in the plasma, which in turn will increase urea synthesis and its accumulation in the plasma (uremia). Thus, ketoacidosis may be the presenting feature of type 1 DM, or may develop in a patient known to be diabetic who omits to take his insulin or whose insulin dosage becomes inadequate because of an increased requirement, for example as a result of infection, any acute illness such as myocardial infarction, trauma or emotional disturbance. The 2 main ketone bodies responsible for the ketoacidosis are acetoacetate and -hydroxybutyrate. Diabetic ketoacidosis is a medical emergency. Following notification to the lab, two primary tests should be performed in diabetic ketoacidosis: 1. Blood test: test for glucose, Na+, K+, HCO3-, pH and pCO2. 2. Urine test: test for glucose and ketone bodies. If the potassium levels are low in DM, it can be due to: 1. Osmotic diuresis; 2. Enhanced aldosterone secretion (water and sodium loss) 3. Vomiting. If the potassium levels are high in DKA, it can be due to: 1. Metabolic acidosis; 2. Disorders of intermediary metabolism; 3. Insulin deficiency; 4. Enhanced muscle catabolism; 5. ICF-to-ECF shift; 6. Renal insufficiency. A ketotic state is not necessarily caused by DM, but can also be caused by fasting state and vomiting. Whereas glucose levels are normal or slightly decreased in fasting and vomiting, they are elevated in DKA.

Avi Sayag

Clinical Biochemistry

Detection of ketone bodies can be performed by sampling urine and blood: 1. Urine: Legal test/Ketostix8 2. Blood: Acetest and/or enzymatic detection of -hydroxybutyrate Note that most tests do not detect -hydroxybutyrate, and ketonemia can appear without ketonuria. This occurs when the GFR is low. Summary of overall lab findings in ketoacidosis: Plasma: hyperglycemia, acidosis (low bicarbonate), hyperkalemia, elevated amylase levels, hyperphosphatemia, presence of ketone bodies, hemoconcentration and mild uremia (note that some parameters are included in the primary tests while some are not). Urine: ketone bodies, glycosuria and low pH. Note that if the patient is comatose due to ketoacidosis, the lab tests that confirm the source of coma are elevated plasma glucose, low plasma bicarbonate levels, elevated urine glucose and elevated urine ketone bodies. Non ketotic hyperglycemia Not all patients with uncontrolled diabetes develop ketoacidosis. In type 2 DM, severe hyperglycemia can develop (blood glucose concentration >50 mmol/L) with extreme dehydration and a very high plasma osmolality, but with no ketosis and minimal acidosis. This complication is often referred to as hyperosmolar non-ketotic hyperglycemia, but patients with ketoacidosis usually also have increased plasma osmolality, although not to the same extent. Why is it non-ketotic? There is sufficient insulin secretion to prevent the excessive lipolysis and to oppose the ketogenic action of glucagon that are essential for the generation of ketoacidosis. Thus, if the patient is comatose due to hyperosmolal non-ketotic hyperglycemia, the lab tests that confirm the source of coma are elevated plasma glucose levels with normal (or slightly depressed) bicarbonate level and elevated urine glucose level. The issue of hypoglycemia is addressed in the next topic.

Ketostix is Bayer's brand name for test strips to measure the level of ketones in the urine. A strip consists of a thin piece of plastic film slightly larger than a matchstick, with a reagent pad on one end that is either dipped into a urine sample or passed through the stream while the user is voiding. The pad is allowed to react for a short time, then its resulting color is compared to a graded shade chart indicating a detection range from negative presence of ketones up to a significant quantity.

Avi Sayag

Clinical Biochemistry

Summary and presentation of the topic: Pathogenesis of DKA: Intracellular glucose + no insulin

Insulin

FA are utilized/lipolysis

Glycogenolysis + gluconeogenesis

Ketone bodies formed Hyperglycemia Ketonemia Osmotic diuresis Coma hyperventilation and vomiting Loss of water, Na, K loss of water, Na, K Hypovolemia Uremia GFR hypovolemia Coma

Coma Urea Proteolysis Insulin

Lab: blood test (glucose, Na, K, pH, HCO3-, pCO2); urine test (glucose and ketone bodies) Reasons for decreased K levels and increased K levels Non-ketotic hyperglycemia High FPG + high urine glucose + normal HCO3 non-ketotic hyperglycemia High FPG + high urine glucose + low HCO3 ketoacidosis

Avi Sayag

Clinical Biochemistry

Topic 33
Hypoglycemia
Hypoglycemia is defined as blood glucose concentration of less than 2.2 mmol/L. Suspicion of hypoglycemia should arise when the patient is unconscious, behaves strangely or presents with an altered mental state. Hypoglycemia is more frequently due to decreased hepatic glucose production than due to increased tissue uptake. The causes of hypoglycemia can be divided into those causing a low blood glucose concentration during fasting (fasting hypoglycaemia at night or early morning, after physical exercise, or in a severe organic disease) and those in which it follows a stimulus (reactive hypoglycaemia), including a meal (post-prandial hypoglycaemia), drugs (the commonest causes are insulin and hypoglycaemic drugs) and alcohol consumption. The clinical features of hypoglycemia are the result of dysfunction of the nervous system (neuroglycopenia): Acute symptoms due to neuroglycopenia include tiredness, confusion, lack of concentration, ataxia, dizziness, paraesthesiae, hemiparesis, coma, vertigo and diplopia. Acute symptoms due to sympathetic stimulation are trembling, pallor, tachycardia and sweating. There are other non-specific signs in the acute phase which are hunger, weakness and blurred vision. In chronic neuroglycopenia the patient presents with personality changes, memory loss, psychosis and dementia. Diagnosis There are two stages in the diagnosis of hypoglycaemia: confirmation of the low blood glucose concentration and elucidation of the cause. In children and young adults, symptoms will usually be present only with a concentration of less than 2.2 mmol/L. The elderly tend to be more sensitive to low blood glucose concentrations; neonates, however, often develop features only when the blood glucose is <1.5 mmol/L. Although reagent strips and glucometers can be used to confirm a clinical suspicion of hypoglycaemia, they are insufficiently accurate at low blood glucose concentrations to provide a definitive diagnosis and formal laboratory measurements should be used. Blood must be collected into a container with sodium fluoride, to inhibit glycolysis. Glucose, insulin and C-peptide should be tested for. Reactive hypoglycemia Reactive hypoglycemia can be caused by drugs (insulin, sulphonylurea), after a meal (essential or following gastric surgery), alcohol and others (galactose, fructose and leucine). Drug-induced hypoglycemia Most patients with type 1 diabetes experience occasional episodes of hypoglycemia. Attempts to attain optimum glycemic control increase the risk of hypoglycemia. The secretion of glucagon becomes impaired in established type 1 diabetes, and the lack of this important counter-regulatory hormone impairs the body's natural defenses against hypoglycemia. Insulin: disturbed patients sometimes deliberately administer excessive insulin to attract attention. Sulphonylureas: in patients with type 2 diabetes, hypoglycemia can complicate treatment with sulphonylureas. Chlorpropamide is most frequently implicated. It has a long plasma half-life and, since it is renally excreted, tends to accumulate in patients with impaired renal function. -Adrenergic blockers: occasionally cause hypoglycemia, but only when other factors such as starvation, severe exercise or liver disease are involved. 2 receptor stimulation increases glycogenolysis in the liver. Thus, blocking these receptors decreases the breakdown of glycogen and facilitates hypoglycemia. Salicylates: children, but not adults, poisoned with salicylates may develop severe hypoglycemia. The mechanism of the enhanced insulin secretion due to salicylic acid appears to be mediated by prostaglandin synthesis inhibition.

Avi Sayag

Clinical Biochemistry

Paracetamol: patients who have taken overdoses of paracetamol (it is probably related to the severe liver damage that this drug can cause). Post-prandial hypoglycemia In patients who have undergone gastric surgery involving either a gastrointestinal anastomosis or a pyloroplasty, hypoglycemia, developing 90-150 min after a meal, particularly a meal rich in sugar, is common. There is rapid passage of glucose into the small intestine and release of hormones that stimulate insulin secretion. The insulin response is excessive and hypoglycemia ensues as glucose absorption from the gut falls off rapidly, rather than slowly as it does when gastric emptying is normal. Symptoms suggestive of hypoglycemia following meals may be described by people who have not undergone surgery (essential or idiopathic post-prandial hypoglycemia). Alcohol and reactive hypoglycemia Insulin- and drug-induced reactive hypoglycemia are potentiated by alcohol. Alcohol also increases insulin release in response to an oral glucose load and this may enhance any tendency to post-prandial reactive hypoglycemia. Other causes of reactive hypoglycemia Various inherited metabolic diseases have reactive hypoglycemia as a feature. Sudden cessation of a hypertonic dextrose infusion being given as part of a parenteral feeding regimen can precipitate hypoglycemia, especially when insulin has been given concomitantly. Hypoglycemia can also occur after dialysis against a glucose-rich dialysate. Fasting hypoglycemia 1. High endogenous insulin: a. Insulinomas: are tumors of the insulin-secreting -cells of the pancreatic islets. The presence of an insulin-secreting tumor can be inferred from the presence of an inappropriately high plasma insulin concentration (>20 pmol/L) at a time when the blood glucose concentration is low (<2.2 mmol/L). C-peptide should also be measured. Although secreted in equimolar amounts with insulin, C-peptide is cleared from the circulation more slowly, so it may be a more reliable marker of endogenous insulin secretion than insulin itself. It should be noted that glucose tolerance tests have no role in the investigation of possible insulinomas. b. Nesidioblastosis: hyperinsulinemic hypoglycemia attributed to excessive function of pancreatic beta cells with an abnormal microscopic appearance. 2. Glucocorticoid deficiency: lack of cortisol can be due either to primary adrenal failure or secondary to panhypopituitarism. Mild hypoglycemia can occur with isolated deficiency of ACTH or growth hormone, but in the latter condition, it is never symptomatic. 3. Liver disease/intoxication: although the liver is central to glucose homoeostasis, its functional reserve is so great that hypoglycemia is a rare feature of hepatic disease. It may occur, however, with the rapid, massive hepatocellular destruction that can follow poisoning with paracetamol and other toxins. The kidneys are the only organs other than the liver capable of gluconeogenesis; they are also responsible for insulin degradation. These facts may in part explain the severe hypoglycemia that is occasionally a feature of end-stage renal disease. 4. Non-pancreatic neoplasms: hypoglycemia can also occur in association with nonpancreatic neoplasms, including hepatocellular and adrenal carcinomas, carcinoid tumors and large mesenchymal tumors such as retroperitoneal sarcomas. Patients are usually not ketotic and, except with some carcinoid tumors, plasma insulin concentrations are not increased. It has been suggested that increased glucose uptake by the tumor may be a factor but this is unlikely ever to be the sole cause. Hepatic glucose output is often reduced although there is a normal glucogenic response to glucagon. It is probable that most such tumor-related hypoglycemia is related to the secretion of insulin-like growth factors (IGFs). Plasma IGF-1 concentrations are

Avi Sayag

Clinical Biochemistry

consistently low in such patients but IGF-2 (particularly in the non-protein-bound form) is often increased, and the ratio IGF-1:IGF-2 is decreased. 5. Alcohol-induced fasting hypoglycemia: hypoglycemia can be induced by ethanol, which inhibits gluconeogenesis and potentiates the action of insulin. 6. Sepsis: hypoglycemia sometimes develops in patients with septicemia. It is thought to be a result of the release of cytokines, which may stimulate insulin secretion or have a direct effect on hepatic glucose production. Renal impairment may also be contributory. 7. Inherited metabolic disease: fasting hypoglycemia is an important feature of glycogen storage disease type I. Hypoglycemia in childhood Neonatal hypoglycemia: hypoglycemia can develop among infants of diabetic mothers (they have islet-cell hyperplasia), erythroblastosis fetalis, in cases of intrauterine malnutrition, and among preterm or small-for-date infants (because they are born with low hepatic glycogen stores and are more likely to have feeding problems. Extensive physiological changes occur at birth and, in terms of glucose metabolism, there is a sudden interruption of the maternal glucose supply, so that glycogenolysis must span the period until feeding becomes established). Hypoglycemia in infancy: in infancy, ketotic hypoglycemia can occur (can be idiopathic, or due to relative carbohydrate deficiency). Hyperinsulinism is another cause (due to nesidioblastosis, islet-cell hyperplasia or insulinomas). Lastly, inborn errors of metabolism may lead to hypoglycemia (e.g. galactosemia, fructose intolerance, disorders of -oxidation of fatty acids and glycogen storage diseases).

Avi Sayag

Clinical Biochemistry

Topic 34
Disorders and laboratory diagnosis of lipid metabolism
Lipoproteins are classified on the basis of their densities as demonstrated by their ultracentrifugal separation. Density increases from chylomicrons through VLDL, intermediate density (IDL) and LDL to HDL. HDL can be separated, on the basis of density, into two metabolically distinct subtypes, HDL2 (density 1.064-1.125) and HDL3 (density 1.126-1.21). Distinct subtypes of LDL (LDL-I, II and III, in increasing order of density) are also recognized. IDL are normally present in the blood-stream in only small amounts but can accumulate in pathological disturbances of lipoprotein metabolism. However, it is important to appreciate that the composition of the circulating lipoproteins is not static. They are in a dynamic state with continuous exchange of components between the various types. Lipoprotein Density (g/mL) Source Main function CM < 0.95 seen after 1h of Intestine in response to a Transport of exo. TG centrifugation fatty meal from gut to tissue + chol. from gut to liver VLDL 0.96-1.006 seen after Liver in response to a Transport of endo. TG 20h of centrifugation high CH meal from tissue to liver and back IDL 1.007-1.019 Catabolism of VLDL Precursor of LDL LDL 1.02-1.063 seen after Catabolism of VLDL via Chol. transport from 24h of centrifugation IDL in peripheral tissues liver to tissues when VLDL gives up TG HDL 1.064-1.21 seen after Liver, intestine; Chol. Transport from 36h of centrifugation catabolism of CM and tissues to liver VLDL Lipoproteins are composed of: 1. A polar lipid surface layer (phospholipids, cholesterol, apolipoproteins); 2. A non-polar lipid core (TG, cholesteryl ester) 3. Enzymes (LCAT, phospholipase A2, paraoxonase) The proteins (the apolipoproteins) are very weakly associated with a specific lipoprotein, and are easily transferred to another lipoprotein of a different class. They have a structural role and a binding site for receptors. There are several apolipoproteins: B-100, B-48, A-I, A-II, AIV, C-I, C-II, C-III and E. The major differences between different lipoproteins (CM, VLDL, IDL, LDL, HDL) are in: 1. The size of the neutral lipid core (the non-polar core) 2. The lipid composition in the core 3. The apolipoproteins in the core CM have the highest percentage of lipids and TG, followed by VLDL. LDL has the highest percentage of cholesterol, and HDL has the highest amount of proteins (LDL has more lipids % than HDL). 5 major complications are associated with hypertriglyceridemia: 1. Risk for atheroma 2. Acute pancreatitis (and recurrent abdominal pain as a result of it) 3. Pseudohyponatremia 4. Focal neurological syndromes 5. Rheumatological manifestations High levels of TG can be observed in the following settings: 1. Alcohol comsumption 2. Estrogen (for contraception or post-menopausal treatment) 3. Use of vitamin A derivatives 4. Thiazide diuretics (in high doses) 5. non-selective -blockers (without intrinsic sympathomimetic activity) The increased levels of TG are associated with: 1. Increased LDL and small dense LDL particles;

Avi Sayag

Clinical Biochemistry

2. Reduced HDL 3. Reduced apoA-I (only) 4. DM is frequently found 5. Obesity may contribute 6. Glucose intolerance with insulin resistance and hyperinsulinemia The disorders of lipid metabolism are based on the levels of the various lipids as well as on electrophoresis or ultracentrifugation of lipoproteins. Thus, before addressing the disorders, the lab diagnosis is described.

Cholesterol, HDL, LDL assays (practical topic 19)


Cholesterol The sample is serum obtained from whole blood, centrifuged at 1500g for 10 minutes at 4C. Samples should be collected after 12-14 hour fasting. The determination of cholesterol has 3 steps: 1. Cholesterol-ester is hydrolyzed by cholesterol esterase (Ch-ester Ch) 2. Cholesterol is oxydized by cholesterol oxydase (Ch + O2 Ch + H2O2) 3. H2O2 + 4aminoantipirin + phenol chinoimine (by peroxidase) Chinoimine is a color product. Its absorbance is measured, and the cholesterol levels are calculated. A reference range for cholesterol does not exist, as the cholesterol level exceeds the desired value in the majority of the population. Rather, the risk limit is used. The risk limit is 5.2 mmol/L. Above 6.8 mmol/L has to be reduced by diet or drugs. If total cholesterol is 5.2-6.8 mmol/L, it is necessary to determine HDL and LDL levels as well as other risk factors (smoking, etc.). A 3-minute venous compression can increase cholesterol values by up to 10% (as well as in blood collected during standing). Also, vitamin C reacts with H2O2 and thus lowers the values of cholesterol. HDL In the presence of divalent cations (Mg+2 for example), lipoproteins can be selectively precipitated with polyanions. By using precipitating reagents in the appropriate concentrations, the -, pre-lipoproteins and chylomicrons can be precipitated, and the lipoprotein (HDL) that remains in the supernatant can be determined (for , and pre bands, see next practical topic 20). The most frequently used precipitating agent is Mg+2-phosphowolframate. Results obtained with this method correlate well with the HDL-C values measured from the HDL fraction after ultracentrifugation. LDL LDL is measured by selective precipitation, direct immunoseparation or by using the following formula: LDL-C(mmol/L) = total cholesterol HDL-C TG/2.2 This formula holds when Tg levels do not exceed 4.5 mmol/L and if type III hyperlipoproteinemia can be excluded. Some current assays contain different detergents and other chemicals, which allow specific blocking or solubilization of lipoprotein classes in order to achieve specificity for LDL. The LDL is measured enzymatically in the same cuvette with HDL. It is desirable to keep the LDL below 3.4 mmol/L, and start treatment above 4.8 mmol/L. The ratio LDL/HDL is diagnostically informative of the risk to develop atherosclerotic vascular disease (e.g. a ratio of 3.6 for women represents an average risk.

Triglyceride assay, visual test, lipoprotein electrophoresis (practical topic 20)


Visual test The appearance of the plasma in the lab may provide the first clue that a patient has hyperlipidemia. In health, in the fasting state, plasma is clear. Following a meal, it often becomes opalescent owing to the light-scattering properties of chylomicrons and VLDL. At triglyceride concentrations above about 4 mmol/L, the plasma becomes increasingly turbid; with severe hypertriglyceridemia, it appears milky (lipemic). If plasma is left undisturbed, chylomicrons float to the surface, leaving a clear infranatant layer; VLDL remain in

Avi Sayag

Clinical Biochemistry

suspension. LDL do not scatter light and even at high plasma cholesterol concentrations the plasma remains clear9. Blood for lipid studies should be drawn after an overnight fast (12-14 h), when chylomicrons, derived from dietary fat, should have normally been cleared; a pathological disturbance may thus be inferred if they are present. The patient should have kept to his or her own normal diet for two weeks before the blood is taken. Alcohol should not have been taken on the evening before blood sampling. Alcohol is a common cause of hypertriglyceridemia even in patients who have otherwise fasted. When lipid studies are done on a patient who has had a myocardial infarction or stroke, blood should either be taken within 24 h or after an interval of three months, since the metabolism of lipoproteins is disturbed during the convalescent. By keeping serum or plasma at 4C for 18-24 hours, chylomicrons are separated and form a creamy layer on top of the sample, while VLDL remain in the infranatant and in high concentration cause turbidity (these 2 are the main carriers of Tg). LDL and HDL (that mostly contain cholesterol) have no effect on the visual appearance of the sample. If the sample if clear, the Tg level is < 2.5 mmol/L in most cases. Above 2.5 mmol/L the serum is opalescent, and above 6.5 mmol/L it is very turbid. Tryglycerides Tg is measured by enzymatic methods only (4 steps): 1. Tg glycerol + fatty acids (lipase, esterase) 2. Glycerol + ATP G-3-P + ADP (glycerol kinase) 3. G-3-P + O2 dihydroxyacetone-P + H2O2 4. H2O2 + 4aminophenazone + p-chlorphenol chinoimine + H2O + HCl (peroxidase) Reference values are age-dependent: <29 y/o : 1.35 mmol/L <39 y/o : 1.68 mmol/L >40 y/o : 1.76 mmol/L Values above these refer to hypertriglyceridemia. Lipoprotein electrophoresis Electrophoresis (mostly on agarose or cellulose acetate) separate lipoproteins according to their charges. Quantitation of the lipoprotein fractions is carried out by densitometric scanning of the electrophotogram. On agarose gel, lipoproteins are separated from the cathode to the anode in the following order: chylomicrons, , pre and bands. The band which migrates the furthest corresponds to the HDL The band which migrates the shortest corresponds to the LDL The pre which migrates to the middle corresponds to the VLDL Chylomicrons do not enter the pores of the gel due to their large size. Ultracentrifugation separates lipoproteins according to their density: HDL>LDL>VLDL>chylomicrons. The disorders associated with lipid metabolism are: hyperlipoproteinemia, hypolipoproteinemia, hypertriglyceridemia and atherosclerosis. Hyperlipoproteinemia There are 5 types according to the WHO: I, II (A and B), III, IV, and V. Type I increased CM Type IIA increased LDL Type IIB increased LDL and VLDL Type III increased IDL or -VLDL Type IV increased VLDL Type V increased VLDL and CM

HDL and LDL do not contribute to the visual examination. Only CM and VLDL do.

Avi Sayag
Type I

Clinical Biochemistry
Signs Elevated Tg and normal-toelevated cholesterol. Clear visual appearnace.

Primary Lipoprotein lipase (LPL) deficiency (AR) apoCII deficiency (because it is a cofactor for LPL)

IIA

IIB III

IV

Secondary Uncontrolled DM, SLE, hypothyrosis, pancreatitis, oral contraceptives, abnormal plasma globulins AD block in LDL Hypothyrosis, synthesis/transport/ uptake/ mixedema, nephrosis, metabolism obstructive biliary apoB-100 deficiency (most disease, acute cases of type II) intermittent porphyria AD (polygenic) AR IDL metabolism is Hypothyrosis, DM, altered; SLE, primary biliary Abnormal apoE; cirrhosis Absence of hepatic lipase Familial Hypothyrosis, hypertriglyceridemia (AD) nephrosis, obesity, and familial combined alcoholism, DM, oral hyperlipidemia (AD) contraceptives and steroid therapy Familial Nephrosis, obesity, hypertriglyceridemia (AD) alcoholism, DM, myeloma, pancreatitis, hyperuricemia

Elevated cholesterol, and nornal Tg. Clear visual appearnace.

Slightly turbid serum Elevated cholesterol, and elevated Tg. Turbid, slim chylomicron layer. Elevated Tg, normal-toelevated chylomicrons. Turbic appearance.

Elevated Tg, normal-toelevated chylomicrons. Turbic appearance.

Hypolipoproteinemia There are 2 categories to this disorder: 1. Low cholesterol with normal/low HDL 2. Low HDL alone 1. Low cholesterol with normal or low HDL 3 different diseases can lead to this condition: 1. Abetalipoproteinemia: this is an AR disorder. In abetalipoproteinaemia, there is a defect in the synthesis of apo B; CM, VLDL and LDL are absent from the plasma. Clinically, there is malabsorption of fat, vitamins A, E and K, acanthocytosis (abnormal star-shaped red blood cells), retinitis pigmentosa and an cerebellar ataxia. 2. Hypobetalipoproteinemia: in this AD condition there is a partial deficiency of apoB due to a mutation in the apoB gene. LDL and cholesterol are decreased, but the level of TG and VLDL remain normal. 3. Chylomicron retention disease: in this condition there is a defect in apoB-48. The condition manifests in fat malabsorption and low circulating lipids. 2. Isolated low HDL 5 different diseases can lead to this condition: 1. Familial hypoalfalipoproteinemia: this AD condition is caused by a gene defect of hepatic lipase and apoA-I, IV/C-III. Low HDL levels are the manifestation with increasing risk for early CHD. 2. ApoA-I and apoC-III deficiency: this AR condition is caused by a mutation in one or more of these genes (including apoA-IV gene). Low HDL, cornea disease and an increased risk for early CHD are the features of this condition. 3. ApoA-I variants: this AR disease is caused by a specific mutation in the apoA-I gene, which leads to increased catabolism of this protein. Low HDL, cornea disease, increased risk for CHD and xanthomas are the main features.

Avi Sayag

Clinical Biochemistry

4. Tangier disease: In Tangier disease (AR), plasma HDL concentrations are reduced; clinically, the condition is characterized by hyperplastic, orange tonsils, the accumulation of cholesteryl esters in other reticuloendothelial tissues (hepatosplenomegaly), early CHD and low HDL. The condition is due to a loss of function mutation in the gene that codes for the protein ABCA1, which normally stimulates the uptake of cholesterol into HDL, thus leading to increased degradation of HDL. 5. LCAT deficiency (fish-eye disease): this AR disease is the result of a mutation in the LCAT (lecithin:cholesterol acetyltransferase) gene. Low HDL, glomeruloscelorosis, anemia, cornea disease and early CHD are the main features. Note: ApoA1 is a cofactor of LCAT. LCAT converts free cholesterol into cholesteryl ester (a more hydrophobic form), which is then sequenstered into the core of the lipoprotein eventually making the newly synthesized HDL spherical, and forcing the reaction to be unidirectional, since the cholesterol particles are removed from the surface. This enzyme is bound to HDL and LDL in the plasma.

Avi Sayag

Clinical Biochemistry

Topic 35
Risk factors of atherosclerosis
1. Gender: estrogen increases cholesterol removal by the liver, and the progression of atherosclerosis is less rapid in premenopausal women than in men. In addition, epidemiologic evidence shows that estrogen replacement therapy protects the cardiovascular system in postmenopausal women. On the other hand, large doses of estrogen increase the incidence of blood clots, and even small doses produce a slight increase in clotting. In addition, in several studies, estrogen treatment of postmenopausal women failed to prevent second heart attacks. The reason for the discrepancies between the epidemiologic and experimental data is currently unsettled. 2. The effect of increased plasma levels of homocysteine and related molecules such as homocystine and homocysteine thiolactone, a condition sometimes called hyperhomocystinemia, is associated with accelerated atherosclerosis. Markedly elevated levels resulting from documented mutations of relevant genes are rare, but mild elevations occur in 7% of the general population. The mechanism responsible for the accelerated vascular damage is unsettled, but homocysteine is a significant source of H2O2 and other reactive forms of oxygen, and this may accelerate the oxidation of LDL. Homocysteine is an intermediate in the synthesis of methionine. It is metabolized by enzymes that are dependent on vitamin B6, vitamin B12, and folic acid. Supplementation of the diet with these vitamins reduces plasma homocysteine, usually to normal.

3. Cholesterol, HDL and Tg: lowering plasma cholesterol and triglyceride levels and increasing plasma HDL levels slows, and in some cases reverses, the atherosclerotic process. The desired decrease in lipids can sometimes be achieved with dietary restriction of cholesterol and saturated fat alone, even though dietary restriction initiates a compensatory increase in cholesterol synthesis in the body. When dietary treatment is not adequate, reducing conversion of mevalonate to cholesterol with statins, drugs that inhibit hepatic 3-methylglutaryl coenzyme A (HMG-CoA) reductase, the enzyme which catalyzes this reaction, is beneficial (an example of an available HMG-CoA reductase inhibitor is simvastatin). 4. In cases in which there is severe hypercholesterolemia because of congenitally defective LDL receptors, gene therapy has been tried with promising preliminary results. (Antioxidant treatment with agents such as vitamin E, and -carotene has been used to inhibit oxidation of LDL, and this reduces the incidence of atherosclerotic changes in experimental animals. However, the results of antioxidant treatment in humans have generally been disappointing or negative). 5. Men who smoke a pack of cigarettes a day have a 70% increase in death rate from ischemic heart disease compared with nonsmokers, and there is also an increase in women. Deleterious effects of smoking include endothelial damage caused by carbon monoxide-induced hypoxia. 6. Because of the increased shear stress imposed on the endothelium by an elevated blood pressure, hypertension is another important modifiable risk factor for

Avi Sayag

Clinical Biochemistry

7.

8. 9.

10.

11. 12.

atherosclerosis. Lowering blood pressure has its greatest effect in reducing the incidence of stroke, but there are beneficial effects on ischemic heart disease as well. With modern methods of treatment, blood pressure in hypertensives can generally be reduced to normal or near-normal values. In diabetics, there are microvascular complications and macrovascular complications, which are primarily related to atherosclerosis. There is a twofold increase in the incidence of myocardial infarction compared with nondiabetics; severe circulatory deficiency in the legs with gangrene is relatively common; there are more thrombotic strokes, and renal failure is a serious problem. It is interesting in this regard that rigorous control of blood pressure in diabetics has been shown to be more efficacious in reducing cardiovascular complications than rigorous control of blood glucose. The nephrotic syndrome: also accelerates the progression of atherosclerosis. In the nephrotic syndrome there is increased hepatic production of lipids and lipoprotein(a). Lp(a): consists of an LDL-like particle and a specific apolipoprotein a, which is covalently bound to ApoB of the LDL-like particle. Apo(a) is expressed by hepatocytes and the assembly of Apo(a) and the LDL-like particle seems to take place at the outer hepatocyte surface. Lp(a) is highly homologous to plasminogen and to tPA, and it competes with plasminogen for its binding site, leading to reduced fibrinolysis. Also, because Lp(a) stimulates secretion of PAI-1, it leads to thrombogenesis. In addition, because of the LDL-like component, Lp(a) contributes to atherosclerosis. Obesity: obesity is associated with type 2 diabetes, hypertriglyceridemia, hypercholesterolemia, and hypertension, all of which are risk factors in their own merit. Family history of ischemic heart disease, stroke: probably multiple genetic mechanisms. Hypothyroidism: decreased formation of LDL receptors in the liver. The thyroid hormone regulates the synthesis of LDL receptor at the gene level (gene expression).

Avi Sayag

Clinical Biochemistry

Topic 36
Disturbances of uric acid metabolism
Gout is a disorder caused by the tissue accumulation of excessive amounts of uric acid, an end product of purine metabolism. It is marked by recurrent episodes of acute arthritis, sometimes accompanied by the formation of large crystalline aggregates called tophi, and chronic joint deformity. All of these result from precipitation of monosodium urate crystals from supersaturated body fluids. Although an elevated level of uric acid is an essential component of gout, not all such individuals develop gout, indicating that influences besides hyperuricemia contribute to the pathogenesis. Gout is divided into primary (90%) and secondary forms (10%). Elevated uric acid levels can result from overproduction of uric acid, reduced excretion, or both. Most cases of gout are characterized by a primary overproduction of uric acid. Uric acid synthesis: after the synthesis of purine nucleotides (guanosine monophosphate, adenosine monophosphate and inosine monophosphate) the catabolism produces uric acid. It should be noted that the pool of purine nucleotides is derived from dietary nucleic acids (300mg) and from endogenous synthesis (400mg de novo synthesis and tissue breakdown). The urate pool is mainly secreted by the kidney (75%) and partly by the GI (25%). In the kidney, 100% is filtered, then almost everything is reabsorbed in the proximal tubule, just to be secreted in the distal part of the proximal tubule (50%). Then, in the distal tubule 38-44% is reabsorbed, so the net urinary excretion is 6-12% of the amount filtered. Note that adenine does not form uric acid. Although the cause of excessive uric acid biosynthesis in primary gout is unknown in most cases, rare patients have identifiable enzymatic defects. For example, complete lack of HGPRT (hypoxanthine guanine phosphorybosyl-transferase) gives rise to the Lesch-Nyhan syndrome (complete loss of the enzyme secondary gout). This X-linked genetic condition is characterized by excessive excretion of uric acid, severe neurologic disease with mental retardation, and self-mutilation (but interestingly, little in the way of gout!). Because of the almost complete absence of HGPRT, purine nucleotide synthesis via the salvage pathway is blocked. This has two effects: an accumulation of PRPP (phosphoribosylpyrophosphate), a key substrate for the de novo pathway, and increased activity of amido-PRT (due to elevated PRPP and reduced feedback inhibition from purine nucleotides). As a consequence, de novo pathway purine biosynthesis is augmented, resulting eventually in excess production of the uric acid end product. Less severe deficiencies of HGPRT (partial deficiency primary gout) cause clinically severe gouty arthritis, occasionally associated with mild neurologic disease. In primary hyperuricemia, 10% is due to overproduction and the remaining 90% is due to underexcretion. In secondary gout, hyperuricemia can be caused by increased urate production (e.g., rapid cell lysis during chemotherapy for lymphoma or leukemia, increased purine intake) or decreased excretion (due to chronic renal insufficiency or administration of thiazides), or both. Hypothyroidism and hyperparathyroidism can also lead to renal retention. The risk for developing gout increases significantly when plasma urate is above 0.42 mmol/L in males and above 0.36 mmol/L in women. Above 0.54 mmol/L the risk is 90% for males. The precipitation of uric acid in the joints triggers a chain of events that lead to joint injury. Uric acid is less soluble in pH less than 5.75. The precipitated crystals are directly chemotactic, and can also activate complement to generate chemotactic C3a and C5a fragments. This leads to a local accumulation of neutrophils and macrophages in the joints and synovial membranes; in attempting to phagocytize the crystals, these cells become activated, leading to the release of additional mediators including chemokines, toxic free radicals, and leukotrienes, particularly leukotriene B4. The activated neutrophils also liberate destructive lysosomal enzymes. Macrophages participate in joint injury by secreting a variety of proinflammatory mediators such as IL-1, IL-6, and TNF. While intensifying the inflammatory response, these cytokines can also directly activate synovial cells and cartilage cells to release proteases (e.g., collagenase) that cause tissue injury. The resulting acute

Avi Sayag

Clinical Biochemistry

arthritis typically remits in days to weeks, even if untreated. Repeated bouts, however, can lead to the permanent damage seen in chronic tophaceous arthritis. Clinical Features Gout is more common in men than in women; it does not usually cause symptoms before the age of 30. Four stages are classically described: (1) asymptomatic hyperuricemia, (2) acute gouty arthritis, (3) "intercritical" gout, and (4) chronic tophaceous gout. Asymptomatic hyperuricemia appears around puberty in males and after menopause in women. In asymptomatic hyperuricemias, quatitation of urinary uric acid excretion is carried out. If it is less than 600 mg/day, the patient is an "underexcretor" and uricosuric drugs are administered (such as probenicid, sulfynpyrazone that block renal reabsorption). However, if the patient excretes more than 600 mg/day, then he is an "overproducer" and allopurinol is administered (inhibits xanthine oxidase). After a long interval of years, acute arthritis appears in the form of sudden onset of excruciating joint pain associated with localized erythema and warmth; constitutional symptoms are uncommon, except for possibly mild fever. The vast majority of first attacks are monarticular; 50% occur in the first metatarsophalangeal joint , and 90% in the instep, ankle, heel, or wrist. Untreated, acute gouty arthritis may last for hours to weeks, but it gradually completely resolves and the patient enters an asymptomatic intercritical period. Although some individuals never have another attack, most experience a second episode within months to a few years. In the absence of appropriate therapy, the attacks recur at shorter intervals and frequently become polyarticular. Eventually, after a decade or so, symptoms fail to resolve completely after each attack, and the disease progresses to chronic tophaceous gout. At this stage, radiographs show characteristic juxtaarticular bone erosion caused by the crystal deposits and loss of the joint space. Progression leads to severe crippling disease. Renal manifestations of gout can appear as renal colic associated with the passage of gravel and stones, and can evolve into chronic gouty nephropathy. About 20% of individuals with chronic gout die of renal failure. Numerous drugs are available to abort or prevent acute attacks of arthritis and mobilize tophaceous deposits. Their use is important because many aspects of gout are related to the duration and severity of hyperuricemia. The diagnosis of gout is primarily clinical. Hyperuricemia can be demonstrated, and confirmation is made by demonstrating tophi or monosodium urate in synovial fluid. It should be differentially diagnosed with rheumatoid arthritis and pseudogout. Hypouricemia less than 0.12 mM (less common). It can be secondary to severe hepatocellular disease (purine synthesis or xantine oxidase defect), to defective renal tubular reabsorption of uric acid, and to overtreatment with allopurinol. Hypouricemia can be caused by congenital deficiency of xantine oxidase (xantinuria).

Avi Sayag

Clinical Biochemistry

Topic 37
Lab diagnostics of AMI (practical topic 21 is included)
There are 2 interrelated types of MI, with different morphology, pathogenesis and clinical significance: Transmural infarct is an MI involving the full thickness of the ventricular wall; it is usually caused by severe coronary atherosclerosis, with acute plaque rapture and superimposed occlusive thrombosis. Subendocardial infarct is typically limited to the inner one third of the ventricular wall; it is caused by increased cardiac demand in the setting of limiting supply due to fixed atherosclerotic disease; alternatively, subendocardial infarction can occur in an evolving transmural infarct when the coronary obstruction is relieved in sufficient time to prevent transmural necrosis. The warning signs are pain in the jaw, neck, arms, shoulders or back, chest pressure, squeezing or pain, shortness of breath, nausea, sweating or feeling faint. The average time from onset of symptoms to presentation is 2.6 hours. The average time from presentation to treatment is 1.7 hours, while the recommended time is 30-60 minutes! Thus, shortening the time from onset of symptoms to therapy is critical. Pathogenesis Transmural infarcts Transmural infarcts are largely a consequence of coronary atherosclerosis and one (or more) disrupted plaques. Significant plaques typically occur in the proximal 2 cm of the LAD and left circumflex coronary arteries and in the proximal and distal thirds of the right coronary artery. In a few cases, vasospasm and platelet aggregation cause MIs without atherosclerotic stenosis. With sufficient collateral blood flow, even complete vessel occlusion will not necessarily result in MI. The initial event in most transmural MIs is erosion, ulceration, fissuring, rupture or hemorrhagic expansion (collectively called acute plaque change or disruption) of an atherosclerotic plaque. Plaques involved in coronary events typically have a large lipid pool, a thin fibrous cap, and a macrophage-rich inflammation; plaques with such features are considered susceptible to rupture, and are termed vulnerable. Patients at risk of cardiovascular events may have multiple vulnerable plaques. If the patient survives, thrombi may either lyse spontaneously or after fibrinolysis; alternatively, vasospasm may relax. In both cases, flow is reestablished and some myocardium is spared from necrosis. Reperfusion of precariously injured cells may restore viability but leave the cells poorly contractile (stunned) for 1-2 days. Nearly all transmural MIs affect the left ventricle; 15% simultaneously involve the right ventricle, particularly in posterior-inferior ventricle infarcts. Isolated right ventricle infarction occurs in 1-3% of cases. Subendocardial infarcts Subendocardial infarcts are usually caused by: Diffuse coronary atherosclerosis and global borderline perfusion made transiently critical by increased demand, vasospasm, or hypotension but without superimposed thrombosis. Plaque disruption with overlying thrombus that spontaneously lyses (or is removed by therapeutic intervention), thereby limiting the extent of myocardial injury. Myocardial injury is usually less than in a transmural infarct and often multifocal. Clinical features About 25% of patients experience sudden death after MI (presumably secondary to a fatal arrhythmia), most before reaching a hospital.

Avi Sayag

Clinical Biochemistry

Of those surviving, the risk of death within 1 month after MI is 7-10%, and 80-90% will develop complications. Early restoration of flow (thrombolysis or balloon angioplasty) through occluded vessels responsible for the infarction yields a better prognosis. Complications Complications of an MI depend on the size and the location of injury, as well as functional myocardial reserves. Overall mortality rate in the first year after MI is 30% and thereafter 5% to 10% per year. Typical complications include: Arrhythmias CHF Cardiogenic shock (usually seen when >40% of the left ventricle is infracted). Ventricular rapture Rarely, papillary muscle infarction with or without rupture can cause severe mitral regurgitation. Fibrinous pericarditis is common 2-3 days after MI (but is not usually clinically significant). Mural thrombosis is next to a noncontractile area, with risk of peripheral embolization. Stretching of a large area of transmural infarction (expansion) that may heal into a ventricular aneurysm; both are prone to mural thrombosis. Repetitive infarction (extension). Diagnosis Molecules released from damaged cells during MI: Ions and metabolites are released to the blood stream, while macromolecules are released to the lymphatic system. Cardiac proteins most commonly used in lab tests are myoglobin, cardiac troponin T and cardiac troponin I. Enzymes most commonly used: CK, CK-MB. Formerly, LDH, LDH isoenzymes and GOT (glutamate oxalacetate transaminase, aka aspartate aminotransferase, ASAT). Determination of more than one enzyme should be made, evaluation of the time curve of changes, and the use of an isoenzyme specific to the organ should be ensured. Future tests will include ischemia modified albumin, hsCRP, NT pro BNP and sCD40L and IL-6. Determination of creatine kinase CK Considering these 3 reactions, the activity of CK can be determined: Creatine-phosphate + ADP creatine + ATP (enzymatic reaction carried by CK) ATP + glucose glucose-6-P + ADP (carried by hexokinase) G-6-P + NADP+ 6-phosphogluconate + NADPH + H+ (carried by G6PDH) There is a linear correlation between the increase in the amount of NADPH and CK activity. NADPH has a maximum absorbance at 340nm. The test is carried out using serum sample, and a reagent is added. The reagent contains hexokinase, glucose, NADP+ and G6PDH. Hemolysis interferes with the determination of CK, because RBCs release adenylate cyclase (not CK!). AC converts 2 ADPs to ATP + AMP, and the ATP carries the second reaction, leading to overestimation of CK levels. The activity of CK is given by U/L one unit is the enzyme activity that converts 1 mol substrate in 1 minute. The reference interval measured at 37C: 24-195 U/L. CK can be elevated in the following settings: o IM injection o Damage to skeletal muscles (or exercise) o Hypo- and hyperthermia o Intoxication

Avi Sayag

Clinical Biochemistry

o CVA o Old age CK has 3 isoenzymes: CK-MM, CK-MB and CK-BB (there is also mitochondrial CK CK-MiMi). These are dimeric combinations of the brain (B) subunit and the muscle (M) subunit. The main source of CK-MB is cardiac muscle. It is released into the circulation upon tissue injury. In the heart, the total CK is 187 U/g. 40% is given by CK-MB and 60% by CK-MM (in the skeletal muscle, the total CK is 1093 U/g. 97% is given by CKMM and 3% by CK-MB). Normally, CK-MB is less than 6% in the serum (and the rest is CK-MM). CK-MB can be determined using the following methods: o Electrophoresis; o Immunoinhibition the M subunit is inhibited by a specific Ab and the residual activity is measured. Thus, after the inhibition of the M subunit in the MM and MB isoenzymes, only the activity of the B subunit of the CKMB will be measured. However, macroCK can occur in the serum leading to overestimation of the CK-MB level. Since the M subunit is inhibited, the measured CK activity is due to the B subunit of the CK-MB and thus the activity has to be doubled. This method is very sensitive and quick; o Determination of CK-MB mass concentration by immunoassay an expensive method, which is unable to give the result as a percentage of the total CK activity. However, it is very specific and precise, and thus it is the recommended method. Reference interval measured at 37C: CK-MB <6% of total CK, < 24 U/L CK enzyme also has isoforms, which are the result of post-translational modification. In the circulation, the C-terminal lysine is cleaved off by a plasmatic carboxypeptidase. Thus, 3 isoforms of CK-MM are present and 2 isoforms of CK-MB. CK-MM1 both M subunits lack the C-terminal lysine. CK-MM2 one M subunit lacks the C-terminal lysine. CK-MM3 both subunits retain their C-terminal lysine. CK-MB1 the M subunit lacks the c-terminal lysine. CK-MB2 the M subunit retains its C-terminal lysine. CK-MM3 and CK-MB2 are characteristic of AMI, because the post-translational modification cannot occur in the freshly released enzymes. The electrophoresis method is suitable for the detection of CK isoforms, but is not commonly used in routine diagnostics. macroCKs are large compounds of CK that are of two types: o type I: these are complexes of a CK isoenzyme (most frequently CK-BB) with immunoglobulins. They frequently occur among women older 50 years of age. They also occur in severe GI disease, vascular disease, adenomas and carcinomas. o Type II: these are oligomeric mitochondrial CK. It occurs mostly in severe malignancies or liver diseases. o macroCKs interfere with immunoinhibition methods of CK-MB determination. Determination of troponins Troponins are very useful in the diagnosis and prognosis of acute coronary syndrome. Normally, cTnT is not detectable in the blood of healthy persons. The 2 troponins (cTnI and cTnT) can be elevated even if CK-MB is not. This implies minor myocardial damage. Troponins are predictive of death when they are elevated, regardless whether CK-MB is elevated or not. Both troponin I and T have 3 isoforms: in slow twitch fibers and in fast twitch fibers of skeletal muscles, and in cardiac muscle. Most troponins in the heart are in their insoluble form, and only 4-5% are in the cytoplasm in their soluble form. It is probably the soluble form that is responsible for the fast elevation of troponins after AMI, while the insoluble form accounts for the sustained troponin increase.

Avi Sayag

Clinical Biochemistry

cTnT is an early marker of MI (but not the earliest, though). It remains elevated longer than CK-MB (troponin reaches its peak after 24 hours, it then decreases a bit in the next 24 hours, and remains elevated for additional 48 hours before it gradually decreases in the course of couple of days. CK-MB, on the other hand, reaches its peak after 24 hours from injury, to a lesser extent than troponin, and sharply decreases in the next 24 hours). cTnT is a sensitive marker, and it can be used for monitoring reperfusion during thrombolysis. Also, it can be used for sizing MI. It is more sensitive than CK-MB (troponin's level is elevated in cases of myocardial damage that is not defined as AMI by the WHO). The upper reference limit (URL) for troponin is 0.04 g/L. Values between 0.04-0.1 g/L suggest myocardial damage, whereas values above 0.1 g/L suggest AMI. cTn false positivity can be obtained in the following cases: o Heterophilic antibodies o HAMA (Human Anti-Mouse Antibodies) o Rheumatoid factor o Fibrin clot o Microparticles o Hemolysis o Lipemia o Icterus o Immunocomplex formation o Technical errors Troponin level is determined by immunochemical assays.

Myoglobin lacks cardiac specificity as a marker. That is, if the myoglobin is elevated 4-8 hours following the onset of pain, but the ECG shows no sign of cardiac condition, then more cardiac-specific markers should be sought for. However, if myoglobin is not elevated 4-8 hours following the onset of pain, myocardial necrosis can be excluded. That is, myoglobin is sensitive. It is the only early marker measured (2-6 hours after the onset of AMI and it decays within 10-16 hours), despite its specificity. Troponin level is determined by immunochemical assays. Determination of LDH (Lactate Dehydrogenase):

Avi Sayag

Clinical Biochemistry

LDH catalyses the reaction: pyruvate + NADH + H+ lactate + NAD+ (it also catalyses the reverse conversion). LDH activity correlates with the NADH consumption, which is monitored by measuring the rate of absorbance decrease at 340 nm. Hemolysed serum should not be used, since RBCs contain 150 times more LDH activity than serum! 5 isoenzymes o LDH-1 (4H) - in the heart and RBCs o LDH-2 (3H1M) - in the heart and RBCs o LDH-3 (2H2M) - in the lungs o LDH-4 (1H3M) - in the kidneys o LDH-5 (4M) - in the liver and striated muscle The relative amount of LDH isoenzymes in the serum: LDH2>LDH1>LDH3>LDH4>LDH5 Usually LDH-2 is the predominant form in the serum. A LDH-1 level higher than the LDH-2 level (a "flipped pattern"), suggests myocardial infarction (the normal ration LDH1/LDH2 is < 0.8). Following a myocardial infarction, levels of LDH peak at 3-4 days and remain elevated for up to 10 days. In this way, elevated levels of LDH can be useful for determining if a patient has had a myocardial infarction if they come to doctors several days after an episode of chest pain. Tissue breakdown elevates levels of LDH, and therefore a measure of it indicates e.g. hemolysis. Other disorders indicated by elevated LDH include cancer, meningitis, encephalitis, acute pancreatitis and HIV. It should be noted that the use of LDH (as well as GOT and total CK) is outdated. IMA Ischemia Modified Albumin: during ischemia, the N-terminus of albumin is altered (copper is released by localized ischemia, and in the presence of vitamin C they carry a fenton reaction yielding free radicals that alter the N-terminus of albumin) such that it can no longer bind metal, such as cobalt. When albumin circulating in the blood comes in contact with an ischemic tissue in the heart, some of the albumin molecules are converted to IMA. The levels of IMA are elevated among ischemic patients; it is continuously produced during ischemia; it rises very quickly and remains elevated during the ischemic event. o In the Albumin Cobalt Binding (ACB) test, cobalt is added to the serum sample. Cobalt normally binds to albumin, but not to IMA. The ACB test measures the unbound cobalt. Higher levels of unbound cobalt indicate greater concentrations of IMA o IMA rises rapidly from the end of balloon inflation (around 10 minutes). The National Academy of Clinical Biochemistry recommends that 60 minutes elapse from sampling to lab results. o Some other conditions lead to elevated IMA: some cancers, cirrhosis, and acute infections (free radicals are produced in these cases) as well as in brain ischemia (stroke) and end-stage renal disease.

Recommended time from presentation to treatment of AMI is 30-60 minutes.

Avi Sayag

Clinical Biochemistry

Topic 38
Hypervitaminosis and hypovitaminosis
Factors leading to vitamin deficiency: 1. inadequate intake (with normal requirement) 2. impaired absorption (general malnutrition or malabsorption) 3. impaired vitamin metabolism (if metabolism is necessary for function) 4. increased requirement 5. increased loss 6. therapy (e.g. hemodialysis or total parenteral nutrition) The classic deficiency syndromes are the end result of a process in which deficiency of a vitamin leads first to mobilization of body stores, then to tissue depletion, biochemical impairment (subclinical deficiency) and eventually to frank deficiency. The functions of vitamins are almost entirely intracellular, and their plasma concentrations do not necessarily reflect intracellular concentrations and thus functional availability. These are the most frequently used tests: i. B1: transketolase activity ii. B2: glutathione reductase iii. B6: aspartate aminotransferase activity iv. C: leukocyte ascorbate concentration; ascorbate in urine after vitamin C load. Vitamin B1 (thiamine) Thiamine pyrophosphate (TPP) is a cofactor in the metabolism of pyruvate (oxidative decarboxylation), carbohydrate metabolism, it's a coenzyme for active aldehydes (Mg is required), and in a reaction of the pentose shunt pathway catalyzed by the enzyme transketolase. In the nervous system, it participates in the hydrolysis of TPP at axonal membranes. The body contains only about 30 times the daily requirement of this vitamin. Diets high in carbohydrate require more thiamine for their assimilation than diets high in fat and so, for example, subclinical thiamine deficiency may be unmasked in malnourished patients given glucose intravenously. The source of the vitamin is cereals, liver, heart, kidney and pork meat. Deficiency of vitamin B1 causes beriberi, in which there are cardiac symptoms and neurological symptoms. The cardiac symptoms include o Peripheral vasodilation; o Retention of sodium and water ( edema); and o Myocardial failure. The neurological symptoms include o Peripheral neuropathy

Avi Sayag
o

Clinical Biochemistry

Wernicke's encephalopathy, characterized by ophthalmoplegia and ataxia and which may progress rapidly to stupor and death, and o Korsakoff's psychosis, of which memory loss is usually the most obvious feature. One method involves administration of a glucose load and measurement of the plasma pyruvate concentration. An excessive rise is seen in thiamin deficiency because the vitamin is a cofactor for the conversion of pyruvate to acetyl CoA. However, the most sensitive method, which will detect subclinical deficiency, is measurement of transketolase in a red cell haemolysate, the enzyme activity being measured both with and without the addition of thiamin pyrophosphate to the reaction mixture. Enzyme activity may be normal in subclinical deficiency but is increased by the addition of the coenzyme. If the deficiency is clinically obvious, the basal enzyme activity will be low. Vitamin B2 (Riboflavin) Riboflavin, when combined with 1 ATP yields FMN (flavin mononucleotide) and when combined with 2 ATPs it yields FAD (flavin adenine dinucleotide). It functions as a cofactor in redox reactions and as a prosthetic group of some enzymes. The source of the vitamin is meat, milk, eggs, liver, kidney, heart and vegetables. Deficiencies are mostly seen in alcoholic patients due to low intake of the vitamin. Clinically, deficiency manifests as glossitis, angular stomatitis (an inflammation of the mucous lining of any of the structures in the mouth), photophobic dermatitis, anemia (normochromic and normocytic) and burning sensation of the skin and eyes. Assessing riboflavin status involves measurement of the red cell enzyme glutathione reductase with and without the vitamin. Nicotinic acid (Niacin) Nicotinic acid is the precursor of nicotinamide. This is a constituent of the coenzymes nicotinamide adenine dinucleotide (NAD) and its phosphate (NADP), which are essential to glycolysis and oxidative phosphorylation. Part of the body's nicotinic acid requirement is met by endogenous synthesis from tryptophan. The dietary source is yeast, liver, poultry, milk, canned salmon, wheat and leafy vegetables. Deficiency causes pellagra or pellagra-like syndrome (carcinoid syndrome in which there is increased metabolism of tryptophan to hydroxyindoles which is consequently less available for nicotinic acid synthesis) that manifests as o weight loss o anemia o dementia o diarrhea o photosensitive dermatitis and o mental changes (apathy). Nicotinic acid status can be assessed by measurement of its urinary metabolites, although this is rarely necessary. Vitamin B6 (Pyridoxine) 2 active derivatives: pyridoxal phosphate and pyridoxamine phosphate The vitamin participates in o Amino acid metabolism (as a prosthetic group of aminotransferases and in decarboxylation of amino acids) o Synthesis of heme precursor delta-aminolevulinic acid (glycine + succinyl CoA); and o Synthesis of sphingomyelin precursor (serine + palmytoyl CoA). The source of the vitamin is meat, poultry, fish, yeasts and seeds.

Avi Sayag

Clinical Biochemistry

Deficiency in children leads to o Dermatitis o Glossitis o Nausea and vomiting o Epileptiform convulsions It should be noted that vitamin B6 is an antagonist of many drugs, including INH and penicillamines. Excess of vitamin B6 leads to o Peripheral neuropathy o Ataxia o Paresthesias o Muscle weakness Lab test: activity of aspartate aminotransferase in RBCs. Vitamin B12 (cobalamine) The vitamin has many functions including nucleic acid synthesis, methylation of homocystein (to yield methionine) and participation in synthesis of myelin sheath in the nervous system. The source of the vitamin is solely dietary (animal products). 2 molecules are necessary for its absorption: IF and TCII. Clinical conditions leading to deficiency: o Autoimmune destruction of parietal cells (no IF) o Gastrectomy (no parietal cells no IF) o Small intestinal disease (Crohn's disease, surgical resection) o Deficiency among veggies Deficiency causes: o Megaloblastic anemia o Subacute combined degeneration of the spinal cord (demyelination) Lab test: plasma B12 immunoassay. (B12 and folic acid are greatly elaborated in other topics, including pathology). Folic acid Folic acid functions as a carrier of hydroxymethyl and formyl groups, it is essential for synthesis of purines and pyrimidines and intimately related to B12 in replication of cellular genes and in maturation of RBCs. The source of folic acid is fruits and vegetables. Deficiency leads to megaloblastic anemia. Lab: plasma immunoassay (RBC is better, as the concentration in red cells reflects the body's folate reserves, while plasma concentrations reflect recent dietary intake). Vitamin C (Ascorbic acid) Ascorbic acid is essential for the hydroxylation of proline residues in collagen and thus for the normal structure and function of this protein. It acts by maintaining the iron in the hydroxylating enzyme in the reduced (Fe2+) state, that is, acting as an antioxidant. It also facilitates the intestinal absorption of dietary non-heme iron by keeping it in the Fe2+ state. Its source is citrus fruits and vegetables. Most animals can synthesize vitamin C from glucose. Deficiency of vitamin C leads to defective and weak collagen fibers, and the intercellular substance thus cannot withstand stress. Scurvy manifests in o Petechiae and purpura o Cutaneous bleeding and

Avi Sayag

Clinical Biochemistry

o Intracranial hemorrhages among infants o Poor wound healing o Salivary and lacrimal gland dryness o Anemia o Osteoporosis The concentration of ascorbate in plasma reflects recent dietary intake and is a poor index of tissue stores of the vitamin. These are better assessed by determination of ascorbate concentration in leukocytes. In practice, this is seldom necessary, since ascorbic acid is cheap and non-toxic, so a therapeutic trial of vitamin supplementation is the simplest procedure to confirm suspected vitamin C deficiency. Also, the vitamin can be measured in the urine after vitamin C loading. Vitamin A (Retinol) Its active derivatives are retinol, retinal and retinoic acid. This vitamin is a constituent of the retinal pigment rhodopsin. It is also essential for the normal synthesis of mucopolysaccharides and growth of epithelial tissue. Vitamin A is present in the diet and can also be synthesized from dietary carotenes. It is stored in the liver for 5-10 months. The normal liver contains considerable stores of the vitamin and deficiency is rarely seen in affluent societies. Deficiency causes o night blindness, while in more severe cases, degenerative changes in the eye may lead to complete loss of vision. o Dry conjunctiva (xerophthalmia) o Keratomalacia (ulcus cornea) o Keratinization in the skin Excess causes: o Dizziness o Headache o Abdominal pain o Vomiting o Skin desquamation within few days o If chronic: anorexia and hepatomegaly It can be measured in plasma, in which it is transported bound to prealbumin and a specific retinol-binding globulin. A low binding protein concentration can cause the plasma concentration of vitamin A to be low and impair its delivery to tissues even when hepatic stores of the vitamin are adequate. Vitamin D (Cholecalciferol D3) Its main function is in regulation of calcium homeostasis. Vitamin D is obtained from endogenous synthesis, by the action of ultraviolet light on 7-dehydrocholesterol in the skin to form cholecalciferol (vitamin D3), and from the diet. Dietary vitamin D is largely vitamin D2 (ergocalciferol); the only important dietary sources are fish and some margarines, which are artificially fortified with vitamin D. Vitamins D2 and D3 undergo the same metabolic changes in the body and have identical physiological actions. It is hydroxylated first in the liver to 25-hydroxycholecalciferol (25-HCC, calcidiol) and then in the kidney to 1,25-dihydroxycholecalciferol, and 24,25dihydroxycholecalciferol. The 24,25-Vit-D has no biological function, and this is the mechanism of the kidney to keep the levels of active vitamin D within acceptable limits. Recent evidence shows that 1-hydroxilation can also occur in target cells, bypassing the kidney for activation of the vitamin directly from the liver. Deficiency: o Rickets in children o Osteomalacia in adults Excess:

Avi Sayag
o o

Clinical Biochemistry
Hypercalcemia Hypercalcuria (predisposition to renal stones)

Vitamin E (Tocopherol) Vitamin E (tocopherol) is an important antioxidant, particularly in cell membranes, protecting unsaturated fatty acid residues against free radical attack. Its source is vegetable oils and plants. Clinical deficiency may occur in severe malabsorption, particularly in infants. Manifestations include hemolytic anemia and neurological dysfunction. In addition, there is a decrease in unsaturated fat level in cells, and abnormal structure and function of cellular organelles (mitochondria, lysosomes and cell membrane). Vitamin K Vitamin K is required for the -carboxylation of glutamate residues in coagulation factors II (prothrombin), VII, IX and X. This process confers physiological activity by permitting the binding of calcium to the proteins. Its source is vegetables and it is synthesized by bacteria in the colon. Vitamin K deficiency results in an increase in the prothrombin time, a functional assay of relevant coagulation factor activity. These factors are synthesized in the liver and the prothrombin time is also used as a test of liver function. Its most frequent use is in the monitoring of patients on anticoagulant treatment with antagonists of vitamin K (e.g. warfarin). Causes of deficiency: o Fat malabsorption o Antibiotics

Avi Sayag

Clinical Biochemistry

Topic 39
Lab diagnosis of hepatocellular damage; evaluation of liver function
Terminology: ALT alanine transaminase (also called serum glutamic pyruvic transaminase (SGPT) or alanine aminotransferase (ALAT): significantly elevated levels of ALT often suggest other medical problems such as viral hepatitis, CHF, liver damage, biliary duct problems, infectious mononucleosis, or myopathy. For this reason, ALT is commonly used as a way of screening for liver problems. However, elevated levels of ALT do not automatically mean that medical problems exist. Fluctuation of ALT levels is normal over the course of the day, and ALT levels can also increase in response to strenuous physical exercise. When elevated ALT levels are found in the blood, the possible underlying causes can be further narrowed down by measuring other enzymes. For example, elevated ALT levels due to liver-cell damage can be distinguished from biliary duct problems by measuring alkaline phosphatase. Also, myopathy-related ALT levels can be ruled out by measuring creatine kinase enzymes. Several drugs elevate the ALT levels. For example, Zileuton, the lipoxygenase inhibitor used in the treatment of acute asthma exacerbation. ALT is found only in the cytoplasm. Reference values: < 40 U/L. AST - Aspartate transaminase (also called serum glutamic oxaloacetic transaminase (S-GOT) or aspartate aminotransferase (ASAT): it is raised in acute liver damage, but is also present in red blood cells, and cardiac and skeletal muscle and is therefore not specific to the liver. The ratio of AST to ALT (GOT/GPT) is called deRitis ratio. In most primary liver diseases, the ALT and AST levels are elevated in roughly a 1:1 (because they are both present in the cytoplasma). The AST:ALT ratio is generally highest in alcoholic liver disease and lowest in acute and chronic viral hepatitis when the ratio may be less than 1:1. For patients with chronic alcohol-induced liver damage, the ratio is often greater than 2:1 (in cases of severe hepatocellular damage, both cytoplasmic and mitochondrial GOT are released, so the relative increase of GOT is higher than that of GPT). However, differences in laboratory methods limit the usefulness of the ratio. Another rule of thumb: if the ALT (GPT) is high (greater than 400 U/L), alcoholic liver disease is unlikely, regardless of the aminotransferase ratio. Elevated AST levels are not specific for liver damage, and AST has also been used as a cardiac marker. AST is found in the mitochondria and in the cytoplasm. ALP - Alkaline phosphatase (ALP) is an enzyme in the cells lining the biliary ducts of the liver. It works best at pH=10. ALP levels in plasma will rise with large bile duct obstruction, intrahepatic cholestasis or infiltrative diseases of the liver. ALP is also present in bones, intestines and placental tissue, as well as in tumors where it retains its activity even at 65C! As it is present in bones, it is higher in growing children (as their bones are being remodelled) and elderly patients with Paget's disease. Reference interval: 100-280 U/L. GGT - Gamma glutamyl transpeptidase: reasonably specific to the liver and a more sensitive marker for cholestatic damage than ALP, GGP may be elevated with even minor, sub-clinical levels of liver dysfunction (in biliary obstruction it can be elevated 30-fold). GGT is raised in alcohol toxicity (acute and chronic). It is also found in the cells lining the biliary ducts. Reference interval: 7-50 U/L. LDH Lactate dehydrogenase: LDH5 is the most abundant LDH isoenzyme in liver cells. Therefore, LDH5 activity and ratio are significantly increased in hepatocellular damage.

Avi Sayag

Clinical Biochemistry

Lab tests used for the diagnosis and monitoring of liver diseases and for evaluation of liver functions can be divided into the following groups: 1. Substances synthesized by the liver: i. Plasma proteins (albumin, haptoglobin, AFP) ii. Cholinesterase activity iii. Clotting factors activity 2. Substances metabolized by the liver: i. Drugs ii. Bilirubin metabolism and excretion iii. Cholesterol and triglycerides 3. Substances released from damaged hepatic tissue: i. Enzymes from damaged hepatocytes (GOT, GPT, LDH5) ii. Increased synthesis/enhanced excretion of other enzymes (GGT, ALP) 4. Substances in the plasma excreted by the liver: i. Endogenous metabolites (bile acids, bilirubin, ammonia) ii. Exogenous materials (lidocain, coffein, galactose) TESTS 1. Tests of cellular injury GPT, GOT and LDH5 are elevated. - In slight inflammation (minor injury) the deRitis ratio is reduced (i.e. GOT<GPT) since the mitochondria remains intact. - In more severe injuries the ratio increases as the mitochondria is injured as well (note again the location of the various enzymes). 2. Tests of liver function 1. Serum bilirubin: the 300 mg/day production of bilirubin are derived from Hb breakdown (80%) and from myoglobin breakdown (20%). Bilirubin can be unconjugated (bound non-covalently to albumin) or conjugated to glucoronic acid (10% to monoglucoronide and 90% to diglucoronide. During persistent conjugated hyperbilirubinemia, when the excretory function of the liver is impaired, a covalentlybound conjugated bilirubin fraction is found delta bilirubin). Bilirubin is water soluble when it is conjugated and in the cis form (and it is water insoluble in the trans form and when it is unconjugated). Reference range - < 17 mol/L (mainly unconjugated). Conjugation occurs in the ER. 2. Bromosulfophthalein tests (BSP test): a very sensitive and specific test of liver excretory function. 5 mg/Kg are administered IV. After 45 minutes, the percent of the BSP that remains in the blood is measured. Reference range: 0-5% - normal; 5-25% moderate dysfunction; 25-75% - severe; >75% very severe. This test is particularly useful in the setting of anicteric hepatitis, residual change after recovery from hepatitis, in cirrhosis and all stages of chronic hepatitis, and when fatty liver is suspected or when toxic liver damage is suspected. 3. Urobilinogen 4. Albumin concentration: reference range 30-60 g/L. 5. Clotting factors due to parenchymal damage: in moderately severe cases PT is elevated, and in severe cases PT, APTT and TT are elevated while the fibrinogen is lowered. In the setting of obstructive icterus PT and APTT are elevated. 6. Pseudocholinesterase activity: this enzyme rapidly metabolizes succinylcholine, thereby controlling its duration of action. This enzyme may be lowered in liver disease (hepatitis, cirrhosis). 7. Galactose tolerance test: this test is based on the ability of the liver to convert galactose to glycogen. It is measured by the rate of excretion of galactose following ingestion/IV injection of a known amount. Normally, less than 3 grams appear in the urine within 5 hours after the ingestion of 40 grams. 8. Lipid parameters

Avi Sayag

Clinical Biochemistry

9. Urea/Ammonia: hyperammonemia can be inherited (due to deficiency of urea-cycle enzymes) or acquired (due to renal failure or liver disease. Cirrhosis, hepatitis and Reye's syndrome may lead to this condition). 10. Bile acids: normal reference range 2-4 grams. The function of the bile acids is to facilitate cholesterol excretion, solubilize lipids (thus facilitating their absorption) and to activate pancreatic enzymes. Most of the bile acid is primary bile acid (cholic acid and chenodeoxycholic acid). Around 30% is secondary bile acids (deoxycholate and lithocholate). Prior to the secretion of primary bile acids, they are conjugated with glycine or taurine to increase their solubility in water. The endogenous microflora converts the primary acids into the secondary form. 95% of the bile pool is recycled (that is, it is reabsorbed in the distal ileum by active transport, and in the jejunum and colon by diffusion). It is decreased when bile acid synthesis is defective, and in cases of resection/inflammation/bypass of the ileum. It is elevated (by 50%) in fasting, following a meal, and in cholestasis. 11. Serum iron and copper: the liver synthesizes transferrin and stores iron in ferritin/hemosiderin. In hepatic icterus the iron is elevated and in obstructive icterus its levels are normal. The liver also incorporates copper into ceruloplasmin (the excess is excreted in the bile). In hepatic icterus the copper is normal and in obstructive icterus its levels are elevated.

Avi Sayag

Clinical Biochemistry

Assay of serum bilirubin. Detection of bilirubin and UBG in the urine (practical topic 22)
Assays of serum bilirubin: There are 4 methods for the determination of serum bilirubin: 1. Direct spectrophotometry: bilirubin concentration is proportional to the absorbance of the sample measured at 455nm. If we substract the absorbance measured at 575nm, we will eliminate the interference of HbO2 (HbO2 has about the same absorbance at both wavelengths, while bilirubin absorbs only at 455nm). This method can be used only in newborn, since they do not have any interfering compounds. 2. Jendrassik-Grof method: the principle is based on the photometric measurement of a color product. This method can be automated and is widely used. Bilirubin reacting with diazo-sulfanylic acid is transformed into a color product called azobilirubin. There is a linear correlation between the absorbance of the color product at 580nm and the concentration of bilirubin. (The conjugated bilirubin reacts with the diazosulfanylic acid, while the unconjugated form reacts with it only in the presence of coffein). Reference interval: <20 mol/L. 3. HPLC: can be used only as a reference method (time consuming). 4. Dry chemistry method: for this method we use slides with 3 layers the top layer consists of Na-benzoate to dissociate the albumin from bilirubin. The middle layer contains gelatine to bind serum proteins, and the bottom layer contains a cationic polymer that binds bilirubin. The amount of both forms of bilirubin can be measured by spectrophotometry. Bilirubin in the urine: No bilirubin can be found in the urine of healthy individuals. In diseases, conjugated bilirubin can be found in the urine. There are 2 principal tests: 1. Rosin test: we put 1% alcoholic iodine solution on the urine sample. If bilirubin is present, a green ring is visible on the border of the 2 liquids. 2. EL-U-TEST test strip: the detection is based on the reaction of bilirubin with diazonium salt. 1 minute after immersing the strip in the urine sample, we compare the color of the strip to that on the box. UBG in the urine: There are 2 principal tests: 1. Ehrlich's reaction: UBG and stercobilinogen form a red color when reacting with Ehrlich's reagent. The sample should be fresh and cooled to room temperature. The color should be pink. Red color implies increased levels. 2. EL-U-TEST: same as for bilirubin. The color of the scale turns from light pink to red, depending on the concentration of UBG. Normal values are above 1.7 mol/L, the borderline value is higher than 17mol/L, and the result is definitely pathological above 68mol/L. False negative results can be obtained if the sample is stored for a long time (especially when exposed to light) due to the oxidation of UBG.

Avi Sayag

Clinical Biochemistry

Topic 40
Lab diagnosis of cholestasis and liver cirrhosis
High pressure in the bile duct and bile tree induces enzyme synthesis. Since the bile secretion is blocked, the enzymes flow back into the systemic circulation. The tests performed in cholestasis are: 1. ALP the optimal pH for the enzyme is 9-10.5. It has several isoenzymes: tissue nonspecific (liver, bone and kidney), intestinal, placental, germ-cell and Regan (in some tumors). In cholestasis, special forms of ALP may appear (high molecular weight ALP, ALP complexed to lipoprotein X and intestinal ALP). The reference interval is 100-280 U/L. 2. GGT this enzyme is a good indicator of cholestasis. It may also be elevated in alcohol abuse, when anticonvulsive drugs are taken, and when rifampicin is taken. Reference interval: 7-50 U/L. 3. Lipoprotein X this lipoprotein is detected in cholestasis with a sensitivity of 0.57. It is composed of high amount of free cholesterol (30%) and phosphatidylcholine (60%). TG and cholesteryl-ester are low (2-5%). Albumin, APO-C and APO-E are also present, and its density resembles LDL. In agar gel electrophoresis it migrates to the cathode, unless LCAT and APO-A1 are added, which reduce its size and cause it to move toward the anode. Since the composition is similar to the bile, LpX represents bile extravasated to the plasma secondary to cholestasis. 4. Bilirubin is elevated (see previous topic) 5. Bile acids are elevated (see previous topic) In liver cirrhosis: Reduced: albumin, cholesterol synthesis and cholinesterase. Elevated: PT, ALP, GGT, AST, ALT, LDH5, ammonia, bilirubin, bile acids. deRitis ratio is elevated. The cause of liver cirrhosis should be elucidated: In 40% of cases, the cause is alcoholism: in acute alcohol abuse, ethanol can be measured in body fluids (very rapidly cleared). In chronic alcohol abuse, GGT, GOT, and GPT are elevated. These markers, though organ-specific, tell little about the etiology of the disease. They are not sensitive, and detect alcohol abuse when it results in organ damage. Another marker is carbohydrate transferrin (sialo-Tf). Most of the Tf is in the isoform of tetrasialo-Tf or trisialo-Tf (98%). The rest is in the asialo-Tf, monosialo-Tf or disialo-Tf. The level of these Tf is elevated in chronic excessive alcohol intake (60g for 30 days), and there is a strong correlation between the levels measured and the amount of alcohol intake. The levels return to normal after 10-18 days. In 30% of cases, the cause is hepatitis B infection: transaminases are elevated, PT is elevated and albumin is low. Hepatitis B is diagnosed by serology to detect HbsAg and core Ab (HbcAb). Detection of HbsAg present for more than 6 months implies chronic carrier state. Drugs and toxins: examples are paracetamol, tetracyclins, methotrexate, rifampicin, methyl-dopa, isoniazid. Wilson's disease: the problem lies in copper deposition. Thus, ceruloplasmin is < 300 mg/L and serum copper is elevated. Hemochromatosis: can be primary or secondary. Serum Fe is elevated, Tf saturation is 80-100%, ferritin levels are high, and definitive diagnosis can be made with molecular biology. Primary biliary cirrhosis: elevated ALP and transaminases, as well as the presence of autoantibodies (antimitochondrial, antinuclear, smooth muscle autoAb).

Avi Sayag

Clinical Biochemistry

Topic 41
Pathobiochemistry and lab diagnosis of the GI tract
6 main pathologies should be considered: 1. Peptic ulcer disease a. Chronic duodenal ulcer b. Chronic benign gastric ulcer c. Zollinger-Ellison syndrome (hereinafter "Z-E syndrome") 2. Gastritis a. Erosive gastritis b. Non-erosive gastritis 3. Gastric Cancer The main diagnostic tools are endoscopy and contrast radiography (but not for Z-E syndrome). 3 main lab tests should be carried out: 1. Gastric acid secretion The acid in fluid aspirated through a nasogastric tube is measured in the resting state to determine the basal stimulation and after the administration of pentagastrin (an analogue of gastrin) to determine the maximal secretion. Reference values: for males, the basal value should be less than 10 mmol/h and the maximal value should be less than 45 mmol/h. For women, the basal value should be less than 6 mmol/h and the maximal one should be less than 35 mmol/h. 3 pathologies can be inferred from these values: 1. Achlorhydria: in this condition the basal value is low (and sometimes even undetected), and there is no response to pentagastrin. But when the gastric antrum is intact in achlorhydria or in renal failure, the gastrin concentration is normal, because the G cells are intact (they secrete gastrin). Several conditions may lead to achlorhydria: autoimmune disorders where there is antibody production against parietal cells which normally produce gastric acid; the use of antacids or drugs that decrease gastric acid production (such as H2-receptor antagonists) or transport (such as proton pump inhibitors, e.g. omeprazole); a symptom of rare diseases such as mucolipidosis (type IV; a symptom of Helicobacter pylori infection which neutralizes and decreases secretion of gastric acid to aid its survival in the stomach; a symptom of pernicious anemia, atrophic gastritis, VIPomas (see next topic) or of stomach cancer; radiation thrapy involving the stomach; and gastric bypass procedures such a Duodenal Switch, where the largest acid producing parts of the stomach are either removed, or blinded. Thus, in this condition it is obvious that nothing will cause the stimulation of the parietal cells). 2. Chronic duodenal ulcer: the peak acid output is increased in this condition. 3. Chronic benign gastric ulcer: the peak gastric output is normal in this condition. 2. Determination of gastrin level This test is good for the diagnosis of Z-E syndrome. Z-E syndrome is a disorder where increased level of gastrin is produced, causing the stomach to produce excess hydrochloric acid. Often the cause is a tumor (gastrinoma) of the duodenum, pancreas or stomach (a triade) producing more gastrin. Gastrin then causes an excessive production of acid which can lead to peptic ulcers (in almost 95% of patients). The diagnosis is made by these lab tests: secretin stimulation test, which measures gastrin levels in response to secretin, fasting gastrin levels and antral acidity. High serum gastrin levels are diagnostic of Z-E syndrome (normal plasma gastrin is less than 50 ng/L. In Z-E syndrome it exceeds 200 ng/L). In uncertain cases, the secretin provocation test10 is applied.

A test that measures the ability of the pancreas to respond to secretin. Secretin is a hormone secreted by the small intestines in the presence of partially digested food from the stomach and stimulates the pancreas to secrete bicarbonate to neutralize stomach acidity. A tube is passed through the nose into the duodenum. Secretin is administered and the contents of the duodenal secretions are aspirated and analyzed over a period of about 2 hours.

10

Avi Sayag

Clinical Biochemistry

60% of all cases of Z-E are malignant, and 30% are MEN I (Multiple Endocrine Neoplams). The symptoms of the disease are ulcers (due to hypersecretion of acid), diarrhea, epigastric pain, steatorrhea (due to inactivation of lipases, as the low pH in the small intestine inactivates these enzymes), and B12 is not absorbed (pernicious anemia). 3. Detection of H. pylori infection a. C13-urea breath test If H. pylori is suspected, this test is performed. The patient arrives after an overnight fasting, and his exhaled air is collected in a tube to determine the basal value of CO2 in his breath. Patients swallow 75 mg urea labeled with an uncommon isotope in 150 ml apple juice (either radioactive C14 or non-radioactive C13). In the subsequent 10-30 minutes, the detection of isotope-labeled CO2 in exhaled breathe indicates that the urea was split; this indicates that urease (the enzyme that H. pylori uses to metabolize urea) is present in the stomach, and hence that H. pylori bacteria are present. The 14CO2/13CO2/12CO2 is analyzed using GC/MS (HPLC). Diagnosis of H. pylori is highly important, as is it associated with chronic active gastritis, peptic ulcer, MALT lymphoma and stomach carcinoma. Diseases of the intestinal tract 1. Malabsorption

Triolein breath test If fat malabsorption is suspected, this test is performed. The patient fasts overnight. A basal sample of expired CO2 is collected. Then, the patient is given isotopically labeled triolein in a 30-gram fat meal, and samples of expired CO2 are collected hourly for 7 hours. The isotope is measured in the CO2 samples. 2. Diarrhea: excess water in the feces due to long exposure to water that exceeds the reabsorption capacity of the colon (3.8L/day), intake of osmotically active substances that are not absorbable, deranged motility, and exudation of mucus, blood and protein due to inflammation of the intestines. 3. Ulcerative colitis and Crohn's disease 4. Celiac disease: In this condition, there is damage to the mucosa of the small intestine resulting in malabsorption. Symptoms include weight loss, bloating, flatulance, diarrhea, and the feces is large in volume, pale, watery and malodorous. Since malabsorption is a feature of the disease, some biochemical alterations may occur in folic acid, B12, iron (these 3 lead to anemia), calcium and vitamin D (leading to osteomalacia, osteoporosis and secondary

Avi Sayag

Clinical Biochemistry

hyperparathyroidosis) and fat-soluble vitamins (leading to neurological problems). Lab tests performed include: Anti-gliadin Ab (IgA and IgG in ELISA) Anti-reticulin Ab Anti-endomisium Ab Anti-transglutaminase Ab IgA Gut biopsy Molecular genetic examination (HLA DQ type) Rate of T cells. (The problem in this condition is sensitivity to gluten. The damage is immune-mediated against the mucosa of the small intestine). Diagnosis of intestinal disorders: 1. General screening tests should include albumin, Ca+2, B12 and peripheral blood smear. 2. Xylose absorption test: a variety of tests involving the ingestion of carbohydrates and the measurement of their plasma concentrations or urinary excretion have been developed for the investigation of small intestinal function. The best known is the xylose absorption test, which involves the administration of a test dose of d-xylose, a plant sugar. This is absorbed from the jejunum without prior digestion. It is only partly metabolized in the body, mainly excreted unchanged in the urine, where it can be measured. Accurately timed urine collection is essential. Misleadingly low results are obtained if the glomerular filtration rate is decreased, as occurs in renal failure and many normal elderly people. Other factors that can produce misleading results include delayed gastric emptying, edema and obesity. An alternative approach is to measure serum xylose concentration 60 min after administering xylose. 3. Disaccharaide test: suspected intestinal disaccharidase deficiency can be investigated by administering the appropriate disaccharide (50 g) orally and measuring the blood glucose response. If the result is abnormal, specificity can be improved by comparing the result with that obtained following administration of the equivalent quantities (25 g each) of the constituent monosaccharides. The test is unphysiological because of the large oral load of, for example, lactose, although patients who do not develop symptoms during the test are not lactose intolerant. 4. Breath Hydrogen test: a more reliable test is to measure breath H2 after giving the disaccharide: because it is not absorbed, the disaccharide reaches the colon where one of the products of bacterial fermentation is H2. The definitive investigation for disaccharidase deficiencies is measurement of the appropriate enzyme in a biopsy sample. 5. Triolein breath test (see above) 6. Fecal fat test 7. Fecal amino acid measurement 8. Gut biopsy 9. Specific tests such as occult blood (colon cancer), CEA, Schilling test 10. Radiological examination

Avi Sayag

Clinical Biochemistry

Topic 42
Lab diagnosis of acute pancreatitis
There are 2 forms of acute pancreatitis: the interstitial (edematous) form, which is the milder form with minimal organ damage and usual full recovery, and the hemorrhagic form (necrotizing), which is more serious with organ failure and local complications (pseudocyst formation, ascites, pleural effusion and pancreatic/peripancreatic abscesses). Acute pancreatitis usually occurs in middle-aged people as a sudden acute abdominal pain. The pain usually occurs after the ingestion of a large meal, or the consumption of alcohol, and is referred to the back. Due to electrolyte imbalance, bradykinin and prostaglandins are released, which may lead to hypotension and circulatory shock. Toxic psychosis and fever are also features of the disease. Mild jaundice may accompany the condition, as the inflamed pancreas compresses the bile duct. Causes: 1. Metabolic causes: alcohol, drugs, hypercalcemia, hyperlipoproteinemia and genetics. 2. Mechanical: gallstones, ERCP (endoscopic retrograde cholangiopancreatography), abdominal injury, perioperative injury. 3. Infections: mumps, coxackievirus and Mycoplasma pneumonia 4. Vascular: atheroembolism and polyarteritis nodosa Alcohol, viruses, drugs, ischemia and trauma cause direct acinar cell injury. Duodenal reflux leads to accumulation of lecithin in duodenal content. Lecithin is converted into lysolecithin which causes direct acinal cell injury as well. Cholelithiasis, chronic alcoholism and obstructing lesions lead to duct obstruction. Metabolic causes, alcohol and duct obstruction cause deranged intracellular transport. Pathomechanism: Intercellular leak of enzymes, the release of intracellular enzymes and activation of enzymes be lysosomal hydrolases all activate enzymes. The activated enzymes cause inflammation, proteolysis of pancreatic substances (by proteases), fat necrosis (by lipases) and necrosis of vessels with hemorrhages (by elastases). All these manifest as acute pancreatitis. The lab tests performed: 1. Serum -amylase 2. Urine amylase 3. Serum lipase can also appear in the urine, but only traces (insignificant). 4. Pancreatic isoamylase in cases of acute pancreatitis in patients with renal failure, and for the diagnosis of the coexistence of mumps and acute pancreatitis. 5. Immunoreactive trypsin 6. Serum elastase -amylase is not specific for the pancreas, and it also occurs in the salivary glands, in the gonads, in the Fellopian tube, in the small intestine, in striated muscles, in adipose tissue, and in the lungs. In the serum, amylase P and S are found (2 isoenzymes: P for pancreatic form, and S for salivary gland form). -amylase is also present in the urine, in ascites and in pleural effusion. Its function is to digest starch (hydrolyzes the -1,4 glycosidic bond), and it is cleared through the kidney. Amylase activity is increased in the following conditions: 1. In acute pancreatitis, its levels are elevated to more than 10 times the upper range limit (reference range: 21-101 U/L) 2. In perforated peptic ulcer, tumors of the lung and ovaries and ruptured ectopic pregnancy, its levels are elevated to more than 5 times the URL. 3. In acute abdominal disorders (such as acute cholecystitis, intestinal obstruction, mesenteric infarction and peritonitis), acute alcoholic intoxication, salivary gland disorders, neoplastic hyperamylasemia, severe glomerular impairment, opiate administration, diabetic ketoacidosis and macroamylasemia, its levels are elevated but rarely more the 5 times the URL.

Avi Sayag

Clinical Biochemistry

In acute pancreatitis, the ratio between the clearance of amylase and creatinine is also elevated (normally, it is 2-5%). In renal insufficiency, though, the ratio is normal (because both Cam and Ccr are low proportionally). Serum amylase starts to rise 2-12 hours after onset of acute pancreatitis, it peaks after 12-48 hours and returns to normal after 3-4 days. Urine amylase starts to rise 2-12 hours after onset of acute pancreatitis, it peaks after 24-72 hours and returns to normal after 5-7 days. Serum lipase starts to rise 4-8 hours after onset of acute pancreatitis, it peaks after 24-48 hours and returns to normal after 7-14 days. The sensitivity of -amylase alone in the diagnosis of acute pancreatitis is 80%, but when the test is combined with lipase the sensitivity rises to 94%. Lipase is produced in the pancreas and hydrolizes emulsified triglycerides (it cleaves the fatty acids from the glycerol). Pancreatic colipase is a coenzymes of lipase. It is mainly found in the serum, but traces can be also found in the urine. The RES probably clears this enzyme. The reference range is 0-190 U/L. Further lab findings in acute pancreatitis include: Coagulation disorders (DIC) Hypocalcemia Hypoglycemia and hyperglycemia Glycosuria Hypoproteinuria Albuminuria Increased serum urea concentration

Avi Sayag

Clinical Biochemistry

Topic 43
Clinical biochemistry of hypothalamus and hypophysis
For details about the hormones and functional structure of the pituitary and hypothalamus please consult Histology Avi's notes, page 1 (or you can use Ross directly, but obviously I recommend mine ). Growth Hormone (STH) This hormone promotes growth. Therefore, it Increases protein synthesis; Increases synthesis of glucose by the liver Increases lypolysis and Decreases uptake of glucose by the tissues Remember that this hormone counteracts the actions of insulin. Growth hormone releasing hormone (GHRH) is secreted from the hypothalamus and stimulates the secretion of GH from the adenohypophysis. Somatostatin inhibits its production (also secreted from the hypothalamus). Somatostatin has many other actions both within the hypothalamo-pituitary axis and elsewhere. For example, it inhibits the release of thyroid-stimulating hormone (TSH) in response to thyrotrophinreleasing hormone (TRH) and it is present in the gut and pancreatic islets, where it inhibits the secretion of many gastrointestinal hormones including gastrin, insulin and glucagon. GH stimulates the liver to produce insulin-like growth factor-1 (IGF-1), also known as somatomedin-C. IGF-1 has homology with insulin and shares some of the actions of this hormone. IGF-1 exerts negative feedback at the level of the pituitary, where it modulates the actions of GHRH, and at the level of the hypothalamus where, together with GH itself, it stimulates the release of somatostatin (that inhibits the release of GH). The concentration of GH in the blood varies widely through the day. Physiological secretion occurs in sporadic bursts, lasting for 1-2 h, mainly during deep sleep. The rate of secretion increases from birth to early childhood and then remains stable until puberty, when a massive increase occurs, stimulated by testosterone in males and estrogens in females; thereafter the rate of secretion declines to a steady level before falling to low levels in old age. Secretion can be stimulated by stress, exercise, a fall in blood glucose concentration, fasting and ingestion of certain amino acids. Diagnosis: stimuli can be used in provocative tests for diagnosing GH deficiency. GH secretion is inhibited by a rise in blood glucose and this effect provides the rationale for the use of the oral glucose tolerance test (OGTT) in the diagnosis of excessive GH secretion (a suppressive test). Insulin hypoglycemia test (see Marshall page 138 for protocol) is a stimulating test. Pathology: excessive secretion (usually due to a pituitary tumor) causes gigantism in children and acromegaly in adults; 95% of cases are due to pituitary tumor/MEN1 and 5% are due to ectopic GHRH secretion. As IGF-I is also elevated, it causes abnormal glucose tolerance, DM, CHF (due to hypertension) and osteoporosis. Deficiency causes growth retardation in children and can cause fatigue, loss of muscle strength, impaired psychological wellbeing and an adverse cardiovascular risk profile (elevated plasma total and LDL-cholesterol concentrations and hyperfibrinogenaemia) in adults, and hypoglycemia (as insulin will work unopposed). Tests include: random GH, GH after OGTT, IGF-1 and prolactin level. Prolactin Functions To initiate and sustain lactation A role in breast development Inhibits the synthesis of GnRH (at high concentrations, therefore it inhibits ovulation and spermatogenesis). Control: dopamine secreted from the hypothalamus inhibits the release of prolactin. Thyroid releasing hormone (TRH) and VIP (vasoactive intestinal polypeptide secreted from the GI) stimulate secretion of prolactin. Physiologically, pregnancy and breast-feeding stimulate its secretion, as well as stress and sleep. Its secretion

Avi Sayag

Clinical Biochemistry

increases during pregnancy, but concentrations fall to normal within approximately seven days after birth if a woman does not breastfeed. With breast-feeding, concentrations start to decline after about three months, even if breast-feeding is continued beyond this time. Pathology: causes include o Physiological: stress, sleep, pregnancy and suckling o Drugs: dopaminergic receptor blockers, dopamine antagonists, pituitary tumors, trauma. o Clinical features: females: oligomenorrhea, amenorrhea, infertility, galactorrhea11; males: impotence, infertility, gynacomastia. Thyroid stimulating hormone (TSH) Thyroid-stimulating hormone (TSH) is composed of an - and a -subunit; the amino acid composition of the -subunit is common to TSH, the pituitary gonadotrophins and human chorionic gonadotrophin (hCG), but the -subunit is unique to TSH. The release of TSH is regulated at the level of the hypothalamus and by the thyroid gland itself: TRH from the hypothalamus stimulates its secretion, while somatostatin and dopamine inhibit its secretion. If plasma concentrations of thyroid hormones decrease (T3 and T4), TSH secretion increases, stimulating thyroid hormone synthesis; if they increase, TSH secretion is suppressed (negative feedback). Pathology: in primary hypothyroidism, TSH secretion is increased; in hyperthyroidism it is decreased. TSH deficiency can cause hypothyroidism but hyperthyroidism due to TSH-secreting tumours is rare. Further details in topic 44. Gonadotrophins (FSH and LH) FSH and LH both consist of two subunits: the -subunits are unique to each hormone but the amino acid sequence of the -subunits is the same, and is found in both TSH and hCG. The synthesis and release of both hormones are stimulated by the hypothalamic decapeptide, gonadotrophin-releasing hormone (GnRH), and their effects are modulated by circulating gonadal steroids. GnRH is secreted episodically, resulting in pulsatile secretion of gonadotrophins with peaks in plasma concentration occurring at approximately 90-min intervals. In males, LH stimulates testosterone secretion by Leydig cells in the testes: both testosterone and estradiol, derived from the Leydig cells themselves and from the metabolism of testosterone, feedback to block the action of GnRH on LH secretion. FSH, in concert with high intratesticular testosterone concentrations, stimulates spermatogenesis; its secretion is inhibited by inhibin, a hormone produced during spermatogenesis. In females, the relationships are more complex. Estrogen (mainly estradiol) secretion by the ovaries is stimulated primarily by FSH in the first part of the menstrual cycle; both hormones are necessary for the development of Graafian follicles. As estrogen concentrations in the blood rise, FSH secretion declines until estrogens trigger a positive feedback mechanism, causing an explosive release of LH and, to a lesser extent, FSH. The increase in LH stimulates ovulation and development of the corpus luteum, but rising concentrations of estrogens and progesterone then inhibit FSH and LH secretion; inhibin from the ovaries also appears to inhibit FSH secretion. If conception does not occur, declining concentrations of estrogens and progesterone from the regressing corpus luteum trigger menstruation and LH and FSH release, initiating the maturation of further follicles in a new cycle. Before puberty, plasma concentrations of LH and FSH are very low and unresponsive to exogenous GnRH. With the approach of puberty, FSH secretion increases before that of LH.

11

A spontaneous flow of milk from the breast associated with breastfeeding.

Avi Sayag

Clinical Biochemistry

Adrenocorticotrophic hormone (ACTH) ACTH stimulates adrenal glucocorticoid (and mineralocorticoids to a lesser extent) secretion. ACTH is a fragment of a much larger precursor, pro-opiomelanocortin (POMC), which is the precursor not only of ACTH but also of -lipotrophin, itself the precursor of endogenous opioid peptides (endorphins). ACTH release is controlled by CRH. ACTH secretion is pulsatile and also shows diurnal variation, the plasma concentration being highest at approximately 0800 h and lowest at midnight. Secretion is greatly increased by stress and is inhibited by cortisol. Thus, cortisol secretion by the adrenal cortex is controlled by negative feedback, but this and the circadian variation can be overcome by the effects of stress. Diagnosis: dexamethason suppresses ACTH and insulin stimulates its secretion. Pathology: increased secretion of ACTH by the pituitary is seen with pituitary tumors (Cushing's disease) and in primary adrenal failure (Addison's disease). The hormone may also be secreted ectopically by non-pituitary tumors (mainly lung cancer). Excessive ACTH synthesis is associated with increased pigmentation, owing to the melanocyte-stimulating action of ACTH and other POMC-derived peptides. Decreased secretion of ACTH may be an isolated phenomenon, but is more commonly associated with generalized pituitary failure. There 4 conditions that should be differentially diagnosed from pituitary tumor. In pituitary tumor, the basal cortisol is elevated, dexamethason test is positive, ACTH is elevated and CRH is positive. This combination is unique to pituitary tumor (CRH is only positive in pituitary tumor).

Problems that arise in determining the reference intervals for these hormones stem from differences in levels among males and females, from circadian changes, from cycles of these hormones and from the methods used to define the reference range. In hypopituitarism, the decrease in the synthesis of hormones is in the following order: GH>FSH/LH>ACTH> TSH (G-FLAT). A combined decrease is more frequent than an isolated decrease. The order of frequency with which hormone secretion occurs in patients with pituitary tumors is prolactin>GH>ACTH>FSH/LH>TSH The posterior pituitary gland secretes ADH and oxytocin, synthesized in the hypothalamus in the supraoptic nucleus and in the paraventricular nucleus. ADH (aka vasopressin) Uncontrolled ADH secretion leads to SIADH (see topic 27). Decreased ADH secretion leads to diabetes insipidus that can be either: Cranial diabetes insipidus (CDI): due to tumor, trauma, meningitis, encephalitis, and familial CDI. Nephrogenic diabetes insipidus (NDI): due to metabolic hypokalemia, hypercalcemia, drugs, chronic renal disease and familial NDI.

Avi Sayag

Clinical Biochemistry

Diabetes insipidus is diagnosed by the fluid deprivation test. The patient is allowed a light breakfast with no fluid, and no smoking is permitted. The patient is then weighted. Fluids are deprived for 8 hours. Every hour the patient is weighted, the osmolality and volume of urine are measured, and the plasma osmolality is measured. After 8 hours, the patient is allowed to drink and desmopressin (a synthetic analogue of ADH) is administered intranasally (according to the book, I.M). Finally, the urine osmolality is measured during 4 hours. In a normal subject, the urine becomes concentrated in response to fluid deprivation and plasma osmolality does not exceed 295 mmol/kg. In diabetes insipidus, the urine does not become concentrated and plasma osmolality rises. In patients who are water overloaded before the test is started, the urine may not become concentrated: plasma osmolality is usually low and may remain so since ADH secretion is only stimulated if it rises above 285 mmol/kg. Thus, the urine becomes concentrated only if the plasma osmolality exceeds this level. If the results of a fluid deprivation test are equivocal (as in practice they often are), the plasma ADH response to hypertonic saline infusion can be assessed. The response is normal in patients with NDI or primary polydipsia, but decreased in patients with CDI. The former two conditions can be distinguished by comparing plasma ADH concentration with urine osmolality after a period of fluid deprivation. In NDI, plasma ADH is much higher than normal.

Avi Sayag

Clinical Biochemistry

Topic 44
Pathobiochemistry and laboratory diagnosis of hypothyroidism and hyperthyroidism
For the histology of the thyroid gland, please consult previous notes (Avi's Histology Notes, Thyroid Gland, page 10). The 2 hormones, T3 and T4 diffuse passively through the membrane of the target cell, where they bind to intracellular nuclear receptors12. The association with the receptor leads to dissociation of heat shock proteins, and the complex is thus in its activated conformation. The complex is translocated to the nucleus (in case the receptor is in the cytoplasm). It then binds to the DNA on non-palindromic sequences. Steroid receptors often form dimers. In the nucleus, the complex acts as a transcription factor, augmenting or suppressing transcription of particular genes by its action on the DNA (for general knowledge, the receptor for the thyroid hormones belong to the RXR receptors (for more details, you may consult Avi's Biochemistry Notes, page 16). There are 3 thyroid hormones: T4 (thyroxine), T3 (triiodothyronin) and calcitonin, which is produced in the parafollicular cells (C cells) and not in the follicular cells. Thyroid hormones have diverse actions: 1. Calorigenic effect on tissues: they stimulate the basal metabolic rate, oxygen consumption and heat production (through actions that include stimulating the Na+/K+ pump and increasing the availability of energy substrates). 2. Stimulating protein synthesis, and thus 3. Growth development 4. Increase the sensitivity of the cardiovascular and nervous system to catecholamines (-adrenergic receptors), and thus 5. Stimulate heart rate and contraction 6. Stimulate carbohydrate metabolism 7. Increase synthesis and catabolism of cholesterol and TG. The overall effect of thyroid hormones is to increase net catabolism. Synthesis of thyroid hormones: Thyroid hormones are synthesized by first trapping serum iodine. Then, iodination processes of tyrosine residues on the thyroglobulin molecule occurs. When one iodine binds to the tyrosine it forms monoiodotyrosine MIT, and when 2 iodine atoms bind to the tyrosine they form diiodotyrosine DIT. When 2 DITs undergo a coupling reaction, they form T4, and when one MIT and one DIT undergo a coupling reaction, they form T3. Finally, the thyroglobulin is proteolytically cleaved to release the hormones. The cleavage occurs in the follicular cells, and the thyroglobulin is stored in the follicular colloid. Whereas the thyroid is the only organ that secretes T4, T3 can be produced by the kidney, the heart and the liver by conversion from T4. In the periphery, most T4 is de-iodinized to T3, which has a 10-fold greater affinity to the nuclear receptors than T4, and is thus 4-5 times more effective (actually, 40% of T4 is converted to T3 and 45% to rT3). It follows that we can regard T4 as a prehormone. Thyroid hormones in the blood The normal plasma concentration of T4 is 60-150 nmol/L, and T3 1-2.9 nmol/L (T4 is > 50 times T3). Almost all is bound to thyroxine (T4) binding globulin (TBG), and to a lesser extent to thyroxine-binding prealbumin and to albumin (99.98% of T4 and 99.66% of T3 are bound). However, the free T4 is only 2-3 times the free T3 measured in pmol/L). TBG is 1/3 saturated. The binding proteins serve to maintain the free T3 and T4 concentration within narrow limits, while ensuring the hormones are readily available to the tissues. Only the free thyroid hormones are physiologically active.
12

NOTE: it is generally agreed today that the receptors for the thyroid hormones are located in the nucleus (and there they heterodimerize), while other receptors for glucocorticoids, aldosterone, testosterone and estradiol are located in the cytoplasm (where they homodimerize).

Avi Sayag

Clinical Biochemistry

The increase in the concentration of the binding proteins (or in their affinity) is determined by 4 main factors: 1. Increased TBG concentration: a. Genetic factors b. Non-thyroidal illnesses (HIV infection, infectious and chronic active hepatitis, estrogen-producing tumors) c. Physiological conditions (pregnancy, newborns) d. Drug use (oral contraceptive, estrogens, tamoxifen, methadone) 2. Increased prealbumin concentration 3. Increased binding to albumin 4. T4 binding to autoantibodies. In the initial steady state, TBG is 1/3 saturated with T4. If TBG concentration increases, it will cause more T4 to be bound, thus reducing the fT4 concentration. This stimulates TSH secretion which leads to an increase in the release of T4 from the thyroid. T4 becomes redistributed between the bound and the free states, leading to a new steady state with the same fT4 concentration but an increased total T4. A decrease in the concentration of the binding proteins (or in their affinity) is determined by 3 main factors: 1. Decreased TBG concentration: a. Genetic factors (those who have a deficiency in the binding proteins show no clinical abnormality, though) b. Non-thyroidal illnesses (major illness or surgical stress, nephrotic syndrome, malnutrition, malabsorption) c. Drug use (androgens, anabolic steroids, large doses of glucocorticoids) 2. Decreased TBG binding capacity (drugs such as salicylates and phenytoin displace thyroid hormones from their binding proteins, thus reducing total, but not free, hormone concentration once a steady state is reached). 3. Decreased prealbumin concentration There are 3 main tests performed for thyroid function: 1. TSH This is the first hormone measured (first line) as its sensitivity is the highest. That is, TSH levels are decreased even at the earliest stages of hyperthyroidism, when the disease may still be subclinical. However, it is best to measure TSH in combination with fT4 (for hypothyroidism) and fT3 (for hyperthyroidism). In primary hypothyroidism, TSH levels are greatly increased (if there is too little thyroid hormones there is no suppression of the secretion of TSH in a negative-feedback mechanism), while in borderline cases the increase is smaller (but still, there is an increase). TSH is also elevated in other diseases which are not primary: TSH secreting tumors and during recovery from severe illnesses (e.g. sick euthyroid state). TSH is decreased in hyperthyroidism (too much thyroid hormones suppress the secretion of TSH), even if the hyperthyroidism is subclinical (as it is very sensitive). It is also decreased in secondary hyperthyroidism and in severe general illnesses. 2. fT3 and fT4 The measurement of free hormone concentrations poses major technical problems since the binding of free hormones in an assay, usually by an antibody, will disturb the equilibrium between bound and free hormone and cause release of hormone from binding proteins. This problem has been solved (I don't know how and it's not interesting). Low levels are measured in various forms of hypothyroidism. fT3 is not an adequate test in this case as it can be normal (especially in mild cases). Measurement of plasma total T4 (tT4) concentration has the major disadvantage in that it is dependent on binding protein concentration as well as thyroid activity. For example, a slightly elevated plasma tT4 concentration, compatible with mild hyperthyroidism, can occur with normal thyroid function if there is an increase in plasma binding protein concentrations (as in pregnancy for example, due to increased synthesis caused by estrogens). In addition, the values of the total thyroid

Avi Sayag

Clinical Biochemistry

hormones has a wide reference range that overlaps with values measured in hypo- and hyperthyroidism. In primary hypothyroidism, TSH levels are elevated and fT4 levels are reduced. In secondary hypothyroidism, TSH levels are reduced and fT4 levels are reduced. (TSH levels are reduced because of a secondary cause such as hypopituitarism, and the low levels of fT4 are due to the low levels of TSH). Primary hypothyroidism can be due to the followings: 1. Autoimmune hypothyroidism (Hashimoto's thyroditis) 2. Idiopathic atrophic hypothyroidism 3. Thyroidectomy, radioactive iodine, anti-thyroid drugs to correct for hyperthyroidism (carbimazole), goiter (in Hashimoto's disease or in iodine deficiency) or cancer. 4. Primary or secondary congenital hypothyroidism 5. Dyshormonogenic hypothyroidism (impaired hormone synthesis due to an enzyme defect) 6. Secondary acquired hypothyroidism (pituitary tumor or hypothalamic tumor, vascular insufficiency, trauma and infection) 7. Iodine deficiency (less iodine for the synthesis of thyroid hormones). It is important, though, to distinguish between patients who have a frank thyroid disease, and patients hospitalized for other non-thyroid diseases. In hospitalized patients, the results of lab tests occasionally resemble hypothyroidism, even though their thyroid is fine. The diagnosis of hypothyroidism that coincides with other diseases they are hospitalized for is quite difficult. Lastly, dopamine and corticosteroid (in many times given to hospitalized people) decrease TSH levels. The occurrence of abnormalities of thyroid function tests in patients with non-thyroidal illness has been termed the 'sick euthyroid syndrome'. Typically, during the acute phase of an illness, fT3 concentration and, less often, fT4 concentration is decreased. TSH is usually normal or low. During recovery, TSH may rise transiently into the hypothyroid range as thyroid hormone concentrations return to normal. In investigating the etiology of primary hypothyroidism, anamnesis should be taken (has the patient received any radioactive iodine treatment? Has he had thyroidectomy? Does he have a family history of hypothyroidism? Has he been taking anti-thyroid drugs?). Then, antibodies should be suspected (anti-TPO13, anti-thyroglobulin). In screening for congenital hypothyroidism, serum TSH levels should be measured 6-8 days after delivery. In investigating the etiology of secondary hypothyroidism (and hyperthyroidism) the third test should be performed: 3. TRH-test In this test, plasma TSH is measured immediately before, and 20 and 60 min after, giving the patient 200 g of TRH i.v. The normal response is an increase in TSH concentration of 2-20 mU/L in 20 min, with reversion towards the basal value at 60 min.

13

TPO Tyrosine Peroxidae, the enzyme that carries out the addition of iodine to tyrosine residues.

Avi Sayag

Clinical Biochemistry

Hyperthyroidism can be primary, secondary or thyroiditis. Primary hyperthyroidism can be the result of: 1. Toxic multinodular goiter; 2. Toxic thyroid adenoma 3. Thyroid carcinoma (rare) 4. Ectopic thyroid tissue (for example in struma ovarii). Secondary hyperthyroidism can due to endogenous or exogenous reasons: 1. Exogenous: iodine or iodine-containing drugs can stimulate synthesis of thyroid hormones. 2. Endogenous: increased levels of TSH or other substances that stimulate thyroid hormones. These conditions can be seen in a. Grave's disease (Ab to TSH receptors that stimulate them) b. Neonatal hyperthyroidism (due to maternal IgG those of a mother sick with Grave's disease that cross the placenta and stimulate the TSH receptors of the baby). This condition is temporary as these Ab will be cleared from the baby's blood eventually. c. TSH-secreting pituitary tumors (rare) d. Trophoblastic tumors (hCG has a weak stimulatory effect on thyroid secretion). This can be the cause of hydatiform moles, choriocarcinoma and embryonal carcinoma of the testes. Regarding Grave's disease, following partial thyroidectomy, serum fT3 and TSH levels change. Before surgery, the patient is rendered euthyroid with anti-thyroid drugs. Initially, TSH secretion remains suppressed, but eventually it rises in response to low fT3. Normal thyroid hormone secretion by the remaining thyroid tissue is maintained by increased TSH stimulation, but eventually, the patient becomes hypothyroid. Hypothyroidism can also develop in patients treated with anti-thyroid drugs or radioiodine. Thyroditis can be subacute (early infection), due to Hashimoto's thyroditis (though hypothyroidism is more common in this condition), after delivery (5-10% of women suffer from thyroditis after pregnancy) and following radiation therapy (radiation thyroditis). Lab of hyperthyroidism: TSH levels are greatly reduced (< 0.1 mU/L. Normal range is 0.3-4 mU/L), while fT3 and fT4 are elevated (fT3 is more appropriate to measure in hyperthyroidism). In mild hyperthyroidism, or when TSH secretion is inhibited by drugs, THS levels are reduced, and fT3 and fT4 are normal. In TSH-secreting pituitary tumors, TSH levels may be normal or elevated, while fT3 and fT4 are elevated too. In summary, these autoantibodies might be found in thyroid diseases: 1. Anti-TPO and anti-thyroglobulin antibodies: in Hashimoto's thyroiditis, in postpartum thyroiditis, in Grave's disease and in subacute thyroiditis. 2. Thyroid stimulating immunoglobulin (TSI): in Grave's disease 3. Anti-TSH receptor Ab: in Grave's disease.

Avi Sayag

Clinical Biochemistry

Off-Topic Introduction to Hypo- and Hypercalcaemia


Plasma calcium concentration is regulated by 3 hormones: PTH, calcitriol (vitamin D) and calcitonin. Most calcium in the body is in the bone (99% of body calcium, 25,000 mmol, which is 1 kg). Calcium in the plasma is found in 3 forms: bound to albumin (46%), bound to anions (7%, to bicarbonate, citrate, lactate, phosphate) and as free calcium (ionized calcium, 47%). The bound form renders calcium non-diffusible. Total calcium in the plasma measures 2.1-2.6 mmol/L. PTH PTH is a 84-amino acid polypeptide, formed in the parathyroid gland as a pre-pro-PTH with 115 amino acids. It is cleaved twice: first to form pro-PTH (after cleavage of 25 AA) and then intact PTH (after cleavage of the remaining 6 AA). The N-terminal, with 34 AA, is the active part of the hormone. In the plasma it is found as an intact hormone (with all 84 AA), as a cleaved hormone (with just the 34 AA of the N-terminal) and as C-terminal fragments (various lengths of amino acids). Only the intact PTH and the N-terminal fragment are the active forms of the molecule, with half life of 3-4 minutes. The C-terminal fragments are inactive, with half life of 2-3 hours. PTH (and calcitriol) act to increase plasma calcium levels, and their secretion is triggered by hypocalcaemia. PTH acts on several target cells: 1. On the bone: it acts rapidly on osteocytes to absorb calcium, which then passes to osteoblasts via canaliculi, where calcium is secreted from them facilitated by vitamin D. It also acts on osteoclasts (in the long term) and stimulates their proliferation (thus more calcium is resorbed from the bone to the plasma). 2. On the kidney: it acts on the distal tubules to increase calcium and Mg+2 reabsorption, and on the proximal tubules to decrease phosphate and bicarbonate reabsorption and to induce 1-hydroxylation (to form calcitriol). PTH binds to target cell receptors (Mg+2-dependent) and activates adenylate cyclase, which forms cAMP. This leads to influx of calcium into the cell to carry the desired signal transduction pathway. High levels of plasma calcium and calcitriol (also severe hypomagnesemia) inhibit PTH secretion in a negative-feedback mechanism. Calcitriol 7-dehydrocholesterol is hydroxylated first in the liver to 25-hydroxycholecalciferol (25-HCC, calcidiol) and then in the kidney to 1,25-dihydroxycholecalciferol, and 24,25dihydroxycholecalciferol. The 24,25-Vit-D has no biological function, and this is the mechanism of the kidney to keep the levels of active vitamin D within acceptable limits. Recent evidence shows that 1-hydroxylation can also occur in target cells, bypassing the kidney for activation of the vitamin directly from the liver. Calcitriol acts on the gut to stimulate calcium and phosphate absorption, and on the osteoblasts in the bone to promote secretion of calcium to the ECM, and thus promotes mineralization. In high concentrations, however, it acts on the bone to stimulate osteoclastic resorption. Calcitriol inhibits its own formation in the kidney. Low levels of PTH also inhibit its 1-hydroxylation in the kidney (because the kidney reads it as "there is enough Ca+ in the blood and there is no need for vitamin D"). It should be noted that vitamin D also participates in cellular differentiation, and thus has a role in malignancies. It also stimulates production of cytokines, and thus has an immunomodulatory effect. Calcitonin This hormone is secreted from the C cells in the thyroid when plasma calcium concentration rises, but its physiological activity is unknown. After total thyroidectomy, when no calcitonin is secreted, there are NO clinical syndromes that can be ascribed to its deficiency, and calcium homeostasis is normal. Its levels are elevated during pregnancy and lactation. The hormone has also been detected in the GI and CNS (as a neurotransmitter). It can be shown experimentally to inhibit osteoclast activity, and thus bone resorption. Also, it serves as a screening marker for medullary thyroid cancer (see topic 8 for more details).

Avi Sayag

Clinical Biochemistry

Topic 45
Hypocalcaemia
Causes of hypocalcaemia: 1. Decreased protein-bound calcium the methods used for determining plasma calcium concentration measure total calcium. So, if the protein-bound calcium is decreased, then the measure will show low plasma calcium. This can be caused by overhydration with calcium-free fluids and hypoalbuminemia (as in the nephrotic syndrome). Recall that 46% of calcium is bound to albumin in the plasma). 2. Decreased PTH secretion: a. Primary hypoparathyroidism b. Autoimmune hypoparathyroidism c. Pseudohypoparathyroidism (in fact, the PTH levels are high, but the body responds as if the levels were low). 3. PTH secretion is appropriate: a. Secondary hyperparathyroidism b. Vitamin D deficiency c. Impaired metabolism of vitamin D d. Renal disease e. Increased inactivation f. Drugs Blood collected into EDTA tube can artefactually yield hypocalcaemia (EDTA binds Ca+2). Hypoparathyroidism This can be congenital or acquired. The congenital form may be associated with DiGeorge syndrome: a genetic anomaly in which the 3rd and 4th pharyngeal pouches fail to develop, leading to many abnormalities, the most important being aplasia of the thymus (immunodeficiency develops) and the parathyroid gland (hypoparathyroidism). The congenital form also includes pseudohypoparathyroidism: a condition that superficially resembles hypoparathyroidism, but plasma concentrations of PTH are elevated. The two types are both hereditary. The effects of PTH are mediated through the formation of cAMP. In type 1, the activation of adenylate cyclase is defective, and cAMP is not formed in response to the binding of PTH to its receptor. This manifests as rounded face, skeletal abnormalities and learning difficulties. In type 2, cAMP is formed, but the responses to it are blocked. The two types can be distinguished by measuring urinary cAMP after administration of PTH. In normal individuals, and in patients with type 2 pseudohypoparathyroidism, there is an increase of urinary cAMP, while in type 1 there is none. The acquired form includes: 1. Surgery: thyroidectomy (total/partial), laryngectomy or parathyroidectomy (the parathyroid is removed in these operations). 2. Autoimmune disorders: patients with hypoparathyreosis have antibodies against parathyroid cells and calcium-sensing receptors14 in the parathyroid gland. 3. Hemochromatosis: accumulation of iron in the gland may lead to dysfunction. 4. Infiltrative states. Secondary hyperparathyroidism Plasma PTH concentrations are also raised in many patients with chronic renal disease, liver diseases and vitamin D deficiencies. All these conditions are associated with decreased synthesis of calcitriol, which causes hypocalcemia. The diagnosis of secondary hyperparathyroidism in blood test: serum Ca+2 is low or normal, Pi can be low or elevated, ALP is elevated and PTH is elevated (most important: high PTH + low Ca+2).

These sensor initiate secretion of PTH when no Ca+2 is bound to them. If these are dysregulated, their set-point changes, they sense that the Ca+2 concentration is adequate (when it's not) and signal not to secrete PTH even though it is required.

14

Avi Sayag

Clinical Biochemistry

The causes of secondary hyperparathyroidism can be classified into 2 groups: with osteomalacia and rickets, and without them. 1. With osteomalacia or rickets: a. Due to a decrease of calcium and vitamin D uptake, a decrease in calcium absorption, diet low in vitamin D, steatorrhea or malabsorption. b. Decrease in the production of vitamin D: this can be due to chronic renal failure, chronic liver disease and lack of 1-hydroxylation. c. Increased inactivation of vitamin D: drugs given in anticonvulsive therapy can inactivate vitamin D. d. Lack of 25-hydroxycholecalciferol e. Vitamin D receptor defect. 2. Without osteomalacia and rickets: a. Acute pancreatitis: when the pancreas is damaged, free fatty acids are generated by the action of pancreatic lipases. Insoluble calcium salts form in the pancreas, and the free fatty acids chelate the salts, resulting in calcium deposition in the retroperitoneum (i.e. saponification of fatty acids). b. Neonatal hypocalcaemia: the problem lies in the parathyroid glands that do not yet function adequately; and in infants of mothers with diabetes or hyperparathyroidism, because these women have higher-than-normal ionized Ca+2 levels during pregnancy. Clinically, hypocalcaemia may cause increased neuromuscular excitability, muscle spasm, convulsions and tetany. If the condition lasts, cataracts may occur. In order to detect the etiology of hypocalcaemia, albumin levels should be measured first (normally 30-60 g/L). If albumin levels are low, there are 2 options: the Ca+2 level is either normal or low. If it is normal, then the problem is due to low albumin levels (hypoalbuminemia) or overhydration (i.v of fluids low in calcium). If the Ca+2 levels are low (or if the albumin level is normal), the phosphate level should be checked (remember that high Pi levels inhibit activation of vitamin D). If the levels of Pi are high, it might be because of renal failure that fails to excrete Pi. Therefore, we have to check the urea level or creatinine level to check renal functions. If creatinine/urea levels are high, then the kidney is to blame, and the hypocalcaemia is due to secondary hyperparathyroidism with renal failure (usually end-stage renal failure). But if the creatinine/urea levels are normal, the cause can be either hypoparathyroidism or pseudohypoparathyroidism. We have to measure PTH for that: if it's low, then it's hypoparathyroidism, and if it's high, it's because of pseudohypoparathyroidism. But if Pi levels are low to begin with, then the reason for the hypocalcaemia is secondary hypoparathyroidism without renal failure. To sum it up, these tests should be performed in calcium metabolism: Serum total calcium with albumin, serum Ca+2, Pi (in fasting), ALP, total CO2 levels, creatinine/urea and PTH. In the urine: Ca+2, Pi and cAMP. Why measure total CO2 levels? Alkalosis and acidosis (also due to hyper- or hypoventilation) can lead to hypo- or hypercalcaemia: in alkalosis, H+ dissociates from albumin and Ca+2 binding to albumin increases. There is also an increase in Ca+2 complex formation. Thus, the concentration of Ca+2 falls, and this may produce clinical hypocalcaemia, although total plasma Ca+2 is unchanged.

Avi Sayag

Clinical Biochemistry

Topic 46
Hypercalcaemia
Causes of hypercalcaemia include: 1. Increased protein-bound Ca+2: for the same reason mentioned in the previous topic. This can result from dehydration and prolonged venous stasis. 2. Increased PTH secretion: this can be due to a. Primary and tertiary hyperparathyroidism (NOT secondary!). Occasionally, patients with end-stage renal failure become hypercalcaemic, owing to the development of autonomous PTH secretion, probably as a result of the prolonged hypocalcaemic stimulus. Such hypercalcaemia may manifest for the first time in a patient given a renal transplant, who becomes able to normally metabolize vitamin D. This is termed tertiary hyperparathyroidism. The diagnosis of primary hyperparathyroidism in blood test: serum Ca+2 is elevated, Pi is low, ALP is normal or elevated, as well as PTH levels. Tertiary hyperparathyroidism is diagnosed in blood tests: elevated: serum Ca+2,ALP, PTH (highly), Pi is low or elevated. b. Ectopic PTH production c. Familial benign hypercalcaemia: a syndrome of lifelong hypercalcemia inherited as an autosomal dominant trait, and caused by a mutation in the calcium sensor gene leading to constitutive PTH secretion. The diagnosis is made by detecting hypercalcemia after parathyroidectomy, and low rate of calcium secretion in the urine. 3. Conditions related to appropriate PTH secretion: a. Vitamin D excess b. Sarcoidosis: the mechanism of action in sarcoidosis most commonly thought to cause hypercalcemia is dysregulated production of vitamin D by activated macrophages, that is, as a result of 1-hydroxylation of 25hydroxycholecalciferol by macrophages in the sarcoid granulomas. Similarly, TB can cause hypercalcaemia. c. Milk-alkali syndrome: hypercalcaemia is associated with the ingestion of milk and antacids for the control of dyspeptic symptoms. Alkali increases the renal reabsorption of filtered calcium but the precise mechanism is unknown. This syndrome is uncommon, and becoming more so since the introduction of drugs that inhibit gastric acid secretion for the treatment and prevention of peptic ulceration. It should also be remembered that dyspepsia itself may be a feature of hyperparathyroidism since calcium stimulates gastrin release. d. Malignancies: parathyroid hormone related peptide (PTH-rP) is elevated in 50-90% of patients with hypercalcemia associated with malignancy. Its Nterminal shows homology to PTH, which can bind to PTH receptors and mimic its biological action. Its gene is located in chromosome 12 (PTH gene is located on chromosome 11). In addition, bone metastases cause direct calcium resorption from the bone. Lastly, cytokines (IL-1 and TNF) or prostaglandins cause osteoclast activation, and thus bone resorption. e. Hyperthyroidism (rare) f. Drugs: Thiazide diuretics sometimes cause mild hypercalcaemia, owing to an effect on renal calcium excretion. Chronic lithium therapy can cause increased PTH secretion and is an occasional cause of hypercalcaemia. The clinical features of hypercalcemia are: 1. Renal failure, renal tubular damage and nephrocalcinosis; 2. Decreased neuromuscular excitability (muscle weakness, tiredness, impaired concentration and other mental changes); 3. Direct heart effect total calcium level above 3.75 mmol/L leads to cardiac arrest, cardiac arrhythmia and hypertension.

Avi Sayag

Clinical Biochemistry

Off-Topic Introduction to Topics 47, 48


The adrenal gland is composed of the medulla, which is part of the sympathetic system and is not essential for life, and the cortex, which has 3 layers: zona glumerulosa (secretes aldosterone), zona fasciculata (cortisol) and zona reticulata (secretes androgens).

All in the green frame takes place in the zona glumerulosa, and all in the black frame in the z. fasciculata and z. reticulata. Note the enzymes common to all 3 zones: cholesterol hydroxylase, 17-hydroxylase and 3-hydroxydehydrogenase. Note also the enzyme that are unique to the zona glumerulosa: corticosterone methyloxidase I and II, and to the zona reticulata: desmolase. ACTH is needed to convert cholesterol to pregnenolone. Therefore, for the production of aldosterone, ACTH is needed (but is not a must), ATII, and probably factor X (the production is also induced by hyperkalemia). Cortisol inhibits the secretion of ACTH and CRH in the hypothalamus (long loop), and ACTH inhibits CRH in a negative feedback mechanism (short loop). 95% of cortisol in the blood is bound to transcortin (cortisol-binding globulin). 5% of cortisol is free in the plasma, and can thus be freely excreted in the urine. Transcortin is fully saturated at normal cortisol concentrations. Because of this, if cortisol production increases, the concentration present in the plasma in the free form, and thus the amount that is excreted, increases proportionately.

Avi Sayag

Clinical Biochemistry

Functions of glucocorticoids: 1. Increase protein catabolism; 2. Increase glycogenolysis; 3. Increase hepatic gluconeogenesis; 4. Inhibit ACTH secretion; 5. Sensitize arterioles to the actions of noradrenaline, hence involved in the maintenance of blood pressure. The adrenal medulla has 2 cell types: 1. Chromaffin cells (so named because of the brown-black color after staining with potassium- dichromate) 2. Neural cells The adrenal medulla is strongly related to the sympathetic nervous system embryologically (derived from neural crest cells) and functionally (it produces catecholamines and has sympathetic innervation). Pathological conditions of the adrenal medulla are associated with catecholamine hypersecretion. In the adrenal medulla there is synthesis of catecholamines: Tyrosine DOPA dopamine norepinephrine epinephrine Tyrosine is found in the plasma, and the formed DOPA is located in the mitochondria. Dopamine is released to the cytoplasm and stored in granules, and the formed norepinephrine is stored in granules. Epinephrine is found in the cytoplasm and stored in granules. Apart from the adrenal medulla, catecholamine synthesis occurs in extramedullary chromaffin cells (paraganglions) and in sympathetic neurons. It follows that storage of catecholamines occurs in the granules of the chromaffin cells as well as in the granules of sympathetic nerve endings. They are released by exocytosis during sress, physical activity, hypoxia, hypoglycemia and hypovolemia. Dopamine is mainly secreted in the CNS and less in the adrenal medulla and sympathetic system. Norepinephrine is secreted mainly in the CNS and sympathetic system and less in the adrenal medulla (just a little bit less, though). Epinephrine is secreted exclusively in the adrenal medulla. Catecholamines can be reuptaken by storage granules in nerve endings, excreted in their free form (1-3%), excreted in their conjugated form, or undergo metabolic inactivation. The metabolic products can be intermediate products (metanephrine) or endproducts (VMA from E and NE, and HVA from dopamine).

Avi Sayag

Clinical Biochemistry

Topic 47
Clinical biochemistry of disturbances of the adrenal cortex
Disorders of the adrenal cortex can be either adrenal hypofunction (Addison's disease) or hyperfunction (Cushing's syndrome, Conn's syndrome and congenital adrenal hyperplasia). Cushing's syndrome Definition: overproduction of glucocorticoids (mineralocorticoids and androgens can also be excessive, depending on the mechanisms and their relation to the biosynthesis of cortisol). Causes: 1. Cushing's disease: adrenal hyperfunction secondary to a pituitary corticotroph adenoma. This accounts for 60-70% of spontaneous Cushing's syndrome. 2. Adrenal adenoma and carcinoma 3. Ectopic ACTH production (e.g. carcinoma of the bronchus and carcinoid tumors). 4. Corticosteroid or ACTH treatment. Clinical features: 1. Weight gain, central obesity, the growth of fat pads along the collar bone and on the back of the neck (buffalo hump) and a "moon face". 2. Thinning of the skin (which causes easy bruising and dryness), purple or red striae (the weight gain in Cushing's syndrome stretches the skin, which is thin and weakened, causing it to hemorrhage) 3. Proximal muscle weakness (hips, shoulders), and hirsutism (facial male-pattern hair growth due to elevations in androgens). 4. The excess cortisol may also affect other endocrine systems and cause, for example, insomnia, reduced libido, impotence, amenorrhoea and infertility. 5. Psychological disturbances, ranging from euphoria to psychosis. Depression and anxiety are also common. 6. Persistent hypertension and hypokalemia (due to cortisol's enhancement of epinephrine's vasoconstrictive effect, and due to the fact that cortisol precursors and cortisol itself have some mineralocorticoid activity, thus causing Na+ retention and hypertension). 7. Insulin resistance (especially common in ectopic ACTH production), leading to hyperglycemia, which can lead to diabetes mellitus. 8. Due to excess ACTH, it may also result in hyperpigmentation. This is due to MSH production as a byproduct of ACTH synthesis from Pro-OpioMelanoCortin (POMC). Pseudo-Cushing's syndrome, in which patients appear cushingoid, can occur in Cushing's disease, alcoholism and obesity (rare). If alcoholism is the inducer, the symptoms resolve rapidly upon withdrawal of alcohol. Diagnosis: 1. 24-hour urinary cortisol excretion: normal 24h cortisol excretion is <300 nmol/L. 2. Dexamethasone suppression test: dexamethasone is a synthetic glucocorticoid that binds to cortisol receptors in the pituitary and suppresses ACTH release (and thus the secretion of cortisol by the adrenals). In the overnight test, 1 mg is given at night and blood is drawn for measurement of cortisol at 9 am the next morning. In normal individuals, this should be less than 50 nmol/L. A failure of suppression is characteristic of Cushing's syndrome. 3. Insulin hypoglycemia test: normal increase in plasma cortisol concentration occurs in response to hypoglycemia, and this response is abolished even in mild Cushing's syndrome. 4. Loss of diurnal variation of cortisol secretion is an early feature of Cushing's syndrome (but a problematic test: see Marshall page 163). 5. ACTH measurement 6. CRH measurement: can be useful to differentiate between Cushing's disease and ectopic ACTH secretion. In Cushing's disease, CRH typically increases plasma ACTH, whereas with ectopic ACTH secretion or an adrenal tumor there is typically no response.

Avi Sayag

Clinical Biochemistry

Here is the DD of Cushing's syndrome:

Conn's syndrome Definition: excessive production of aldosterone. Causes: 1. Adrenal adenoma and carcinoma; 2. Bilateral hypertrophy of zona glumerulosa cells; 3. Glucocorticoid-remediable aldosteronism: an autosomal dominant condition in which aldosterone synthesis is under the control of ACTH (rare). Hyperaldosteronism can be also secondary to enhanced renin secretion: 1. Hypoalbuminemia (edema renal hypoperfusion increased renin secretion) 2. Nephrotic syndrome (causes hypoalbuminemia) 3. CHF (renal hypoperfusion increased renin secretion) 4. Renal artery stenosis (renal hypoperfusion increased renin secretion) 5. Renin-secreting tumors (renal juxtaglomerular cell tumor) Clinical features: 1. Hypokalemia (always) and hypertension; 2. Muscle weakness and parasthesias 3. Polydipsia, polyuria and nocturia Diagnosis: Lab diagnosis consists of measuring low K+ and high Na+ in the blood, and high K+ in the urine. Screening for hyperaldosteronism includes measuring low K+ in the serum, high K+ in the urine, renin and aldosterone measurements, and Na+ loading test, in which 200 mmol/L of Na+ is given over 24 hours, followed by Na+ and K+ measurements. Normally, aldosterone should be inhibited by the increased Na+ reaching the distal tubule, and less Na+ should be found in the plasma. Aldosterone secretion is stimulated through the action of renin; therefore, it is helpful to measure the plasma renin activity at the same time as the concentration of aldosterone to establish whether aldosterone secretion is autonomous or under normal control. Congenital adrenal hyperplasia (CAH) Definition and causes: a group of inherited metabolic disorders of adrenal steroid hormone biosynthesis. Their clinical features depend upon the position of the defective enzyme in the synthetic pathway, which determines the pattern of hormones and precursors that is produced. 21-hydroxylase deficiency: accounts for around 95% of all cases of CAH. The remaining 5% are due to deficiency of 11-hydroxylase. The deficiency can be complete or partial. 21-hydroxylase deficiency is often incomplete and adequate cortisol synthesis can be maintained by increased secretion of ACTH by the pituitary. It is this that causes hyperplasia of the glands. Because of the metabolic block, the substrate of the enzyme (17hydroxyprogesterone) accumulates and there is increased formation of adrenal androgens. Clinical features: female infants affected by CAH may be born with ambiguous genitalia, but when the enzyme deficiency is only partial, the condition may not present until early adulthood with hirsutism, amenorrhoea or infertility (late onset CAH). Males may present

Avi Sayag

Clinical Biochemistry

with pseudoprecocious puberty in their second or third year of life but are not virilized at birth. Diagnosis: diagnosis is made by demonstrating an elevated concentration of 17hydroxyprogesterone (17-OHP) in the plasma at least two days after birth (before this time, maternally derived 17-OHP may still be present in the infant's blood). Treatment is monitored by measurement of plasma 17-OHP. Partial 11-hydroxylase deficiency is also more common than complete deficiency of the enzyme. Increased androgen production causes virilization (development of male secondary sexual characteristics), which tends to be more severe than in 21-hydroxylase deficiency (but again, it is not present in males at birth). Hypertension develops, owing to the accumulation of 11-deoxycorticosterone, a substrate of the defective enzyme that has salt-retaining properties. The diagnosis rests upon the demonstration of an increased plasma concentration of either 11deoxycortisol or its urinary metabolite. Treatment is with cortisol alone: although aldosterone secretion is defective, 11-deoxycorticosterone provides an adequate mineralocorticoid activity. Hypofunction of the adrenal gland Addison's disease Definition: a rare endocrine disorder in which the adrenal gland does not produce enough steroid hormones (glucocorticoids and often mineralocorticoids). Causes: 1. The commonest cause of adrenal hypofunction is suppression of the pituitary-adrenal axis by glucocorticoids used therapeutically. 2. Autoimmune adrenalitis 3. AIDS 4. TB 5. Amyloidosis 6. Hemochromatosis 7. Adrenalectomy Symptoms: fatigue, weight loss, weakness, pigmentation (because of high concentrations of ACTH, which occur because of the loss of negative feedback by cortisol: ACTH has some melanocytes stimulating activity) and hypotension. Acute adrenal failure is a medical emergency characterized by severe hypovolemia, shock and hypoglycemia. It can be precipitated by stress (infection, trauma and surgery). Hemorrhage into the adrenal glands may occur as a complication of anticoagulant treatment and in meningococcal septicaemia, and can result in acute adrenal failure. Adrenal failure can occur secondarily to pituitary failure as a result of decreased stimulation by ACTH. Diagnosis: 1. Hyponatraemia can occur, since the lack of cortisol reduces the ability of the kidneys to excrete the water load, but there is no renal salt wasting since aldosterone secretion is not dependent upon ACTH. 2. Hyperkalemia 3. Cortisol levels below 50 nmol/L is diagnostic of adrenal failure. Cortisol levels above 550 nmol/L excludes the diagnosis. In cases when the levels of cortisol are between 50-550 nmol/L, the short Synacthen test should be performed (= ACTH stimulation test): Blood should be taken for ACTH assay before giving ACTH. Blood is taken at 9am for measurement of cortisol. 250 microgram of soluble ACTH are given i.m (Synacthen) Blood sampling for cortisol is taken at 9:30 and at 10:00am. Normal results: plasma cortisol after ACTH increase of 200 nmol/L with peak of >550 nmol/L.

Avi Sayag

Clinical Biochemistry

Topic 48
Clinical biochemistry of disturbances of the adrenal medulla
Consult again the introductory section to topics 47 and 48, regarding the adrenal medulla. There are several conditions associated with increased catecholamine metabolism derived from chromaffin cells: 1. Phaeochromocytoma (or phaeochromoblastoma) 2. Paraganglioma The followings are derived from neural cells: 1. Neuroblastoma 2. Ganglioneuroma (or ganglioneuroblastoma) And: 1. Melanoblastoma Phaeochromocytoma (phaeochromoblastoma and paraganglioma) These are neoplasms composed of chromaffin cells. Their features follow the "rule of 10": 1. 10% are malignant; 2. 10% are familial, or are associated with familial syndromes (MEN type 2a and 2b, neurofibromatosis type 1 and Hippel-Lindau disease); Some familial forms are without any other abnormalities or associated with islet cell tumors. 3. 10% are bilateral; and 4. 10% are extra-adrenal (paraganglioma) Clinical symptoms: 1. Hypertension: in attacks or sustained hypertension (in 5% there is no hypertension) 2. Tachycardia, headache, sweating 3. Abdominal pain 4. Nausea and vomiting 5. Visual disturbances 6. Nervousness, irritability 7. Increased appetite but loss of weight (hypermetabolic condition) 8. Dyspnea 9. Angina and cardiac hypertrophy If you suspect that your patient might have phaeochromocytoma, you should screen for it. Why suspect? The patient present with characteristic symptoms, among which hypertension resistant to therapy. Also, patients with neurofibromatosis, and those who experienced hypertensive episodes during anesthesia or delivery of a baby. Diagnosis: We should remember that foods and drugs and other factors may influence plasma level and excretion of catecholamines, and that catecholamines show diurnal variation (therefore a 24h urine collection is useful). So, when preparing the patient for catecholamine determination, the patient should avoid taking drugs that interfere with catecholamine determination for at least 8 days prior to the test (methyl-DOPA, levo-DOPA, -adrenergic antagonists, etc), as well as certain foods (spices) that should be avoided for at least 3 days. Catecholamines in the plasma: the patient should lie for 30 minutes, and a canule should be introduced 20-30 minutes in advance. This is in order to avoid any stress, physical and psychological. Blood should be drawn at the same time of the day, and not immediately after eating. In addition, these tests can be performed: 1. Suppression test with clonidine or pentolinium: pentolinium is a sympathetic ganglion-blocking drug that reduces catecholamine secretion in normal subjects but not in patients with phaeochromocytomas; in such patients, secretion is autonomous. Blood is taken for catecholamine measurement before and 15 minutes after giving 2.5 mg pentolinium by intravenous injection. 2. Provocative test with glucagon 3. Imaging: CT, MRI

Avi Sayag

Clinical Biochemistry

Catecholamines in the urine: urine should be collected over 24 hours, and heavy exercise should be avoided. Urine can be collected spontaneously in neonates and small children, together with creatinine ratio determination. Metanephrine should be measured in the urine, VMA (but NOT HVA/HMMA) and catecholamines. Measurement is done with HPLC. Further lab changes in this condition include: 1. Hyperglycemia and glycosuria during an attack 2. Impaired glucose intolerance 3. Renal disease caused by the hypertension 4. Changes caused by volume depletion Catecholamines may also rise in these conditions: Diabetic ketoacidosis, AMI, acute CNS disturbance, post-operative, hypo- and hyperthyroidism, following heavy exercise, volume depletion, renal disease, heavy alcohol intake, hypoglycemia, stress and various drugs. What are the drugs that can interfere with catecholamine determination? These are grouped into 3: Drugs interfering with HPLC Drugs inhibiting Drugs increasing assay catecholamine production / catecholamine excretion release / excretion Methyldopa Phenothiazine Theophylline Levodopa MAO inhibitors Aminophylline Clonidine Caffeine Reserpine Ethanol Bromocriptine Glucagon Dexamethasone Insulin Amphetamine Vasodilators The treatment can be surgery (if diagnosed and done early enough, full recovery results) with administration of -adrenergic receptor blockade (phenoxybenzamine). If surgical removal is not possible, -adrenergic receptor blockers or drugs that inhibit the synthesis of catecholamines (metyrosine) should be administered. Neuroblastoma This tumor can occur in the neuroblasts of the adrenal medulla, or in the sympathetic trunk (ganglioneuroma, ganglioneuroblastoma). It's the 3rd most frequent malignancy in childhood, and is mostly located in the abdomen (most cases arise in either the adrenal medulla or in the retroperitoneal sympathetic ganglia). Symptoms include general symptoms of malignancy with metastases, but hypertension is usually absent (as opposed to phaeochromocytoma). Diagnosis: Urine: NE, normetanephrine, dopamine, VMA, HVA increased excretion. Excretion of epinephrine is normal, though. If a single assay is made, then the sensitivity is around 75%. Therefore, a combination of 2 assays is better: VMA + HVA, or VMA + catecholamine, then sensitivity rises to 95%. NSE (neuron specific anolase) may increase too. The higher the increase, the poorer the prognosis.

Avi Sayag

Clinical Biochemistry

Topic 49
Clinical biochemistry of the reproductive system
(This topic is quite long, but it actually verbalizes the entire lectures on this topic, and aims to explain the "coded" slides of the department. It is thus longer than the actual recapped version of the topic). Male Reproductive System The hypothalamus-pituitary-testis axis:

Testosterone is produced in Leydig cells (also known as "thecal cells"). It is also produced in the adrenal cortex, but in very small amounts the majority there is DHEA (recall from the introductory section for the adrenal cortex). In the circulation, about 97% of testosterone is bound mainly to sex hormone binding globulin (SHBG) with high affinity, and to albumin with lower affinity. The free fraction (2-3%) is readily available to tissues; albumin binds testosterone more loosely than SHBG and albuminbound testosterone may be in part available (i.e. active). In most tissues, testosterone is converted into dihydrotestosterone (DHT) by the aid of 5reductase. Why is it converted to DHT in target tissues? It appears that DHT has a greater affinity for androgen receptors in target tissues than testosterone (both have receptors, though), and that the DHT-receptor complex is more effective. Defects in the receptors for either DHT, testosterone or both lead to various clinical abnormalities (collectively termed "androgen insensitivity"). If there is a deficiency of 5-reductase, DHT will not be formed. Male internal genitalia develop normally (they are not dependent on DHT), but masculinization, which requires DHT, is incomplete. In the brain and in adipose tissue, testosterone might be converted into estrogen. This might be related to libido, sexual behavior and feed-back regulation. Testosterone has anabolic effects, through the action of GH, which increases protein synthesis and decreases protein breakdown. It promotes the fusion of epiphyseal plates and promotes the retention of sodium, potassium, water, sulfates and phosphates. Together with FSH that acts on Sertoli cells, they promote spermatogenesis in puberty (FSH acts on Sertoli cells, and there is a paracrine communication between Sertoli and germ cells). As mentioned, testosterone has a role in the development of secondary male sex features (size of penis, deeper voice, pubic hair, body configuration, size of muscles, and everything that makes a man A man, Amen). Recall from the introduction to topics 47, 48, that testosterone is formed mainly in the testis and to a lesser degree in the adrenal cortex; androstendione is formed in the testis and in the adrenal cortex, and DHEA is, too, formed in the testis and in the adrenal cortex (these are the 3 musketeers of androgens).

Avi Sayag

Clinical Biochemistry

Measurements of testosterone in the plasma: Testosterone is measured in the plasma using immunoassays. After birth, the plasma concentration increases for 3 months and returns to baseline by the age of 1 year. It remains in its baseline until puberty. Around age 6-7, there is an increase in DHEA and androstendione, and around age 16-19, adult values of testosterone are reached. Male reproductive abnormalities can be classified into 3 main groups: 1. Hypogonadotrophic hypogonadism (secondary hypogonadism) a. Panhypopituitarism (congenital or acquired) b. Hypothalamic syndrome (congenital or acquired) i. Structural defects (neoplasms, inflammations) ii. Prader-Willi syndrome (7 genes on paternal chromosome 15 are missing, and hypogonadism is part of the clinical features) iii. Laurence-Moon syndrome c. GnRH deficiency (Kallmann's syndrome - secondary hypogonadism) d. Hyperprolactinemia (through the inhibition of GnRH by prolactin. It can be caused by drugs that block dopaminergic receptors, like haloperinol, or deplete dopamine, like methyldopa and reserpine; it can also be caused by prolactinomas). e. Malnutrition and anorexia nervosa f. Drug-induced suppression of LH, such as androgens, estrogens, tranquilizers, antidepressants, antihypertensives, barbiturates, cimetidine (an H2-receptor blocker given to treat stomach acidity, but it increases the activity of estrogens through a non-interesting mechanism of interference with cyt-P450). 2. Hypergonadotrophic hypogonadism (primary hypogonadism) a. Acquired condition: irradiation, mumps (can be complicated with orchitis), castration and cytotoxic drugs. b. Chromosome defects: i. Klinefelter's syndrome (47,XXY) ii. Autosomal and sex chromosomes, polyploids iii. True hermaphroditism c. Defective androgen biosynthesis (look at the chart in topic 47) i. 20-hydroxylase (desmolase) ii. 17,20-lyase deficiency iii. 3-hydroxysteroid dehydrogenase deficiency iv. 17-hydroxylase deficiency v. 17-hydroxysteroid dehydrogenase deficiency d. Testicular agenesis e. Selective seminiferous tubular disease f. Others: i. Noonan's syndrome (cryptochidism is one of the symptoms) ii. Streak gonads (hypoplastic and dysfunctioning gonads) iii. Myotonic dystrophy iv. Acute and chronic disease 3. Defects in androgen action a. Complete androgen insensitivity (testicular feminization) b. Incomplete androgen sensitivity: i. Due to androgen-receptor defects ii. 5-reductase deficiency Diagnosis of hypogonadism should distinguish between primary and secondary hypogonadism: in both cases, testosterone levels are low. But: In primary hypogonadism the problem is confined to the testis; thus, low concentrations of testosterone do not inhibit FSH and LH secretion. The lab results will be: low testosterone and high FSH and LH. Furthermore, seminiferous tubule defects are associated with high FSH,

Avi Sayag

Clinical Biochemistry

and Leydig cell defects with high LH levels. hCG can be used to test Leydig cell function, because it has an action similar to LH. In secondary hypogonadism, the problem is extrinsic to the testis. Thus, low (or sometimes normal) FSH and LH lead to low testosterone levels. In addition, depending on the basal underlying disease, some other tests should be considered such as: chromosome analysis, sperm analysis (to detect problems in the SN tubules), prolactin determination, SHBG measurement and dynamic tests (GnRH stimulation test). An unwelcome consequence of hypogonadism is infertility that may be caused by 5 main disorders: 1. Endocrine disorders: a. Hypothalamic dysfunction (kallmann's syndrome) b. Pituitary failure (tumor, surgery) c. Hyperprolactinemia (drug, tumor) d. Exogenous androgens e. Thyroid disorders f. Adrenal hyperplasia g. Testicular failure 2. Anatomical factors a. Congenital absence of vas deferens b. Obstructed vas deferens c. Congenital abnormalities of the ejaculatory system d. Varicocele e. Retrograde ejaculation (that's a funny condition in which the man ejaculates into his urinary bladder and then urinates the sperm. It's not a problem of infertility anymore, because the sperm can be separated from the urine by centrifugation. My roommate was curious to know if they can reach orgasm, so I checked, and they do, but not a strong one though). 3. Abnormal spermatogenesis a. Idiopathic azoospermia b. Chromosomal abnormalities c. Mumps complication (orchitis) d. Cryptochidism e. Chemical/radiation exposure 4. Abnormal motility a. Absent cilia (Kartagener's syndrome) b. Antibody formation 5. Psychosocial factors a. Impotence (psychological) b. Decreased libido Diagnosis of male infertility differentiates between 4 causes: 1. If LH levels are high, but testosterone is low, the problem is in Leydig cells 2. If LH levels are low, and testosterone is low, the problem lies in the hypothalamuspituitary axis. 3. If FSH levels are high, and the sperm count is low, the problem lies in the SN tubules. 4. If FSH levels are low, and the sperm count is low, the problem lies in the hypothalamus-pituitary axis. Semen analysis should be performed to check the motility and number of sperm. Immunological tests should also be performed.

Avi Sayag

Clinical Biochemistry

Female Reproductive System Description of the hypothalamus-pituitary-gonad axis: 1. The hypothalamus secretes GnRH (a decapeptide) in a pulsatile manner, with a peak secretion every 90 minutes. It induces FSH and LH secretion from the pituitary. 2. In the pituitary, LH and FSH are produced in the basophilic cells. They have an subunit and a subunit. The subunit is identical in these hormones and in TSH and hCG. The subunit is unique to each of them, therefore a specific assay is possible to detect them. a. FSH causes follicular maturation (granulosa cell appearance), increases the LH receptors on granulosa cells, together with LH it stimulates estrogen secretion from granulosa cells, and increases inhibin secretion. It inhibits the secretion of GnRH in the hypothalamus (short loop). b. LH increases androgen synthesis in thecal cells, in combination with FSH it stimulates estrogen secretion, it starts luteinization, and increases progesterone synthesis. It inhibits the secretion of GnRH in the hypothalamus (short loop). c. Prolactin is produced in the acidophilic cells, and it is necessary for lactation and progesterone secretion. TRH from the hypothalamus stimulates its secretion (and NOT GnRH) and is inhibited by dopamine. 3. In the ovary, estrogen, progesterone and androgens are produced. a. Estrogen: an 18-carbon long hormone, which is synthesized in the granulosa cells of the follicle. It promotes feminization of the genital tract and secondary sex features, it regulates the proliferative phase of the menstrual cycle, and together with FSH it increases the number of LH receptors on granulosa cells. In low or sustained concentrations, it inhibits LH and FSH secretion (in days 1-13, and 15-28), but on day 14 of the menstrual cycle, its levels surge abruptly and cause an increase in LH secretion. It also has an anti-osteoporotic effect, and a protective role against atherosclerosis. Apart from the follicle, it is also produced in the placenta and in adipose tissues (like testosterone, it is produced there from androgens). The most effective form of estrogen is 17-estradiol. This is converted into estrone. Estriol is the metabolic product of DHEA in pregnant women. Like testosterone, 97% is bound to albumin and SHBG, and the remaining is free. The free form, and to some extent the albumin-bound form, are the active ones. As mentioned, SHBG binds both estradiol and testosterone, but it has a greater affinity for testosterone, and the serum concentration in males is lower. Estrogen is mainly metabolized in the liver (to form estriol), and the conjugated forms are made soluble and thus excreted in the urine. Estrogen is not specific to the ovary, as it is also produced in the adrenal cortex. It is present in both males and females, but the quantities are different. Estrogens inhibit the secretion of GnRH in the hypothalamus, and of LH and FSH in the pituitary (long loop). b. Progesterone is a 21-carbon long hormone produced in the corpus luteum in the ovary. It regulates the secretory phase of the menstrual cycle, prepares the uterus for implantation and maintains pregnancy. With estradiol, between days 15-28 of the menstrual cycle, it inhibits LH secretion. Progesterone is also secreted from the placenta (trophoblast). In the first 4 months, the corpus luteum secretes progesterone and thus maintains pregnancy. Then, the corpus luteum regresses, and the placenta takes over the secretion of progesterone and thus it is now the placenta that maintains pregnancy. Progesterone is transported in the blood bound to CBG (transcortin, with high affinity and low amount) and to albumin (90-98%) and the remaining (2-10%) is free and biologically active. It is metabolized in the liver and the end-products are excreted in the urine. There are 3 major forms: pregnendione, pregnenalone and pregnenediol. Like estrogen, it is produced also in the adrenal cortex, present in both sexes and differs in quantity. Progesterone inhibits the

Avi Sayag

Clinical Biochemistry

secretion of GnRH in the hypothalamus, and of LH and FSH in the pituitary (long loop). c. 66% of androgens (testosterone, DHEA and androstendione) in the female are produced in the adrenal cortex, and 34% in the ovaries. They are necessary for the growth of pubic hair and axillary hair. Here, too, the active form of testosterone is DHT formed by the assistance of 5-reductase. The metabolites (17-oxosteroids) are excreted in the urine. Before puberty, the pituitary is unresponsive to GnRH; thus, FSH and LH levels are low, and therefore estradiol levels are low. During the climacteric (menopause), estradiol levels are low due to ovarian failure, but the levels of FSH and LH are high (not inhibited). First, FSH levels rise, then LH. In menopause, the only source of estradiol is from the metabolism of adrenal androgens, in adipose tissues and in the liver. The increase of FSH concentrations is a better indicator of ovarian failure than the decrease in estradiol levels, which shows considerable variability. Diagnosis of hypo- and hypergonadism has to differentiate between primary and secondary hypo- or hypergonadism. In primary hypogonadism (hypergonadotrophic hypogonadism), estrogen levels are low, but LH and FSH are high; whereas in secondary hypogonadism (hypogonadotrophic hypogonadism) estrogen levels, FSH and LH levels are low. In primary hypergonadism, estrogen levels are high, but FSH and LH levels are low; whereas in secondary hypergonadism, estrogen levels, FSH and LH levels are all high. Female hypogonadism can be symptomatically classified into 3 categories: 1. Amenorrhea; 2. Hirsutism and virilism 3. Infertility and sterility 1. Amenorrhea Primary: menstruation has never occurred until the age of 16. The cause may be in one or more of the following glands: - Hypothalamus: weight loss, intensive exercise, anorexia; - Pituitary: hypopituitarism, prolactinemia - Gonads (most cases): dysgenesis/agenesis/aplasia of ovaries, the Fallopian tube, the uterus, the vagina - Adrenal: congenital adrenal hyperplasia - Thyroid: hypothyroidism Secondary: an absence of menstruation for at least 6 months among women who had previously normal cycles. The causes may be: - Physiological: pregnancy, menopause - Hypothalamus (most cases 80%): weight loss, stress, anorexia, drugs - Pituitary: acquired hypopituitarism, Sheehan syndrome (post-partum pituitary necrosis), hyperprolactinemia (leads to galactorrhea-amenorrhea syndrome) - Ovary: poly-cystic ovary, tumors, inflammation, autoimmune dysfunction - Uterus: tumor or inflammation (rare) - Endocrine cause: DM, hypo- and hyperthyroidism, late onset CAH, Cushing's syndrome - Post-pill amenorrhea: caused by hypothalamus suppression, increased prolactin levels and endometrium atrophy. 2 causes are worth further elaboration: 1. Prolactinemia: galactorrhea-amenorrhea syndrome: Amenorrhea is caused by high concentration of prolactin in the plasma that interferes with the pulsatility of GnRH. Causes include prolactinomas, of which 90% are microadenomas (tumor is <1cm) and 10% are macroadenomas (>1cm in diameter). Obstruction of blood supply to the pituitary gland leads to less dopamine reaching the pituitary, thus less inhibition of prolactin secretion. Drugs

Avi Sayag

Clinical Biochemistry

that block dopamine receptors or deplete dopamine can cause the same effect. Surgery, ectopic secretion and hypothyroidism can also be the cause. Symptoms include galactorrhea, amenorrhea and infertility. Diagnosis should include prolactin determination and TRH stimulation test. Therapy includes administration of bromocriptin (a dopamine agonist) and surgery. 2. Polycystic ovary syndrome: a condition of hyperandrogenization and chronic anovulation in the absence of specific underlying adrenal or pituitary disease. Causes include abnormalities in enzyme synthesis and hypothalamic causes. Elevated androgens may lead to virilism15. Inhibition of FSH (due to increased estrogen) leads to anovulation, and stimulation of LH (due to increased estrogen) increases androgen secretion. Symptoms include amenorrhea and infertility, virilism, acne, obesity, enlargement of the ovaries and cysts in the ovaries with hyperplasia of thecal cells. Diagnosis requires at least 2 of the following features: polycystic ovaries, oligo-ovulation or anovulation, and clinical/biochemical evidence of androgen excess. Lab diagnosis should include elevated levels of testosterone, estrogen, and LH; low levels of FSH, and dexamethasone suppression test to differentiate adrenal origin. Diagnosis of amenorrhea: 1. Exclude pregnancy: high LH levels may suggest pregnancy before a pregnancy test is performed: hCG cross-reacts in some assays with LH. 2. Inspection: obvious signs that might require suspicion of congenital syndromes (and performance of chromosomal analysis). 3. Gynecological examination (US, cytology, biopsy) 4. Measurement of LH, FSH, estradiol: a. In ovarian failure: LH, FSH are elevated, estradiol is low b. In hypothalamus/pituitary insufficiency: LH, FSH and estradiol are low 5. Prolactin measurement 6. Androgen concentration 7. Measurement of thyroid hormones, cortisol and ACTH. There are 6 functional tests that should be considered: 1. Progesterone challenge test: uterine dysfunction is an uncommon cause of amenorrhoea. It can be excluded, if necessary, by the progestogen challenge test. If medroxyprogesterone acetate is given orally (10 mg daily for 5 days), the occurrence of vaginal bleeding 5-7 days later indicates that the uterus was adequately oestrogenized. If bleeding does not occur, the test is repeated, giving oestrogen (ethinyloestradiol, 50 mg daily for 21 days, with progestogen on the last 5 days). Absence of bleeding indicates uterine disease. If bleeding occurs, oestrogen deficiency is present. 2. Estrogen + progesterone challenge test: performed to differentiate between uterus abnormality and ovarian abnormality. Estrogen is administered for 21 days and then progesterone for 7-10 days. Stop medication for 7 days. Interpretation: if no withdrawal of bleeding occurs, it suggests uterine bleeding outflow obstruction; if withdrawal of bleeding occurs (but not after progesterone alone), obtain serum FSH and serum LH if both are elevated, it suggests hypergonadotrophic hypogonadism, but if both are low, it suggests hypogonadotrophic hypogonadism 3. Clomifene citrate test: clomifene binds to hypothalamic receptors and blocks the inhibitory effect of peripheral steroids on GnRH release. Normally, the block of the negative feedback on GnRH causes the increase in GnRH and thus elevation of LH and FSH. A positive response refers to the intact function of the hypothalamuspituitary axis. This test differentiates between primary and secondary hypogonadism. 4. GnRH test: this test is aimed to differentiate between pituitary abnormality and hypothalamus abnormality. 100g of GnRH are administered, and the levels of LH and FSH are measured after 20 minutes and 60 minutes. Normal response is recorded
15

A female disorder in which there is development of secondary male sexual characteristics

Avi Sayag

Clinical Biochemistry

if FSH and LH levels increase, suggesting hypothalamus insufficiency. If the levels of FSH and LH do not increase, it suggests pituitary insufficiency. 5. Gonadotrophin test: FSH and LH (or hCG) are administered to evaluate the ovarian functions. Estrogen and progesterone levels should be elevated if the problem lies outside the ovaries. 6. Dexamethasone suppression test: see Topic 47, "Diagnosis". Treatment of amenorrhea includes replacement therapy with whatever is missing: progesterone, estrogen +progesterone, FSH, LH, GnRH, clomifene citrate. Protocol for investigation of amenorrhea is found in Marshall page 198. 2. Hirsutism and virilism Hirsutism is an increase in body hair. Causes include increased androgen secretion, deceased SHBG and increased androgen sensitivity. 1. Drugs: androgens and progesterone 2. Adrenal: CAH (where there is a decrease in aldosterone and cortisol production and an increase in androgen synthesis); acquired adrenal hyperplasia; androgenproducing tumor; Cushing's syndrome 3. Ovary: polycystic ovary syndrome; androgen-producing tumor; hyperthecosis; post-menopause 4. Familial hirsutism 5. Idiopathic hirsutism Symptoms include menstrual irregularity or amenorrhea, obesity, acnes, infertility, clitoromegaly, deep voice and increased body hair. Diagnosis: 1. Testosterone measurements: should be performed 3 times at 20-minute intervals, due to pulsatile secretion. 2. DHEA, androstendione (to determine adrenal origin) 3. SHBG 4. FSH, LH (LH increases in PCOS) 5. 17-hydroxyprogesterone (CAH) 6. Dexamethasone suppression test: low doses can inhibit ovarian production, but high doses can cause suppression in case of adrenal hyperplasia but not in adrenal tumors. The DD should include 5 conditions differentiated on basis of 5 parameters: 17OHFSH LH DHEA Testosterone progesterone Hirsutism Normal Normal Normal Normal/high Normal/high PCOS Normal Normal/low High Normal High Ovarian tumor Normal Normal/low Normal/low Normal High Adrenocortical Normal Normal/low Normal/low High Normal/high tumor CAH High Normal/low Normal/low High High 3. Infertility and sterility Sterility is defined when pregnancy cannot occur during 2 years of attempts; whereas infertility refers to the ability to get pregnant but pregnancy ends spontaneously (spontaneous abortion). Causes for these conditions include psychological causes, as well as anything in the hypothalamus-vagina axis: hypothalamus, pituitary, ovary, uterus, Fallopian tubes, vagina, androgen predominance, endocrine disorders and alcoholism. Immunological factors can also lead to these conditions (antisperm antibodies). As some causes are reversible (hormonal causes, some anatomical changes and immunological causes) some are irreversible (chromosomal abnormalities, severe congenital abnormalities). Diagnosis should include:

Avi Sayag
1. 2. 3. 4.

Clinical Biochemistry

History taking Gynecological examination Sperm examination Post-coital test: mucus with an adequate estrogen stimulation is clear and thin. Before mucus dries, more than 20 motile sperm should be seen in one HPF (high power field). This test is considered unreliable nowadays. 5. Examination of the cervical mucus: the penetration of the sperm is investigated after the mucus and the sperm are mixed in vitro. 6. Hormone examination: a. Progesterone measurement: its levels peak 5-9 days after ovulation b. Basal body temperature: a rapid increase of 0.5C upon ovulation c. LH surge: 24-36 hours before ovulation d. Estrogen, FSH, PRL, androgens and other dynamic tests

Oral Contraceptives These inhibit ovulation. When the administration is withdrawn, vaginal bleeding resumes. The pills can include estrogen and progesterone (either equally or with different concentrations); it is taken for 21 days with a 7-day break. The first time a woman takes a pill, it is recommended to take it on the first day of bleeding, for 21 days, and then stop for a week during which she bleeds. If she bleeds for more than a week, the pill should be replaced with a more suitable one for her. The other pill contains only progesterone and is taken continuously. Side effects include post-pill amenorrhea, irregular cycles, thromboembolism, VLDL elevation, and cortisol and thyroxine elevation. Absolute contraindications include cardiovascular diseases, liver and renal disorders, and history of thromboembolism; whereas relative contraindications include hormonal disturbances, migraines, DM, smoking, preoperative state and myomas.

Avi Sayag

Clinical Biochemistry

Topic 50
Lab procedures in the diagnosis of bone and skeletal disorders
Some general information about bone tests and markers: Markers of bone formation (osteoblast formation) 1. Osteocalcin: a non-collagen protein of 49 amino acids produced by osteoblasts. Active vitamin D stimulates its production. It promotes bone mineralization by binding calcium after undergoing post-translational carboxylation (a vitamin K-dependent process). It also has a chemotactic and nitrogen effect on osteoclasts (a negative regulator of bone formation, but the mechanism is not fully understood). Serum osteocalcin concentration is a good indicator of osteoblast activity. Its half life in the serum is 5 minutes, and the kidney eliminates it. Its fast proteolysis requires special sampling processing. False elevated results can be obtained by renal insufficiency (less clearance of osteocalcin), immobilization and therapy with active vitamin D (as it stimulates its production). 2. BSAP (bone specific alkaline phosphatase): a tetrameric glycoprotein found on the surface of osteoblast cells. The gene for alkaline phosphatase expresses isoenzymes which are non-tissue specific, intestinal, placental and germ cells. 3 isoforms are derived from the non-specific isoenzymes: liver, bone and kidney, which undergo posttranslational modification (glycosylation) in the specific tissue. The liver isoform travels furthest in electrophoresis, while the intestinal one travels the shortest distance. BSAP has advantages over osteocalcin: its half life is longer (1-2 days), it has no diurnal variation, it is stable in the serum (no proteolysis), it does not require special sampling, transport and storage, and the kidney's function has no effect on its serum concentration. 3. Pro-collagen type I- terminal peptides: collagen type I is the sole collagen type found in bones and tendons. Carboxyterminal propeptide, derived and cleaved from procollagen type I (PICP) during collagen synthesis, and aminotermianl (PINP), are delivered into the blood, where they can be measured. According to current knowledge, PICP correlates with bone collagen synthesis and bone formation rate. However, these peptides are neither specific nor sensitive for bone formation, because collagen type I is formed in other tissues as well. Markers of bone resorption (osteoclast function) 1. Clearance of calcium, hydroxyproline, hydroxylysine and galactosyl-hydroxylysine (the last 3 are part of collagen type 1, and their clearance indicates the degradation of collagen). 2. Pyridine/deoxypyridine crosslaps (measured in the urine) 3. C- and N- telopeptide crosslaps (CTX, NTX): the assay is specific for an octapeptide in the C-terminus of the 1 chain of type 1 collagen and accurately reflects osteoclastmediated bone resorption. 4. Tartrate resistant acid phosphatase (TRAP): in osteoclasts, TRAP is localized within the ruffled border area, within lysosomes, and in Golgi cisternae and vesicles. Its elevation indicates osteoclastic function. There are 4 diseases in the skeleton that are worth mentioning: 1. Paget's disease A chronic disease of the adult skeleton, in which localized areas of bone become hyperactive and are replaced by a softened and enlarged osseous structures. About 3% of persons over the age of 40 suffer from this disease (more males than females). Diagnosis: 1. X-ray: bones with increased bone density, abnormal architecture, cortical thickening, bowing and overgrowth. Mainly localized in the pelvic bones, spine and skull. 2. Lab: se BSAP (bone-specific alkaline phosphatase) is increased, se Ca+2 and phosphate are normal and collagen crosslaps are elevated. 2. Rickets and osteomalacia A metabolic bone disease resulting from vitamin D deficiency (in children it's called Rickets and in adults osteomalacia).

Avi Sayag

Clinical Biochemistry

Causes include: a. Vitamin D deficiency (low UV light exposure, inadequate intake, malabsorption of vitamin D, hepatic 25-hydroxylase deficiency) b. Vitamin D-dependent rickets type 1: 1-hydroxylase deficiency c. Vitamin D-dependent rickets type 2: receptor defect (resistance to the actions of calcitriol (vitamin D)). d. Vitamin D-resistant rickets: X-linked; defective bone mineralization due to an inadequate supply of phosphate. The cause is usually a renal tubular phosphate leak. e. Calcium deficiency: excess loss, malabsorption and inadequate intake f. Drugs: heparin, methotrexate and anticonvulsive drugs. Diagnosis: 1. X-ray: bowing legs, deficient mineralization of bone matrix; in children, there is an inability to mineralize cartilage and remodel new bone at the epiphyseal growth plates. 2. Lab: se BSAP is elevated, calcium and phosphate are low. 3. Renal osteodystrophy Pathogenesis:

Diagnosis: serum phosphate, BSAP and creatinine are all elevated; serum calcium and vitamin D are low. 4. Osteoporosis Osteoporosis is characterized by reduced bone mass and abnormalities of bone microarchitecture, which render it more fragile and susceptible to fracture. It is defined as a bone mineral density >2.5 standard deviations below the mean for young people or by the occurrence of a typical fracture. The lifetime risk of fracture due to osteoporosis is about 40% in women and 15% in men; they are particularly likely to occur in the proximal femur (both sexes) and in the vertebral bodies (to a much greater extent in women). Fractures of the distal radius (Colles' fracture) are also more common in women. Risk factors are associated with female sex, age (menopausal women), immobility, hypogonadism, alcoholism, chronic renal disease, RA and cortisol therapy. Diagnosis is based on bone densitometry and increased CTX (collagen crosslaps). Secondary osteoporosis is related to hyperthyroidism, hypogonadism, multiple myeloma and chronic liver disease. Hyperthyroidism: test TSH, T3 and T4 Hypogonadism: test serum testosterone (estrogen) MM: test ESR, serum protein electrophoresis and serum calcium Chronic liver disease: test GOT, GPT, GGT and ALP.

Avi Sayag

Clinical Biochemistry

Topic 51
Lab procedures in the diagnosis of muscle disorders
There are 3 main muscle disorders to address in this topic: periodic paralysis, malignant hyperthermia and muscular dystrophies. 1. Periodic paralysis: Periodic paralysis is a group of rare genetic diseases that lead to weakness or paralysis (rarely death) from common triggers such as cold, heat, high carbohydrate meals, not eating, stress or excitement and physical activity of any kind. The underlying mechanism of these diseases are malfunctions in the ion channels in skeletal muscle cell membranes that allow electrically charged ions to leak in or out of the muscle cell, causing the cell to depolarize and become unable to move (a channelopathy). The attacks can be hypo- or hyperkalemic, depending on the inducing trigger. In the hypokalemic form, K+ flows into the myocytes from the bloodstream, while in the hyperkalemic form K+ leaks out of the myocytes into the blood. Hypokalemic Hyperkalemic Induced by Glucose (insulin causes reduction of K+) KCl or exercise Duration Hours 30-40 minutes Serum K+ <3 mmol/l Increased Urine K+ Decreased Normal inheritance AD (chromosome 1) AD (chromosome 17) 2. Malignant hyperthermia: Malignant hyperthermia is a rare life-threatening AD condition that is triggered by exposure to certain drugs used for general anesthesia and the neuromuscular blocking agent succinylcholine. The defect lies in the ryanodine receptor gene (Ca+2 channel). In susceptible individuals, these drugs can induce a drastic and uncontrolled increase in skeletal muscle oxidative metabolism, which overwhelms the body's capacity to supply oxygen, remove CO2 (leading to respiratory acidosis), and regulate body temperature (heat in the muscle is increased as well as the body temperature), eventually leading to circulatory collapse (shock) and death if not treated quickly. Therefore, Lab tests may reveal: i. Acidosis (pH<7.2) ii. Increased lactic acid iii. CK, AST, LD greatly increased iv. Myoglubinuria v. Tubular necrosis vi. Acute renal failure Screening can be done by measuring a 100-fold increase in CK levels. The diagnosis should rest on muscle biopsy and molecular biology (Ryanodin receptor defect) Muscular dystrophies: A large group of inherited progressive muscle weakness diseases that can be inherited as AD, AR or X-linked. The lab diagnosis is made with a combination of DNA, RNA and protein analysis to identify the molecular defect and to distinguish between the many forms of MDs that have an overlapping phenotype. MD involve problems in the dystrophin-glycoprotein complex. In a nutshell, dystrophin, actin and syntrophin ( and 1) are located intracellularly; sarcoglycans (,,,) and dystroglcans are transmembrane structures; -dystroglycan and 2-laminin (aka merosin connects dystroglycans to integrins in the basal lamina) are extracellularly located. The dystrophin gene (Xp21) is a very large gene (100 times larger than average). Its product (dystrophin) is produced by alternative splicing yielding at least 7 tissue specific isoforms (muscle, brain, retina, kidney, etc.). The muscular type is 175 nm long, with 4 domains: N terminal binds actin, the 2nd domain is proline rich and is similar to -actinin, the 3rd domain is cystein-rich and binds dystroglycan, and the C terminal binds syntrophins. It functions in assembling and maintaining the complex, and it connects contractile element within the cell to merosin located extracellularly.

Avi Sayag

Clinical Biochemistry

Dystrophin-related disorders: Duchenne and Becker MD BMD (Becker) Still ambulant at age 16 No mental deficiency Slower progression Approx. 5:100,000 males 50-100-fold increase in CK Immunostaining of muscle biopsyinterrupted staining of dystrophin Immunoblotting- altered dystrophin (larger or truncated) Deletion/duplication/point mutations of genes Deletion/duplication/point mutations of genes No reading frame alteration Reading frame alteration (frame shift) X- linked X- linked Diagnosis: all those in bold are used for diagnosis. Deletion screening is done using multiplex-PCR in which more than one pair of primers used to amplify and detect dystrophin deletions (deletions account for 60% of cases, point mutation for 35% and duplications for 5%). MLPA (multiplex ligation dependent probe amplification) can also be used. It is one of the best quantitative methods to detect small differences. The method uses a pair of probes that are amplified only when both are attached AND ligated to each other. The results are given in a graph, and the peak intensities are compared and analyzed. Prenatal testing is performed at the 11th week when 30 mg of villi are taken to determine the sex of the fetus (by cytogenetics). If it is a male, then further examinations are carried on the same biopsy. In case of a male fetus, multiplex PCR/MLPA are carried out, and the results are obtained in 1-2 weeks. Other MD: Limb Girdle MD- LGMD: inherited as AD/AR, 10-120-fold increase in CK, mainly with sarcoglycans ( type is deficient mostly) Merosin deficiency: AR, no merosin, no mental problems. Oculopharyngeal MD: AR/AD, repeat of GCG creating stabile fibril, which accumulates in the nuclei causing cell death. It is prevalent in French-canadian and Bukharik Jews in Israel.
If SG are normal, merosin should be tested (merosin def. MD) and calpain (calpainopathy). However, if both are normal, then Emerin immunostain should be performed. A deficient Emerin indicates Emerin-Dreiffus

DMD (Duchenne) Wheelchair by age 12 Reduction in IQ Sarcolemma destroyed Approx. 28:100,000 males 50-100-fold increase in CK Immunostaining of muscle biopsy- dystrophin absent Immunoblotting- no protein detected

Avi Sayag

Clinical Biochemistry

Topic 52
Clinical biochemistry at the extremes of age
Lab values are greatly affected by some factors such as: 1. Age 2. Sex 3. Race 4. Circadian changes 5. Posture 6. Pregnancy 7. Altitude The reference ranges of normal adults would not apply to the values received from newborns, children during puberty and the elderly. Infants and neonates Testing neonates, there are some problems to confront. Neonates have a limited amount of blood (up to 300 ml) and immature ones have even less (100-200 ml). The reference intervals are different for them, and differ even further for whole blood samples and for serum ones. Lastly, it is problematic to get a neonate to fast. Changes in lab parameters which are age relateda. Glucose: from 1.6-3.3 mmol/l in neonates to 3.9-5.6 mmol/l in healthy adults (caused by small amount of glycogen stores and adrenal immaturity) b. Bilirubin: in the cord <34 mol/l and rises up to <273 mol/l on days 3-5 and then falls again. Adult values: <17 mol/l. c. Urea: 3.5-6.6 mmol/l in the cord and drops to 0.7-2 mmol/l in the neonate and starts to rise again (due to increased protein synthesis). Adult values: 2.9-8.2 mmol/L. d. Leukocyte count: high at birth (18.1 G/L), and drops with the years to 7.8 G/L (at 16 years of age). e. Other changes can be seen also in MCV, Hb, PLT, neutrophils, lymphocytes, monocytes and reticulocytes. Elderly Testing the elderly, there are also some problems to confron. Similar to neonates, the reference range is different. There are prevalent diseases to take into consideration, and some of them present as atypical form of the disease. Lastly, most elderly patients are taking medications that might interfere with the lab results. 1. Endocrinological changesa. Estrogen/testosterone decrease b. FSH/LH increase c. Hypothyroidism (T3 decreases up to 40% over age 40) d. Peripheral blood glucose uptake is decreased and the Tm for glucose is reduced as well. e. PTH decreases f. Aldosterone decreases up to 50% 2. Renal concentrating capacity decreases. 3. Comparison (also see Topic 34 for further details of the normal ranges): Cholesterol (mmol/l) Normal: <5.2 Triglycerides (mmol/l). Normal: 1.35 Newborn 1.37-3.5 0.36-1.12 Adolescent 3.11-5.44 0.45-1.81 Elderly 3.63-8.03 0.62-2.79 For both the elderly and neonates, CSF values are also different (protein level in CSF can reach up to 0.9g/L, while in adults it is 0.15-0.45g/L).

Avi Sayag

Clinical Biochemistry

Topic 53
Clinical biochemistry and lab diagnostics of porphyrias
Heme synthesis begins when glycine and succinylCoA combine to form AminoLevulinic Acid (catalyzed by ALA synthase). Then, 2 ALAs condense to form porphobilinogen (PBG, catalyzed by PBG synthase), and 4 PBGs condense to form the first porphirins. Porphyrias are a group of inherited diseases in which a partial deficiency of one of the enzymes of porphyrin synthesis leads to decreased formation of heme and thus, by releasing ALA synthase from inhibition, results in the formation of excessive quantities of porphyrin precursors (ALA and PBG) or porphyrins (heme, the end product, normally inhibits ALA synthase). When precursors are produced in excess, the clinical manifestations are primarily neurological (the precursors are neurotoxins). When porphyrins themselves are the major product, the predominant feature is photosensitivity: the porphyrins absorb light and become excited, inducing the formation of toxic free radicals. The porphyrias are classified as acute or chronic, according to their clinical presentation, and hepatic or erythropoietic, depending on the major site of abnormal metabolism. The 3 acute forms are acute intermittent porphyria, hereditary coproporphyria and variegate porphyria. All acute forms are hepatic. The chronic forms are porphyria cutanea tarda (hepatic), and congenital erythropoietic porphyria and erythropoietic protoporphyria (both are erythropoietic). All forms are AD except congenital erythropoietic porphyria and ALA dehydratase deficiency porphyria (an hepatic form). All hepatic acute forms are characterized by neurological symptoms, and all the chronic forms are characterized by photosensitivity. Acute intermittent porphyria: The defect lies in HMB synthase (aka PBG deaminase). The inheritance is AD with over 60 mutations described. Women are affected more than men but around 90% of carriers are asymptomatic. The acute attacks, which begin over several hours, may last between days to weeks, and are followed by complete remissions. Some factors may precipitate the attacks (e.g. anesthesia, drugs, alcohol, stress, pregnancy and infection). The heme precursors are neurotoxic and cause vascular damage. Symptoms usually begin around puberty and include GI symptoms (vomiting, constipation), peripheral neuropathy (muscle weakness and numbness), seizures, depression, hysteria and psychosis, sinus tachycardia and systemic hypertension. Recall that there is no photosensitivity. Diagnosis includes: i. Urinary PBG during an attack ii. Quantitative analysis of other precursors in the urine (e.g ALA) iii. The results might be normal if the patient is not in the midst of an attack Treatment includes control of the fluid and electrolytes balance, IV glucose, pain relief if needed, IV hamatin (which decreases ALA synthase action) and avoiding the precipitating factors. Porphyria cutanea tarda: The most common cutaneous porphyria caused by a decreased activity of uroporphyrinogen decarboxylase enzyme in the liver. 15-20% are inherited (type 2), while the sporadic cases (type 1) occur among alcoholics or people who consume drugs/other heptotoxic substances. Photosensitivity dominates the clinical picture (due to accumulation of porphyrins) and it manifests in photodermatitis, blisters, necrosis of the skin, itching, dark urine after an exposure to light (when the porphyrins are excreted) and red teeth in case they accumulate there. In contrast to AIP, there are no neuro-visceral symptoms and the porphyrins can be found in the urine AS WELL AS in the stool. Congenital erythropoietic porphyria: The defective enzyme is the uroporphyrinogen synthase in RBC. Photosensitivity is very high in comparison to the other types mentioned. The porphyrins bind to the dental Ca+ leading to their red color in fluorescent light (erythrodontia). In contrast to AIP, there are no neurovisceral symptoms, the porphyrins are excreted in BOTH the urine and stool, and accumulate within the RBC themselves.

Avi Sayag

Clinical Biochemistry

Topic 54
Lab diagnostics of CNS diseases; lab investigation of the CSF
While the diagnosis of some metabolic neurological disorders requires complex investigations that will usually be performed only in specialist laboratories, simple investigations that should be in the repertoire of most laboratories are frequently required in the initial investigation of patients presenting with neurological signs and symptoms. While most of these investigations are performed on plasma or serum, assays of CSF are also valuable in certain conditions: CSF sampling is indicated in cases of meningeal infection, subarachnoid hemorrhage (SAH), CNS malignancy and a demyelinating disease. The results should be correlated with the plasma values, with the clinical symptoms and with the radiological findings. The CSF can be sampled from the lumbar region (anywhere below L1, preferably L3-L4). Alternative methods of CSF collection are rarely used, but may be necessary if the person has a back deformity or an infection. Cisternal puncture uses a needle placed below the occipital bone. It is always done with fluoroscopy. Ventricular puncture is even more rare, but may be recommended in people with possible brain herniation. This test is usually done in the operating room. A hole is drilled in the skull, and a needle is inserted directly into one of brain's ventricles. CSF may also be collected from a tube that is already placed in the fluid, such as a shunt or a venitricular drain. These sorts of tubes are usually placed in the intensive care unit. The opening pressure in adults reaches 180mmH2O, while that of infants reaches only 10-100 mmH2O. Blood should be drawn 2-4 hours before sampling the CSF. Once the CSF needle is inserted, 5 tubes are collected: tube 1- cell count; tube 2 Stat gram stain and culture; tube 3 glucose and protein; tube 4 cell count (for comparison to tube 1); tube 5 virology, mycology, cytology, etc. (an optional tube). Those tubes destined to cell count procedure need not be centrifuged, while those sent to immunochemical investigations are centrifuged. The examination of the CSF starts with a gross examination: healthy patients have a colorless CSF sample, and its viscosity is similar to water. A viscous sample may indicate metastatic mucin-secreting adenocarcinoma or cryptococcal meningitis. Also, a clot can sometimes be seen, which implies a traumatic tap, suppurative and tuberculous meningitis or spinal block. Note that a clot does not suggest SAH. Lastly, following hemorrhage into the CSF, red cells undergo lysis and phagocytosis, and the oxyhemoglobin that is released is slowly converted into bilirubin (and sometimes methemoglobin). In some cases, the bilirubin imparts a yellow colour (xanthochromia) to the CSF that is visible to the naked eye. Scanning of apparently colourless CSF for xanthochromia may help to distinguish SAH from a traumatic tap. The next step is a microscopic examination: Cells Fuchs-Rosenthal chamber is used for cell counting. In adults, there are 0-5 cells/l, while infants have 0-30 cells/l. The differential for leukocytes is also obtained. As known, the differential of neutrophils in the serum is around 60%. In the CSF, however, it is 2% in adults and 3% in newborns. Increased neutrophils indicate meningitis (mainly bacterial), CNS hemorrhage or infarction, foreign materials in the SAS (such as contrast media), and metastatic tumor in contact with the CSF. Lymphocytes in the CSF are 60% (+/-30) in adults and 20% (+/-18) in newborns. In the serum they are 20-40%. Increased lymphocytes in the CSF suggests meningitis (mainly nonbacterial), degenerative disorders (e.g. MS, Guillain-Barre) and other conditions such as sarcoidosis and polyneuritis. Proteins More than 80% of proteins in the CSF are derived from the plasma, and most are albumin. The normal reference range in the CSF is 0.15-0.45 g/L (while in the plasma it is 60-80g/L). Note that neonates and the elderly usually have higher values (up to 0.9g/L). CSF protein level can be increased in cases of BBB malfunction, decreased resorption, mechanical obstruction and elevated intrathecal Ig synthesis. Decreased levels can be found in youns

Avi Sayag

Clinical Biochemistry

children (not neonates) 0.5-2 years old, when the turnover of CSF is increased and in hyperthyroidism. Albumin: albumin is 230 times more abundant in the serum than in the CSF (the ratio between albumin in the CSF and in the serum is 1:230). As long as this ratio is below 9, the barrier is intact. A slight impairment is suggested by a ratio of 9-14, and it becomes a moderate one when it rises to 14-30. The barrier impairment is said to be severe when the ratio is 30-100. The ratio between CSF Ig and serum Ig is also an indicator. Normally, it is kept between 3-8. It significantly increases in MS where there is an increased intrathecal Ig synthesis. S-100B: this is a calcium-binding protein found in Schwann cells, astroglia, melanocytes and others. It is found both in the serum (<0.15g/L) and in the CSF (<5g/L). When hypoxia occurs, the glia cells are damaged, protein S-100B leaks to the intercellular space and the CSF, and as the BBB is damaged, it reaches the serum. Cranial trauma results in increased levels by the same mechanism. The levels of S-100B are elevated in viral encephalitis, less so in bacterial meningitis and even less in viral meningitis. CRP: elevated CRP in the CSF is highly suggestive of bacterial meningitis (the predictive negative value of a low result ruling out bacterial meningitis is 97%). Enzymes Neuron specific enolase: this is a prognostic marker after cerebral infarct and intracerebral hemorrhages. In the serum it is found in less than 12.5g/L, and in the CSF it is 2.5 times more (<32g/L). LDH: should be less than 40 U/L CK: normally less than 5 U/L and usually the CK-BB isoforms is present. Its levels increase in demyelinating diseases, CVA, brain malignancies, meningitis and head injuries. Glucose The normal value is 60% of that in the plasma. Lower levels suggest meningitis (bacterial, TB, fungal and sometimes viral), and other conditions involving the meninges (SAH, tumors, sarcoidosis). During recovery from meningitis, the glucose levels normalize before these of proteins. Ammonia: the levels should be 30-50% of that in the plasma. Its elevation is proportional to the degree of hepatic encephalopathy. -ketoglutarate protects the CNS by binding NH3 to form glutamine. There are 5 conditions to address: 1. CSF leakage: diagnosed by clinical symtoms (otorrhea, rhinorrhea), lab tests for glucose, S-100B and protein electrophoresis with immunofixation for transferrin. Radiological examinations will complete the picture. 2. CVA: lactate, CK-BB, LDH, S-100B and NSE are all elevated. 3. Tumors: CRP is elevated, neutrophils are elevated, tumor cells are present in a microscopic slide, electrophoresis shows a monoclonal band, and tumor markers are detected (AFP, CEA, hCG and NSE). Flow cytometry is used to characterize the cells. 4. SAH: xanthochromia is detected, and D-dimers, LDH, CK-BB and S-100B are all elevated. 5. Meningitis: total protein, lactate, LDH, Ig, CRP and S-100B are elevated (more in bacterial than in viral meningitis) and glucose is reduced (more in bacterial than in viral meningitis).

You might also like